Functions

You might also like

Download as pdf or txt
Download as pdf or txt
You are on page 1of 76

FUNCTION

Preface

As you have gone through the theory part that consists of given fundamental principles,
definitions, concepts involved and solved problems. After going through theory part it
becomes necessary to solve the unsolved problems based on the concepts given. To
solve this purpose we are providing exercise part that comprises of various exercises
based on the theory. By solving various kinds of problems you can check your grasp on
the topic and can determine whether you have been able to find optimum depth in relevant
topic or not.

Students are advised to solve the questions of exercises (Levels # 1, 2, 3, 4) in the same
sequence or as directed by the faculty members, religiously and very carefully.

Level # 3 is not for foundation course students, it will be discussed in fresher & target
courses.

The list of exercises is as following :

Total No.of questions in Function are -

Level # 1 ........................................ 109


Level # 2 ........................................ 49
Level # 3 ........................................ 35
Level # 4 ........................................ 45

Total No. of questions......................................................... 238

Corporate Office: CP Tower, Road No.1, IPIA, Kota (Raj.), Ph: 0744-2434159 FUNCTION 1
LEVEL # 1
Questions
inequation Q.8 If x2 – 1  0 and x2 – x – 2  0, then x line in the
based on
interval/set
(A) (–1, 2) (B) (–1, 1)
2 (C) (1, 2) (D) {– 1}
Q.1 The inequality < 3 is true, when x belongs to-
x
2   2 Questions
Definition of function
(A)  ,   (B)     based on
3   3

2  Q.9 Which of the following relation is a function ?


(C)  ,    (–, 0) (D) none of these (A) {(1,4), (2,6), (1,5), (3,9)}
3  (B) {(3,3), (2,1), (1,2), (2,3)}
(C) {(1,2), (2,2,), (3,2), (4,2)}
(D) {(3,1), (3,2), (3,3), (3,4)}
x4
Q.2 < 2 is satisfied when x satisfies-
x 3 Q.10 If x, y  R, then which of the following rules is
(A) (–, 3) (10, ) (B) (3, 10) not a function-
(C) (–, 3) [10, ) (D) none of these (A) y = 9 –x2 (B) y = 2x2
(C) y = x – |x| (D) y = x2 + 1
x7
Q.3 Solution of > 2 is-
x3 Questions
(A) (–3, ) (B) (–, –13) based on Even and odd function
(C) (–13, –3) (D) none of these
Q.11 Which one of the following is not an odd
2x  3 function -
Q.4 Solution of  3 is- (A) sin x (B) tan x
3x  5
(C) tanh x (D) None of these
 12   5 12 
(A) 1,  (B)  , 
 7  3 7  sin4 x  cos 4 x
Q.12 The function f(x) = is -
 5 12  x  tan x
(C)   ,  (D)  ,   (A) odd
 3 7 
(B) Even
(C) neither even nor odd
Q.5 Solution of (x – 1)2 (x + 4) < 0 is-
(D) odd and periodic
(A) (–, 1) (B) (–, –4)
(C) (–1, 4) (D) (1, 4) Q.13 A function is called even function if its graph is
symmetrical w.r.t.-
Q.6 Solution of (2x + 1) (x – 3) (x + 7) < 0 is- (A) origin (B) x = 0
(C) y = 0 (D) line y = x
 1  1 
(A) (– , –7)    , 3  (B) (– , – 7)   ,3 
Q.14 A function is called odd function if its graph is
 2  2 
symmetrical w.r.t.-
 1  (A) Origin (B) x = 0
(C) (–, 7)    , 3  (D) (–, –7)  (3, ) (C) y = 0 (D) line y = x
 2 
Q.15 The even function is-
Q.7 If x2 + 6x – 27 > 0 and x2 – 3x – 4 < 0, then- (A) f(x) = x2 (x2 +1) (B) f(x) = sin3 x + 2
(A) x > 3 (B) x < 4 (C) f(x) = x (x +1) (D) f(x) = tan x + c
7
(C) 3 < x < 4 (D) x =
2

Corporate Office: CP Tower, Road No.1, IPIA, Kota (Raj.), Ph: 0744-2434159 FUNCTION 2
Q.16 A function whose graph is symmetrical about Q.25 In the following which function is not
the y-axis is given by- periodic-
(A) tan 4x (B) cos 2x
(A) f(x) = loge (x + x2  1 )
(C) cos x2 (D) cos2x
(B) f(x + y) = f(x) + f(y) for all x, y  R
(C) f(x) = cos x + sin x Domain, Co-domain and range
of function
(D) None of these
1
Q.17 Which of the following is an even function ? Q.26 Domain of the function f(x) = is-
x
x2
a 1
(A) x (B) tan x (A) R (B) (–2,  )
ax  1 (C) [2,  ] (D) [0,  ]
ax  a x ax  1
(C) (D) Q.27 The domain where function f(x) = 2x2 – 1 and
2 ax  1
g(x) = 1 – 3x are equal, is-
Q.18 In the following, odd function is - (A) {1/2} (B) {2}
(A) cos x2 (B) (ex + 1)/(ex – 1) (C) {1/2,2} (D) {1/2,-2}
(C) x – |x|
2
(D) None of these
3x
Q.28 The domain of the function log is-
Q.19 The function f(x) = x – |x| is -
2 2
(A) an odd function (A) (3,  ) (B) (–  ,3)
(C) (0,3) (D) (–3,3)
(B) a rational function
(C) an even function Q.29 Domain of the function cos–1 (4x –1) is-
(D) None of these (A) (0,1/2) (B) [0,1/2]
(C) [1/2,2] (D) None of these
Questions
based on Periodic function Q.30 Domain of the function log |x2 – 9| is-
(A) R (B) R– [–3,3]
(C) R – {–3,3} (D) None of these
Q.20 The period of sin4 x + cos4 x is -
(A)  (B) /2
Q.31 The domain of the function-
(C) 2 (D) None of these
Q.21 The period of function |cos 2x| is - f (x) = x  1 + 6  x is-
(A) (B) /2 (A) (1,6) (B) [1,6]
(C) 4 (D) 2 (C) [1,  ) (D) (–  ,6]

F
Gx I
J Fx I
+ cos G Jis- The domain of the function f(x) = (2  2x  x2 )
Q.22 The period of function sin
H2 K H2 K Q.32
is -
(A) 4 (B) 6
(C) 12 (D) 24 (A) – 3 x  3
Q.23 The period of the function (B) – 1– 3  x  –1 + 3
f(x) = log cos 2x + tan 4x is -
(C) – 2  x  2
(A) /2 (B) 
(C) 2 (D) 2/5 (D) –2 + 3  x  –2– 3
1
Q.24 The period of the function f(x) = 2 cos (x–) Q.33 Domain of a function f(x) = sin–1 5x is-
3
F
G IJ
1 1 L
M O
1 1
N P
is -
(A) 6 (B) 4 H K
(A)  ,
5 5
(B)  ,
5 5Q
(C) 2 (D)  F
G I
HJ
1
(C) R (D) 0,
K
5

Corporate Office: CP Tower, Road No.1, IPIA, Kota (Raj.), Ph: 0744-2434159 FUNCTION 3
Q.34 If f : R+  R, f(x) = log x, then range of f is -
(A) R0 (B) R Q.43 The range of f : R+  R+, f(x) = ex is -
(C) R+ (D) None of these (A) (0,  ) (B) [1,  )
(C) (1,  ) (D) None of these

Q.35 The range of the function f : R  R, f(x) = tan–1 x Q.44 The range of f(x) = cos 2x – sin 2x contains
is- the set -
L
 
M O O
 L (A) [2,4] (B) [–1,1]
N P
(A)  ,
2 2Q P
Q M
(B)  ,
2 2N (C) [–2,2] (D) [–4,4]
(C) R (D) None of these | x|
 Q.45 If the domain of the function f(x) = be
x
Q.36 The range of f(x) = sin [x] is -
2 [3,7] then its range is-
(A) {–1,1} (B) {–1,0,1} (A) [–1,1] (B) {–1,1}
(C) {0,1} (D) [–1,1]
(C) {1} (D) {–1}

| x  3|
Q.37 Domain and range of f(x) = are
x3 1
respectively- Q.46 The domain of the function f(x) = is-
x  [ x]
(A) R, [–1,1] (B) R– {3}, {1,–1}
(C) R+, R (D) None of these (A) R (B) R–Z
(C) Z (D) None of these
Q.38 The domain of the function f(x) = sin 1/x is - Q.47 The range of the function
(A) R (B) R+
f(x) = 2 + x – [x–3] is-
(C) R0 (D) R– (A) [5,6] (B) [5,6)
(C) R (D) None of these
Q.39 Range of the function f(x) = 9 – 7 sin x is-
(A) (2,16) (B) [2,16]
Questions
(C) [–1,1] (D) (2,16] based on Value of function
Q.40 For real values of x, range of function
1 Q.48 If f(x) = log x, then f (x/y) equals-
y= is - (A) f(x) + f(y) (B) f(x) – f(y)
2  sin 3 x
(C) f(x) / f(y) (D) f(x) . f(y)
1 1
3   3  
(A) y 1 (B) – y 1
2x
Q.49 If f(x) =
1  x2
, then f (tan  ) equals-
1 1
(C) – > y > – 1 (D) > y > 1
3 3 (A) cot 2  (B) tan 2
(C) sec 2 (D) cos 2 
R
S 1, when x  Q
Q.50 If f(x) = ax, then f(x+ y) equals-
 R, f(x) =
Q.41 If f : R
T1, when x Q , then (A) f(x) + f(y) (B) f(x) – f(y)
image set of R under f is - (C) f(x) f(y) (D) f(x) /f(y)
(A) {1,1} (B) (–1,–1)
(C) {1,–1} (D) None of these Q.51 If f(x) = log x, then correct statement is-
(A) f(x + y) = f( x ) + f(y) (B) f(x + y) = f( x) . f(y)
Q.42 If f : R  R, f(x) = x2, then {x| f (x) = –1} equals- (C) f(xy) = f(x) + f(y) (D) f(xy) = f( x) . f(y)
(A) {–1,1} (B) {1}
(C)  (D) None of these x f (a / b)
Q.52 If f (x) = , then =
x1 f (b / a)
(A) ab (B) a/b
(C) b/a (D) 1

Corporate Office: CP Tower, Road No.1, IPIA, Kota (Raj.), Ph: 0744-2434159 FUNCTION 4
Q.53 If f(x) = 2 cos x + sin2 x, then f(2– x) equals- f ( xy)  f ( x / y)
(A) – f(x) (B) f(x) Q.61 If f(x) = cos (log x), then equals-
(C) – 2f(x) (D) 2f(x) f ( x)f ( y)
(A) 1 (B) –1

If f : R  R, f(x) =
R
S1, when x  Q
(C) 0 (D) 2
Q.54
T1, when x Q , then which
 Q.62 If f (x) = |x| + |x – 1|, then for 0 < x < 1, f (x)
of the following statement is wrong ? equals-

(A) f  2 = –1 (B) f() = –1


(A) 1
(C) 2x + 1
(B) –1
(D) 2x – 1

(C) f(e) = 1 (D) f d4 i = 1 | x|


Q.63 The function f(x) = , x > 0 is -
x
F
GI
H3 J
(A) 0 (B) 1
Q.55 If f(x) = 2 sin x, g(x) = cos2x, then (f + g)
K= (C) 2 (D) –2

2 3 1
(A) 1 (B) Q.64 If f : N  R+, f(x) = x , then the value of
4
f (25)
1 is -
(C) 3 + (D) None of these f (9)  f (16)
4
(A) 0 (B) 1
Q.56 If f : R  R , f(x) = 2x ; g : R  R, g(x) = x + 1, (C) 5/7 (D) 9/7
then (f .g) (2) equals -
(A) 12 (B) 6 Q.65 If f(x) = log ax, then f(ax) equals-
(C) 3 (D) None of these
(A) f(a) f(x) (B) 1+ f(x)
b( x  a) a( x  b) (C) f(x) (D) a f(x)
Q.57 If f(x) = + , then f(a + b) =
(b  a) (a  b)
Q.66 If f(x) = (ax – c)/(cx – a) = y, then f(y)
(A) f(a). f(b) (B) f(a) – f(b)
equals-
(C) f(a) /f(b) (D) f(a) + f(b)
(A) x (B) 1/x
x f (a) (C) 1 (D) 0
Q.58 If f( x) = then is equal to -
x1 f (a  1)
(A) f(–a) (B) f(1/a) Questions
Mapping
based on
Fa I
(D) f G J
(C) f(a2)
Ha  1K Q.67 If f : I  I,f (x) = x3+ 1, then f is -
(A) one - one but not onto
x( x  1) (B) onto but not one-one
Q.59 If f (x) = , then the value of f (x + 2) is-
2 (C) One-one onto
(D) None of these
( x  2)
(A) f (x) + f(x + 1) (B) f(x + 1)
x Q.68 Function f : R  R , f(x) = x |x| is -
(A) one-one but not onto
( x  1) ( x  2)
(C) f(x +1) (D) f(x +1) (B) onto but not one- one
2 2 (C) one-one onto
(D) neither one-one nor onto
Q.60 If f(x + ay, x – ay ) = axy, then f (x,y) equals-
x2
2
x y 2 2
x y 2 Q.69 f:R  R , f(x) = , is -
(A) (B) 1 x2
4 4 (A) many- one function (B) odd function
(C) x2 (D) y2 (C) one- one function (D) None of these

Corporate Office: CP Tower, Road No.1, IPIA, Kota (Raj.), Ph: 0744-2434159 FUNCTION 5
Q.80 Which of the following function is onto ?
1
Q.70 If f : R0  R0, f(x) = , then f is - (A) f : R  R ; f(x) = 3x
x
(A) one-one but not onto (B) f : R  R+; f(x) = e–x
(B) onto but not one-one (C) f: [0,  /2]  [–1,1]; f(x) = sin x
(C) neither one-one nor onto (D) f : R  R: f(x) = cosh x
(D) both one-one and onto
Q.81 Which of the following function defined from
Q.71 Function f : R  R, f(x) = x + |x| is R to R is onto ?
(A) one-one (B) onto (A) f(x) = |x| (B) f(x) = e–x
(C) one-one onto (D) None of these
(C) f(x) = x3 (D) f(x) = sin x.
O
 3 L
P
2 2 M
,
Q.72 Function f :
Q N R, f(x) = tan x is Q.82 If f :   , f(x) = x2 – x, then f is -
(A) one-one (B) onto (A) one-one onto (B) one-one into
(C) one-one onto (D) None of these
(C) many-one onto (D) many-one into
L
M 3 O
Q.73 Function f :
N2 , 2 P
Q [–1,1], f(x) = sin x is - Questions
(A) one-one (B) onto based on Composite function
(C) one-one onto (D) None of these
Q.83 If f(x) = 2x and g is identity function, then-
L1 3 O [–1,1], f(x) = cos x is
Function f : M,  P
(A) (fog) (x) = g(x) (B) (g + g) (x) = g(x)
Q.74
N2 2 Q (C) (fog) (x) = (g + g) (x) (D) None of these
(A) many-one onto (B) onto Q.84 gof exists, when-
(C) one-one onto (D) many one into (A) domain of f = domain of g
Q.75 If f : R  R, f(x) = ex + e–x, then f is - (B) co-domain of f = domain of g
(C) co-domain of g = domain of g
(A) one-one but not onto
(D) co-domain of g = co-domain of f
(B) onto but not one-one
(C) neither one-one nor onto Q.85 If f : R  R, f(x) = x2 + 2x – 3 and g : R  R,
(D) both one-one and onto g(x) = 3x – 4 , then the value of fog (x) is-
Q.76 If f : R  [–1,1], f(x) = sin x, then f is - (A) 3x2 + 6x – 13 (B) 9x2 –18x + 5
(C) (3x– 4) + 2x – 3
2
(D) None of these
(A) one-one onto (B) one-one into
(C) many-one onto (D) many-one into
Q.86 If f : R  R, f(x) = x2 – 5x + 4 and g : R  R,
Q.77 If f : R  R , f(x) = sin2 x + cos2 x , then f is - g(x) = log x , then the value of (gof) (2) is -
(A) one-one but not onto (A) 0 (B) 
(B) onto but not one-one (C) –  (D) Undefined
(C) neither one-one nor onto Q.87 If f : R+  R+,f(x) = x2+ 1/x2 and g : R+  R+,
(D) both one-one onto
g(x) = ex then (gof) (x) equals-
Q.78 Which of the following functions from Z to itself x2 1
are bijections ? (A) e x2  e x 2 (B) e  x2
(A) f(x) = x3 (B) f(x) = x + 2 e
(C) f(x) = 2x + 1 (D) f(x) = x2 + x (C) e2 x  e 2 x (D) e x2 . e x2
Q.79 Which of the following functions from Q.88 If f : R  R, g : R  R and f(x) = 3x + 4 and
A = {x: –1  x  1} to itself are bijections ? (gof) (x) = 2x – 1, then the value of g(x) is -
F
Gx I (A) 2x – 1 (B) 2x – 11
H2 J
x
(A) f(x) =
2
(B) g(x) = sin
K (C)
1
(2x – 11) (D) None of these
(C) h(x) = |x| (D) k(x) = x 2 3

Corporate Office: CP Tower, Road No.1, IPIA, Kota (Raj.), Ph: 0744-2434159 FUNCTION 6
Q.89 If f : R  R, g : R  R and g(x) = x + 3 and R
S1, when x  Q
(fof) ( ) will be-
(fog) (x) = (x + 3)2, then the value of f(–3) is -
(A) –9 (B) 0
Q.98 If f(x) =
(A) 2
T0, when x Q , then
(B) 0
(C) 9 (D) None of these
(C) 1 (D) Undefined
Q.90 If f(x) = ax + b and g(x) = cx + d, then y y
f(g(x)) = g(f(x)) is equivalent to- Q.99 If f(y) = , g(y) = , then
1  y2 1  y2
(A) f(a) = g(c) (B) f(b) = g(b) (fog)(y) equals-
(C) f(d) = g(b) (D) f(c) = g(a)
y y
1 x (A) 2 (B)
Q.91 If f : [0,1]  [0,1], f(x) = . g : [0,1]  [0,1], 1 y 1  y2
1 x
g(x) = 4x (1–x), then (fog) (x) equals- 1  y2
(C) y (D)
1  y2
1  4 x  4 x2 8 x (1  x)
(A) (B) Q.100 If f(x) = [x] and g(x) = cos (x), then the
1  4 x  4 x2 (1  x)2 range of gof is -
1  4 x  4 x2 (A) {0} (B) {–1,1}
(C) (D) None of these (C) {–1,0,1} (D) [–1,1]
1  4 x  4 x2

Questions
Q.92 If f, g, h are three functions in any set, then based on Inverse fucntion
wrong statement is -
(A) (fog)–1 = g–1 of –1 Q.101 If f : R  R, f(x) = x2 + 3, then pre- image of 2
(B) gof  fog under f is -
(C) (fog)oh = fo(goh) (A) {1,–1} (B) {1}
(D) (gof)–1 = g–1of –1
(C) {–1} (D) 

1 x Q.102 Which of the following functions has its inverse-


Q.93 If f(x) = , then f [f (sin)] equals -
1 x (A) f : R  R , f(x) = ax
(A) sin  (B) tan (/2) (B) f : R  R, f(x) = |x| + |x – 1|
(C) cot (/2) (D) cosec  (C) f : R0  R+, f(x) = |x|
(D) f : [, 2]  [–1,1], f(x) = cos x
Q.94 If f(x) = (a – x n)1/n, n  N, then f [f(x)] is equal to-
(A) 0 (B) x Q.103 If function f : R  R+, f(x) = 2x , then f –1 (x) will
(C) xn (D) (an – x)n be equal to-
(A) logx 2 (B) log2 (1/x)
 3x  x 3  (C) log2 x (D) None of these
 1 x   
Q.95 If f (x) = log   and g(x) =  1  3x 2  ,
 1 x    ex  e x
Q.104 The inverse of the function f(x) = +2
then f[g(x)] is equal to- e x  e x
(A) –f(x) (B) 3f(x) is given by -
(C) [f(x)]3 (D) None of these Fx  2I
(A) log G J
1/ 2
Fx  1I
(B) log G J
1/ 2

Q.96 If  (x) = x2 + 1 and  (x) = 3x, then  {  (x)}


Hx  1K Hx  1K
and  {  (x)} = F x I1/2
(C) log G J
Fx  1I1/2
(D) log G J
2
1
2
1
H2  xK H3  x K
(A) 32x+1, 3x (B) 32x+1, 3x 1
x 2 1 Q.105 If f : [1, )  [2, ) is given by ƒ(x) = x +
(C) 3 +1, 3
2x
(D) None of these x
then f–1(x) equals -
R
S1, when x  Q
4) x  x2  4 x
Q.97 If function f(x) =
T0, when x Q , (fof) ( (A)
2
(B)
1 x2
the value will be -
(A) 0 (B) 2 x  x2  4
(C) (D) 1 + x 2  4
(C) 1 (D) None of these 2
Corporate Office: CP Tower, Road No.1, IPIA, Kota (Raj.), Ph: 0744-2434159 FUNCTION 7
Q.106 If f(x) = loge(x + 1  x2 ), then f –1
(x) equals-

(A) log (x – 1  x2 )

ex  e x
(B)
2

ex  e x
(C)
2

ex  e x
(D)
ex  e x

Q.107 If f(x) = x3 – 1 and domain of f = {0,1,2,3},


then domain of f–1 is -
(A) {0,1,2,3}
(B) {1,0,–7,–26}
(C) {–1,0,7,26}
(D) {0,–1,–2,–3}

Q.108 If f(x) = {4 – (x – 7)3}1/5, then its inverse is-


(A) 7 – (4 – x5)1/3 (B) 7 – (4 + x5)1/3
(C) 7 + (4 – x5)1/3 (D) None of these

Q.109 If f : R  R, f(x) = ex and g : R  R,


g(x) = 3x – 2 , then the value of (fog)–1(x) is
equal to -
2  log x
(A) log (x – 2) (B)
3
F
Gx  3I
(C) log
H2 JK (D) None of these

Corporate Office: CP Tower, Road No.1, IPIA, Kota (Raj.), Ph: 0744-2434159 FUNCTION 8
LEVEL # 2
1
Q.1 If f(x) = x + , then - Q.11 Function f : R  R + , f(x) = x 2 + 2 and
x
(A) f(x2) = [f(x)]2 (B) f(x + y) = f(x) + f(y) F
G 1 IJ
(C) f(–x) = f(x) (D) f(1/x) = f(x)
g : R+
H K
 R, g(x) = 1  1  x then the value of
gof (2) is -
Q.2 If x is the radius of a circle and f(x) = x2, then
(A) 5/6 (B) 8/7
domain of f is -
(C) 1/6 (D) 6/5
(B) R (B) R+
(C) R ¯
(D) R0
Q.12 The period of function f (x) = |sin3 (x/2)| is
(A) 4  (B) 16 
1
Q.3 If f(x) = x2 – 3x + 1 and g(x) = , then (C) 2  (D) None of these
x2
domain of (f – g) is - Q.13 The inverse of the function y = logex is -
(A) R (B) R+ (A) 10x (B) 10–x
(C) R – {2} (D) None of these (C) ex
(D) e–x
Q.4 If f : R  R, f(x) = tan x, then pre-image of
–1 under f is - 1 x
Q.14 If f(x) = log , when – 1 < x1, x2< 1, then
R
S
(A) n 

n I
U
V R
S
(B) n 

n I
U
V
1 x
f(x1) + f(x2) equals-
T 4 W T 4 W F
x1  x2 IJ F
x1  x2 IJ
(C) {n  | n I } (D) None of these G
H K
(A) f 1  x x
1 2
G
H K
(B) f 1  x x
1 2

x2  2 x  1 Fx1  x2 IJ Fx1  x2 IJ
(C) f G (D) f G
Q.5 f(x) =
x2  3 x  2
is not defined for-
H1 x1x2 K H1 x1x2 K
(A) x = 2 (B) x = 1, 2
(C) x = 2,–1 (D) x = 0 Q.15 Function f : [–1,1]  R, f(x) = sin (  /2) x is -
(A) one-one (B) onto
Q.6 If f : R  R, f(x) = x3 + 3, and g : R  R,
(C) one-one onto (D) None of these
g(x) = 2x + 1, then f–1og–1(23) equals-
(A) 2 (B) 3 Q.16 If the domain of function f(x) = x2 – 6x + 7 is
(C) (14)1/3 (D) (15)1/3 (–  ,  ), then the range of function is -
Q.7 If f(x) = log x, g(x) = x3, then f[g(a)] + f [g(b)] is (A) (–  ,  ) (B) [–2,  )
equal to- (C) (–2,3) (D) (–  ,–2)
(A) f [g(a) + g(b)] (B) 3 f(ab)
(C) g [f(ab)] (D) g [f(a) + f(b)] Q.17 Function f : R  R, f(x) = [x] is -
(A) one-one onto (B) one-one into
Q.8 Function sin–1 x is defined in the interval- (C) many-one onto (D) many-one into
(A) (–1,1) (B) [0,1]
(C) [–1,0] (D) (–1,2) Q.18 If S be the set of all triangles and f : S  R +,
f (  ) = Area of  , then f is -
 (A) One-one onto (B) one-one into
Q.9 The interval for which sin–1 x + cos–1 x= (C) many-one onto (D) many-one into
2
holds-
(A) [0,  ) (B) [0,3] Q.19 If f : C  R , f(z) = |z|, then f is -
(C) [0,1] (D) [0,2] (A) one-one but not onto
(B) onto but not one-one
Q.10 The domain of the function f(x) = x! is - (C) neither one-one nor onto
(A) (0,  ) (B) N (D) both one-one and onto
(C) W (D) R+

Corporate Office: CP Tower, Road No.1, IPIA, Kota (Raj.), Ph: 0744-2434159 FUNCTION 9
L 1 , 1 O
If f : M
Q.29 If f(x) = x2 – x–2, then f(1/x) equals-
Q.20
N2 2 P Q [–1,1], f(x) = sin x, then f (A)
1
(B) –1/f(x)
is - f ( x)
(A) one-one (B) one-one onto (C) f(x) (D) – f(x)
(C) onto (D) None of these

Q.21 If f(x) = 1/x then f(a) – f(b) equals- Q.30 The domain of function
F
Gb  aI F ab I 1
(A) f
Hab JK (B) f G
Ha  b J
K f(x) = log (3  x) + x  2 is -
10
(A) [–2, 3) (B) [–2, 3) – {2}
Fab I
(C) f G J
Fa  b I
(D) f G J
(C) [–3, 2] (D) [–2, 3] – {2}
Hb  a K Ha  b K x3
Q.31 Domain of the function f(x) = is-
x , correct statement is - ( x  1) x2  4
Q.22 f(x) = cos
(A) (1,2)
(A) f(x) is periodic and its period = 2 (B) (–  , –2)  (2,  )
(B) f(x) is periodic and its period = 4  2 (C) (–  ,–2)  (1,  )
(D) (–  ,  ) – {1,  2}
(C) f(x) is periodic and its period = 
(D) f(x) is not periodic Q.32 Range of the function f(x) = sin2(x4) + cos2(x4)
is-
Q.23 If f be the greatest integer function and g be the (A) (–  ,  ) (B) {1}
modulus function, then (C) (–1,1) (D) (0,1)
F
G 5I
J F 5 IJ=
– (fog) G
(gof) 
H3 K H3 K Q.33 Let f : R  R be a function defined by
f(x) = x + x 2 , then f is-
(A) 1 (B) –1
(C) 2 (D) 4 (A) injective (B) surjective
(C) bijective (D) None of these
Q.24 The domain of function f(x) = log |log x| is-
(A) (0,  ) (B) (1,  )
(C) (0,1)  (1,  ) (D) (–  ,1) Q.34 If f (x) = e3x and g(x) =  n x, x > 0, then (fog) (x)
is equal to-
Q.25 Domain of the function tan–1 x + cos–1 x2 is - (A) 3x (B) x3
(A) R– [–1,1] (B) R– (–1,1) (C) log 3x (D) 3 log x
(C) (–1,1) (D) [–1,1]
Q.35 If f : R  R f(x) = cos (5x + 2) then the value
Q.26 Which of the following functions are equal ? of f –1(x) is -
(A) f(x) = x, g(x) = x
2 cos 1( x)  2
(A) (B) cos 1 ( x )  2
(B) f(x) = log x2 , g(x) = 2 log x 5
(C) f(x) = 1, g(x) = sin2x + cos2 x
cos 1( x)
(D) f(x) = x/x, g(x) = 1 (C) 2 (D) Does not exist
5
Q.27 If f : Q  Q, f(x)= 2x and g : Q  Q, R
|S4  x2 U
|
g(x) = x + 2, then (fog)–1(20) equals- Function f(x) = sin log (1  x) Vhas domain
(A) 10 (B) 12
Q.36
|T |W
(C) 8 (D) 6 (A) [–2,1) (B) [–2,1]
(C) (–2,1) (D) (–  ,1)
2 cosh x  sin2 x
Q.28 f(x) = is - Q.37 The domain of function
x2  1
f(x) = log (3x –1) + 2 log (x +1) is -
(A) an algebric function
(A) [1/3,  ) (B) [–1,1/3]
(B) a trigonometrical function
(C) (–1,1/3) (D) None of these
(C) an even function
(D) an implicit function
Corporate Office: CP Tower, Road No.1, IPIA, Kota (Raj.), Ph: 0744-2434159 FUNCTION 10
x
F
G 1I
H xJ
1
Q.38 If f(x) =
1  x2
, then (fofof) (x) is equal to- Q.44 Let f x 
K= x + x2 (x  0), then f(x) equals-
2

3x x (A) x2 – 2 (B) x2 –1
(A) (B) (C) x2 (D) None of these
1  x2 1  3 x2
Q.45 The graph of f(x) = – |x| is -
3x
(C) (D) None of these
1  x2

Q.39 Which one of the following graphs represents


the function y = 1+ |x| for all x  R ?
(A) (B)
(A)

(B)

(C) (D)

(C) (D) Q.46 If a2 + b2 + c2 = 1, then range of ab + bc + ca is-


(A) [–1/2,  ) (B) (0,  )
(C) [–1/2,1] (D) [1,  )

Q.40 If f (x) = x3 – x and g(x) = sin 2x, then - Q.47 If x = logabc, y = log b ca, and z = logcab, then
(A) g [f(1)] = 1 1 1 1
(B) f (g (/12)) = – 3/8 + + equals-
1 x 1 y 1 z
(C) g {f(2)} = sin 2
(A) 1 (B) x + y + z
(D) None of these
(C) abc (D) ab + bc + ca
1 1 Q.48 The range of 5 cos x – 12 sin x + 7 is -
Q.41 If f(x) = and g (x) = , then (A) [–6,20] (B) [–3,18]
x1 x 1
common domain of function is - (C) [–6,15] (D) None of these
(A) {x | x <1, x  R } Q.49 The domain of the function log 2 log 3 log 4(x)
(B) {x | x  0, x  1, x  R} is -
(C) {1} (A) (1,  ) (B) (2,  )
(D) {–1} (C) (3,  ) (D) (4,  )

Q.42 The natural domain of the real valued function

defined by f (x) = x2  1 + x2  1 is-


(A) 1 < x <  (B) –  < x < 
(C) –  < x <–1 (D) (–  ,  ) – (–1,1)

9  x2
Q.43 If f(x) = , then domain of f is -
sin 1(3  x)
(A) [2,3] (B) [2,3)
(C) (2,3] (D) None of these

Corporate Office: CP Tower, Road No.1, IPIA, Kota (Raj.), Ph: 0744-2434159 FUNCTION 11
LEVEL # 3
Q.1 The domain of definition of Q.7 The value of nI for which the function

 x 1  1 sin nx
log0.4   2 f(x) = has 4 as its period is -
f(x) = is– x
 x  5  x  36 sin  
n
(A) (x : x < 0, x  – 6}
(A) 2 (B) 3 (C) 4 (D) 5
(B) (x : x > 0, x  1, x  6}
(C) (x : x > 1, x  6} Q.8 If f(x) is an odd periodic function with period
(D) (x : x  1, x  6} 2, then f (4) equals to -
(A) 0 (B) 2
(C) 4 (D) –4
Q.2 The function f : R  R defined by
f (x) = (x – 1) (x – 2) (x – 3) is -
Q.9 Domain of the function
(A) one-one but not onto
1
 x 2 
(B) onto but not one-one f(x) = sin  log 5 is -
(C) both one and onto  5 

(D) neither one-one nor onto (A) [–5, –1]  [1, 5] (B) [–5, 5]
(C) (–5, –1)  (1, 5) (D) None of these

Q.3 Set A has 3 elements and set B has 4


elements. The number of injections that can 1 | x |
be defined from A to B is - Q.10 Domain of f(x) = is -
2 | x |
(A) 144 (B) 12 (A) R – [–2, 2]
(C) 24 (D) 64 (B) R – [–1, 1]
(C) [–1, 1]  (–, –2)  (2, )
Q.4 The number of bijective functions from set A (D) None of these
to itself when a contains 106 elements -
(A) 106 (B) (106)2
  
 4  x2   is -
(C) 106! (D) 1106 Q.11 Range of sin  log 
  1 x  
  
Q.5 Let A be a set containing 10 distinct (A) (–1, 1) (B) [–1, 1]
elements, then the total number of distinct (C) R (D) None of these
functions from A to A is -
(A) 10 ! (B) 1010
(C) 210 (D) 210 – 1 2
Q.12 If f(x) = 3 sin  x 2 , then values of f(x) lie
16
in -
Q.6 Let f : R  R be a function defined by
  
(A)  ,  (B) [–2, 2]
e | x|  e  x  4 4
f (x)  . Then -
e x  e x  3 
(C) 0 ,  (D) None of these
(A) f is a bijection  2
(B) f is an injection only
(C) f is a surjection only
Q.13 The function f (x) = cos (log (x + x 2  1 )) is-
(D) f is neither an injection nor a surjection
(A) even (B) odd
(C) constant (D) None of these
Corporate Office: CP Tower, Road No.1, IPIA, Kota (Raj.), Ph: 0744-2434159 FUNCTION 12
Q.14 The function f(x) = max. [1 – x, 1 + x, 2] Q.20 If [x] denote the greatest integer  x, the
x  R is equivalent to - domain of definition of function

 1  x , x  1 4  x2
 f (x) = is -
(A) f ( x )  2 ,  1  x  1 [ x]  2
 1 x, x  1 (A) (–, –2)  [–1, 2] (B) [0, 2]

(C) [–1, 2] (D) (0, 2)
 1  x , x  1
 Q.21 The function f : [–1/2, 1/2]  [–/2, /2]
(B) f ( x )  2 ,  1  x  1
 1 x , x  1 defined by f(x) = sin–1(3x – 4x3) is–
 (A) both one-one and onto
(B) neither one-one nor onto
1  x , x  1
 (C) onto but not one-one
(C) f ( x )  1,  1  x  1
(D) one-one but not onto
 1 x, x  1

(D) None of these Q.22 The function f satisfies the functiona equation
 x  59 
3f (x) + 2f    10 x  30 for all real x  1.
Q.15 The domain of the function f(x) = 9–xPx–5 is-  x 1 
(A) [5, 7] (B) {5, 6, 7} The value of f (7) is -
(C) {3, 4, 5, 6, 7} (D) None of these (A) 8 (B) 4
(C) –8 (D) 11
Q.16 The range of the function f(x) = 9–xPx–5 is -
(A) {1, 2, 3} (B) [1, 2] Q.23 The domain of the function
(C) {1, 2, 3, 4, 5} (D) None of these f (x) = log 3+x(x2 – 1) is -
(A) (–3, –1)  (1, )
Q.17 Domain of the function (B) [–3, –1)  [1, )
 (C) (–3, –2)  (–2, –1)  (1, )
 1  
f ( x )  log 2   log1 / 2 1    1 is - (D) [–3, –2)  (–2, –1)  [1, )
 4
x 
   
(A) (0, 1) (B) (0, 1] Assertion & Reason Type Question :-
(C) [1, ) (D) (1, )
All questions are Assertion & Reason type
Q.18 The period of f(x) = [sin 5x] + |cos 6x| is - questions. Each of these questions contains
two statements : Statement-I (Assertion) and

(A) (B)  Statement-2 (Reason). Answer these ques
2
tions from the following four option.
2 (A) Statement-1 is false. Statement-2 is true
(C) 2 (D)
5 (B) Statement-1 is true. Statement-2 is true;
Statement-2 is a correct explanation for
x x Statement-1
Q.19 Period of f (x) = sin x + tan + sin 2 +
2 2 (C) Statement-1 is true. Statement-2 is true;
x x x Statement-2 is not a correct explanation
tan + ... + sin n  1 + tan n is -
23 2 2 for Statement-1
(A)  (B) 2 (D) Statement-1 is true. Statement-2 is false

(C) 2n (D)
2n

Corporate Office: CP Tower, Road No.1, IPIA, Kota (Raj.), Ph: 0744-2434159 FUNCTION 13
Q.24 Statement-1 : The period of Passage :-
1 Let here we define f : R  [–1, 1] and
f(x) = sin 2x cos [2x] – cos 2x sin [2x] is g : R  [–1, 1]. Now f(x) = 2 cos2 x – 1,
2
Statement-2 : The period of x – [x] is 1 g(x) = cos 2x, h(x) = f(x) + g(x),
f(x)
I(x) = f(x) – g(x), j (x) = g(x) are 5 functions.
Q.25 Statement-1 :
If f(x) = |x – 1| + |x – 2| + |x – 3| On the basis of above information, answer
Where 2 < x < 3 is an identity function. the following questions-
Statement-2 : f : A  A defined by
f(x) = x is an identity function. Q.31 Which statement is correct-
(A) Period of f(x), g(x) and h(x) are same
Q.26 Statement-1 : f : R  R defined by 2
and value is
f(x) = sin x is a bijection 3
(B) Period of f(x), g(x) and h(x) makes
Statement-2 : If f is both one and onto it is

bijection the A.P. with common difference
4
Q. 27 Statement-1 : f : R  R is a function defined (C) Sum of periods of f(x), g(x) and
2x  1 h(x) is 3
by f(x) = . (D) None of these
3
–1 3x  1 Q.32 Which statement is correct regarding function
Then f (x) =
2 j(x) and I(x)-
Statement-2 : f(x) is not a bijection. (A) The domain of j(x) and I(x) are the
same
(B) Range of j(x) and I(x) are the same
Q.28 Statement-1 : If f is even function, g is odd (C) The union of domain of j(x) and I(x) are all
f real numbers
function then g , (g  0) is an odd function. (D) None of these

Statement-2 : If f(–x) = –f(x) for every x of its Q.33 If the solution of equation I(x) – g(x) = 0 are
domain, then f(x) is called an odd function and x1, x2, x3, .... xn when x  [0, 10] then which
if f(–x) = f(x) for every x of its domain, then f(x) option is correct-
(A) x 1 , x 2 , x 3 ... x n makes the A.P. with
is called an even function. common difference 
(B) Total no. of solutions of I (x) – g(x) = 0 is 20
Q.29 Statement 1 : Function f(x) = sinx + {x} is for x  [0, 10]
periodic with period 2 (C) Sum of all solutions of the given
equation is 100 in the interval [0, 10]
Statement 2 : sinx and {x} are both periodic
(D) (B) and (C) are correct
with period 2 and 1 respectively.
Q.34 If h : R  [–2, 2], then -
(A) h(x) is one-one function
2 (B) h(x) is one-one and onto function
Q.30 Statement 1 : y = f(x) = x  2x  4 , (C) h(x) is onto function
x 2  2x  5
x  R Range of f(x) is [3/4, 1) (D) h(x) is many one and into function
Q.35 Domain and range of j(x) respectively -
Statement 2 : (x – 1)2 = 4 y  3 . (A) R and {1}
1 y (B) R and {0, 1}
(C) R – {(2n + 1) /4}, n I and {1}
(D) R – {(2n + 1) /2}, n  I and {1}

Corporate Office: CP Tower, Road No.1, IPIA, Kota (Raj.), Ph: 0744-2434159 FUNCTION 14
LEVEL # 4
(Questions asked in Previous AIEEE & IIT-JEE)

SECTION - A Q.8 A function f from the set of natural numbers to


Q.1 Which of the following is not a periodic function - integers defined by
[AIEEE 2002]
 n 1
(A) sin 2x + cos x (B) cos x  , when n is odd
(C) tan 4x (D) log cos 2x f(n) =  2 n is
 , when n is even
 2
[AIEEE 2003]
Q.2 The period of sin2 x is- [AIEEE 2002]
(A) neither one-one nor onto
(A) /2 (B) 
(C) 3/2 (D) 2 (B) one-one but not onto
(C) onto but not one-one
Q.3 The function f : R  R defined by f(x) = sin x is- (D) one-one and onto both
[AIEEE-2002]
Q.9 The range of the function f(x) = 7– xPx–3 is-
(A) into (B) onto
[AIEEE 2004]
(C) one-one (D) many-one
(A) {1, 2, 3} (B) {1, 2, 3, 4, 5, 6}
(C) {1, 2,3,4} (D) {1, 2, 3, 4, 5}
2x
Q.4 The range of the function f(x) = , x  2 is -
2x
Q.10 If f : R  S, defined by f(x) = sin x – 3 cos x + 1,
[AIEEE-2002]
(A) R (B) R – {–1} is onto, then the interval of S is-
(C) R – {1} (D) R – {2} [AIEEE 2004]
(A) [0, 3] (B) [–1, 1]
(C) [0, 1] (D) [–1, 3]
Q.5 The function f(x) = log (x + x 2  1 ), is-
[AIEEE 2003] Q.11 The graph of the function y = f(x) is symmetrical
(A) neither an even nor an odd function about the line x = 2, then- [AIEEE 2004]
(B) an even function (A) f(x+ 2) = f(x – 2) (B) f(2 + x) = f(2 – x)
(C) an odd function (C) f(x) = f(–x) (D) f(x) = – f(–x)
(D) a periodic function

Q.6 Domain of definition of the function sin 1( x  3 )


Q.12 The domain of the function f(x) = is-
3 9  x2
f(x) = + log10 (x3 – x), is- [AIEEE 2003]
4  x2
[AIEEE 2004]
(A) (– 1, 0)  (1, 2)  (2, ) (A) [2,3] (B) [2,3)
(B) (1, 2)
(C) [1,2] (D) [1, 2)
(C) ( – 1, 0) (1, 2)
(D) (1, 2)  (2, )
Q.13 Let f : (–1, 1)  B, be a function defined by

Q.7 If f : R  R satisfies f(x+ y) = f(x) + f(y), for all 2x


f(x) = tan–1 , then f is both one-one and
n 1 x2
x, y  R and f(1) = 7, then  f (r ) is- onto when B is the interval - [AIEEE-2005]
r 1
[AIEEE 2003]    
(A)  0,  (B) 0, 
7n (n  1) 7n  2  2
(A) (B)
2 2      
7(n  1) (C)  ,  (D)   , 
 2 2  2 2
(C) (D) 7n (n+1)
2
Corporate Office: CP Tower, Road No.1, IPIA, Kota (Raj.), Ph: 0744-2434159 FUNCTION 15
Q.14 A real valued function f(x) satisfies the functional SECTION - B
equation f(x – y) = f(x) f(y) – f (a – x) f(a + y)
where a is a given constant and f(0) = 1, then 1  x 
Q.1 If function f(x) = – tan  2  ; (–1 < x < 1)
f(2a – x) is equal to - [AIEEE-2005] 2  
(A) –f(x) (B) f(x)
and g(x) = 3  4 x  4 x 2 , then the domain of
(C) f(a) + f(a – x) (D) f(–x)
gof is – [IIT 90]
 1 1
    (A) (–1, 1) (B)   2 , 2 
Q.15 The largest interval lying in  ,  for which  
2 2
the function is defined, is- [AIEEE 2007]  1  1 
(C)   1, 2  (D)   2 , 1
   
   
(A) [0, ] (B)  , 
2 2
Q.2 If f(x) = cos [2]x + cos [–]x, where [x]
    
(C)  ,  (D) 0,  stands for the greatest integer function, then
 4 2  2 [IIT 91]
Q.16 Let f : N  Y be a function defined as 
(A) f  2  = –1 (B) f () = 1
 
f(x) = 4x + 3 where Y = |y  N : y = 4x + 3 for

some x  N|. Show that f is invertible and its (C) f  4  = 2 (D) None of these
 
inverse is [AIEEE 2008] Q.3 The value of b and c for which the identity
f(x + 1) – f(x) = 8x + 3 is satisfied,
y3 y3
(A) g(y) = 4 + (B) g(y) = where f(x) = bx2 + cx + d, are [IIT 92]
4 4
y 3 3y  4 (A) b = 2, c = 1 (B) b = 4, c = –1
(C) g(y) = (D) g(y) = (C) b = –1, c = 4 (D) None
4 3

Q.17 For real x, let f(x) = x3 + 5x + 1, then - Q.4 Let f(x) = sin x and g(x) = ln |x|. If the ranges
of the compositie functions fog and gof are
[AIEEE 2009]
R1 and R2 respectively, then – [IIT 94]
(A) f is one – one but not onto R (A) R1 = {u : –1 < u < 1},
(B) f is onto R but not one – one R2 = {v : –  < v < 0}
(C) f is one – one and onto R (B) R1 = {u : – < u < 0},
(D) f is neither one – one nor onto R R2 = {v : – 1 < v < 1}
(C) R1 = {u : –1 < u < 1},
Q.18 Let f(x) = (x + 1)2 –1, x > –1 R2 = {v : –  < v < 0}
Statement – 1 : [AIEEE 2009] (D) R1 = {u : –1 < u < 1},
The set {x : f(x) = f–1(x)} = {0, –1}. R2 = {v : –  < v < 0}
Statement – 2 :
f is a bijection. Q.5 Let 2 sin2 x + 3 sin x – 2 > 0 and x2 – x –
(A) Statement -1 is true, Statement -2 is true; 2 < 0 (x is measured in radians). Then x lies
Statement -2 is a correct explanation for in the interval [IIT 94]
Statement -1
  5   5 
(B) Statement -1 is true, Statement -2 is true; (A)  6 , 6  (B)   1, 6 
Statement -2 is not a correct explanation    
for Statement -1.
 
(C) Statement -1 is true, Statement -2 is false. (C) (–1, 2) (D)  6 ,2 
 
(D) Statement -1 is false, Statement -2 is true.
Corporate Office: CP Tower, Road No.1, IPIA, Kota (Raj.), Ph: 0744-2434159 FUNCTION 16
Q.6 Let f(x) = (x + 1)2 – 1, (x > – 1). Then the
Q.11 If g(f(x)) = |sin x| and f(g(x)) = (sin x )2,
set S = {x : f(x) = f –1(x)} is – [IIT 95]
then [IIT 98]
(A) Empty
(A) f(x) = sin2 x, g(x) = x
(B) {0, –1}
(B) f(x) = sin x, g(x) = |x|
(C) {0, 1, –1}
 (C) f(x) = x2, g(x) = sin x
 3  i 3  3  i 3 
(D) 0,1, 2
,
2
 (D) f and g cannot be determined
 

Q.12 If f(x) = 3x – 5, then f–1 (x) [IIT 98]


Q.7 If f(1) = 1 and f(n + 1) = 2f(n) + 1 if n  1, 1
then f(n) is- [IIT 95] (A) is given by
3x  5
(A) 2n+1 (B) 2n x5
(B) is given by
(C) 2n – 1 (D) 2n–1 – 1 3
(C) does not exist because f is not one - one
(D) does not exist because f is not onto
Q.8 If f is an even function defined on the interval
(– 5, 5), then the real values of x satisfying Q.13 If the function f : [1, )  [1, ) is defined
 x 1 by f(x) = 2x(x–1) , then f–1 (x) is [IIT 99]
the equation f(x) = f   are-
 x  2  1
x ( x 1)

(A)  
 2
[IIT 96]

(A)
 1 5  3  5
,
(B)
1
2

1  1  4 log2 x 
2 2

(B)
 1 3  3  3
2
,
2
(C)
1
2

1  1  4 log2 x 
2 5 (D) not defined
(C)
2
(D) None of these Q.14 The domain of definition of the function y(x)
given by the equation 2x + 2y = 2 is –
[IIT Scr. 2000]
  
Q.9 Let f(x) = [x] sin   , where [.] denotes the (A) 0 < x < 1 (B) 0 < x < 1
 [ x  1] 
greatest integer function. The domain of f is ....... (C) – < x < 0 (D) – < x < 1
(A) {x  R| x  [–1, 0)}
(B) {x  R| x  [1, 0)}
Q.15 Let f() = sin (sin + sin 3), then f()
(C) {x  R| x  [–1, 0)} [IIT 2000]
(D) None of these [IIT 96]
(A)  0 only when   0
(B)  0 for all 
  (C)  0 for all real 
Q.10 If f(x) = sin2x + sin2  x  3  + cos x cos
  (D)  0 only when   0
  5
 x   and g   = 1, then (gof) (x) =
 3 4 Q.16 The number of solutions of log4 (x – 1) =
[IIT 96] log2 (x – 3) is – [IIT Scr. 2001]
(A) –2 (B) –1 (A) 3 (B) 1
(C) 2 (D) 1 (C) 2 (D) 0
Corporate Office: CP Tower, Road No.1, IPIA, Kota (Raj.), Ph: 0744-2434159 FUNCTION 17
x Q.22 Let function f : R  R be defined by
Q.17 Let f(x) = , x  – 1, then for what value
x 1 f(x) = 2x + sin x for x  R. Then f is–
of  f{f(x)} = x. [IIT Scr. 2001]
[IIT Scr. 2002]
(A) 2 (B) – 2 (A) one to one and onto
(C) 1 (D) –1 (B) one to one but NOT onto
(C) onto but NOT one to one
(D) neither one to one nor onto
log2 ( x  3)
Q.18 The domain of definition of f (x) =
x2  3 x  2 x
is – [IIT Scr. 2001] Q.23 Let f(x) = defined as [0, )  [ 0, ),
1 x
(A) R / { –2, –2} f(x) is– [IIT Scr.2003]
(B) (– 2, )
(A) one one & onto
(C) R/ {–1, –2, –3} (B) one- one but not onto
(D) (–3, ) / {–1, –2}
(C) not one-one but onto
(D) neither one-one nor onto
1
Q.19 If f : [1, )  [2, ) is given by f(x) = x +
x x2  x  2
then f–1 (x) equals – [IIT Scr. 2001] Q.24 Find the range of f(x) = is–
x2  x  1
[IIT Scr.2003]
x  x2  4 x
(A) (B)  11 
2 1  x2 (A) (1, ) (B) 1, 
 7
x  x2  4  7  7
(C) (D) 1 + x2  4
2 (C) 1,  (D) 1, 
 3  5

Q.20 Let g(x) = 1 + x – [x] and Q.25 Domain of f(x) = sin 1(2x )   / 6 is–
[IIT Scr.2003]
 1 ; x  0
  1 1  1 1
f(x) =  0 ; x  0 . Then for all x, f(g(x)) is (A)  , (B)  ,
1 ; x0  4 2   2 2 

 1 1  1 1
(C)  , (D)  ,
equal to :
 4 4   2 4 
(where [.] denotes the greatest integer
function): [IIT Scr. 2001] Q.26 Let f(x) = sinx + cos x & g(x) = x2 – 1, then
(A) x (B) 1 g(f(x)) will be invertible for the domain-
(C) f(x) (D) g(x) [IIT Scr.2004]
  
(A) x  0,  (B) x   , 
Q.21 Suppose f(x) = (x + 1)2 for x  – 1. If g(x) is  4 4
the function whose graph is the reflection of     
(C) x  0,  (D) x   , 0
the graph of f(x) with respect to the line  2  2
y = x, then g(x) equals– [IIT Scr. 2002]
x x  Q 0 x  Q
(A) – x – 1, x  0 Q.27 f (x)   ; g (x)  
0 x  Q x x  Q
1
(B) , x > – 1 then (f – g) is [IIT Scr.2005]
( x  1)2
(A) one-one , onto
(C) x 1, x  – 1 (B) neither one-one, nor onto
(C) one-one but not onto
(D) x – 1, x  0
(D) onto but not one-one

Corporate Office: CP Tower, Road No.1, IPIA, Kota (Raj.), Ph: 0744-2434159 FUNCTION 18
ANSWER KEY
LEVEL # 1
Q.No. 1 2 3 4 5 6 7 8 9 10 11 12 13 14 15 16 17 18 19 20
Ans. C A C B B A C D C C D A B A A D A B C B

Q.No. 21 22 23 24 25 26 27 28 29 30 31 32 33 34 35 36 37 38 39 40
Ans. B A B A C B D B B B B B B B B B B B B A
Q.No. 41 42 43 44 45 46 47 48 49 50 51 52 53 54 55 56 57 58 59 60
Ans. C C C B C B B B B C C B B C C A D C B B
Q.No. 61 62 63 64 65 66 67 68 69 70 71 72 73 74 75 76 77 78 79 80
Ans. D A B C B A A C A D D C C D C C C B B B
Q.No. 81 82 83 84 85 86 87 88 89 90 91 92 93 94 95 96 97 98 99 100
Ans. C D C B B D D C C C A D A B B C C C C B
Q.No. 101 102 103 104 105 106 107 108 109
Ans. D D C D A C C C B

LEVEL # 2
Q.No. 1 2 3 4 5 6 7 8 9 10 11 12 13 14 15 16 17 18 19 20
Ans. D B C A B A B B C C D C C A A B D C C B
Q.No. 21 22 23 24 25 26 27 28 29 30 31 32 33 34 35 36 37 38 39 40
Ans. C D A C D C C C D B B B A B D C D B C B
Q.No. 41 42 43 44 45 46 47 48 49
Ans. B D B A C C A A D

LEVEL # 3
Q.No. 1 2 3 4 5 6 7 8 9 10 11 12 13 14 15 16 17 18 19 20
Ans. C B C C B D A A A C B C A A B A A C C A

Q.No. 21 22 23 24 25 26 27 28 29 30 31 32 33 34 35
Ans. A B C B B A D B A B C C D C C

LEVEL # 4
SECTION - A
Q.No. 1 2 3 4 5 6 7 8 9 10 11 12 13 14 15 16 17 18
Ans. B B A,D B C A A D A D B B D A D C C B

SECTION - B
Q.No. 1 2 3 4 5 6 7 8 9 10 11 12 13 14 15
Ans. B A B D D B C A C D A B B D C
Q.No. 16 17 18 19 20 21 22 23 24 25 26 27
Ans. B D D A B D A B C A B A

Corporate Office: CP Tower, Road No.1, IPIA, Kota (Raj.), Ph: 0744-2434159 FUNCTION 19
FUNCTION

Preface

As you have gone through the theory part that consists of given fundamental principles,
definitions, concepts involved and solved problems. After going through theory part it
becomes necessary to solve the unsolved problems based on the concepts given. To
solve this purpose we are providing exercise part that comprises of various exercises
based on the theory. By solving various kinds of problems you can check your grasp on
the topic and can determine whether you have been able to find optimum depth in relevant
topic or not.

Students are advised to solve the questions of exercises (Levels # 1, 2, 3, 4) in the same
sequence or as directed by the faculty members, religiously and very carefully.

Level # 3 is not for foundation course students, it will be discussed in fresher & target
courses.

The list of exercises is as following :

Total No.of questions in Function are -

Level # 1 ........................................ 109


Level # 2 ........................................ 49
Level # 3 ........................................ 35
Level # 4 ........................................ 45

Total No. of questions......................................................... 238

Corporate Office: CP Tower, Road No.1, IPIA, Kota (Raj.), Ph: 0744-2434159 FUNCTION 1
LEVEL # 1
Questions
inequation Q.8 If x2 – 1  0 and x2 – x – 2  0, then x line in the
based on
interval/set
(A) (–1, 2) (B) (–1, 1)
2 (C) (1, 2) (D) {– 1}
Q.1 The inequality < 3 is true, when x belongs to-
x
2   2 Questions
Definition of function
(A)  ,   (B)     based on
3   3

2  Q.9 Which of the following relation is a function ?


(C)  ,    (–, 0) (D) none of these (A) {(1,4), (2,6), (1,5), (3,9)}
3  (B) {(3,3), (2,1), (1,2), (2,3)}
(C) {(1,2), (2,2,), (3,2), (4,2)}
(D) {(3,1), (3,2), (3,3), (3,4)}
x4
Q.2 < 2 is satisfied when x satisfies-
x 3 Q.10 If x, y  R, then which of the following rules is
(A) (–, 3) (10, ) (B) (3, 10) not a function-
(C) (–, 3) [10, ) (D) none of these (A) y = 9 –x2 (B) y = 2x2
(C) y = x – |x| (D) y = x2 + 1
x7
Q.3 Solution of > 2 is-
x3 Questions
(A) (–3, ) (B) (–, –13) based on Even and odd function
(C) (–13, –3) (D) none of these
Q.11 Which one of the following is not an odd
2x  3 function -
Q.4 Solution of  3 is- (A) sin x (B) tan x
3x  5
(C) tanh x (D) None of these
 12   5 12 
(A) 1,  (B)  , 
 7  3 7  sin4 x  cos 4 x
Q.12 The function f(x) = is -
 5 12  x  tan x
(C)   ,  (D)  ,   (A) odd
 3 7 
(B) Even
(C) neither even nor odd
Q.5 Solution of (x – 1)2 (x + 4) < 0 is-
(D) odd and periodic
(A) (–, 1) (B) (–, –4)
(C) (–1, 4) (D) (1, 4) Q.13 A function is called even function if its graph is
symmetrical w.r.t.-
Q.6 Solution of (2x + 1) (x – 3) (x + 7) < 0 is- (A) origin (B) x = 0
(C) y = 0 (D) line y = x
 1  1 
(A) (– , –7)    , 3  (B) (– , – 7)   ,3 
Q.14 A function is called odd function if its graph is
 2  2 
symmetrical w.r.t.-
 1  (A) Origin (B) x = 0
(C) (–, 7)    , 3  (D) (–, –7)  (3, ) (C) y = 0 (D) line y = x
 2 
Q.15 The even function is-
Q.7 If x2 + 6x – 27 > 0 and x2 – 3x – 4 < 0, then- (A) f(x) = x2 (x2 +1) (B) f(x) = sin3 x + 2
(A) x > 3 (B) x < 4 (C) f(x) = x (x +1) (D) f(x) = tan x + c
7
(C) 3 < x < 4 (D) x =
2

Corporate Office: CP Tower, Road No.1, IPIA, Kota (Raj.), Ph: 0744-2434159 FUNCTION 2
Q.16 A function whose graph is symmetrical about Q.25 In the following which function is not
the y-axis is given by- periodic-
(A) tan 4x (B) cos 2x
(A) f(x) = loge (x + x2  1 )
(C) cos x2 (D) cos2x
(B) f(x + y) = f(x) + f(y) for all x, y  R
(C) f(x) = cos x + sin x Domain, Co-domain and range
of function
(D) None of these
1
Q.17 Which of the following is an even function ? Q.26 Domain of the function f(x) = is-
x
x2
a 1
(A) x (B) tan x (A) R (B) (–2,  )
ax  1 (C) [2,  ] (D) [0,  ]
ax  a x ax  1
(C) (D) Q.27 The domain where function f(x) = 2x2 – 1 and
2 ax  1
g(x) = 1 – 3x are equal, is-
Q.18 In the following, odd function is - (A) {1/2} (B) {2}
(A) cos x2 (B) (ex + 1)/(ex – 1) (C) {1/2,2} (D) {1/2,-2}
(C) x – |x|
2
(D) None of these
3x
Q.28 The domain of the function log is-
Q.19 The function f(x) = x – |x| is -
2 2
(A) an odd function (A) (3,  ) (B) (–  ,3)
(C) (0,3) (D) (–3,3)
(B) a rational function
(C) an even function Q.29 Domain of the function cos–1 (4x –1) is-
(D) None of these (A) (0,1/2) (B) [0,1/2]
(C) [1/2,2] (D) None of these
Questions
based on Periodic function Q.30 Domain of the function log |x2 – 9| is-
(A) R (B) R– [–3,3]
(C) R – {–3,3} (D) None of these
Q.20 The period of sin4 x + cos4 x is -
(A)  (B) /2
Q.31 The domain of the function-
(C) 2 (D) None of these
Q.21 The period of function |cos 2x| is - f (x) = x  1 + 6  x is-
(A) (B) /2 (A) (1,6) (B) [1,6]
(C) 4 (D) 2 (C) [1,  ) (D) (–  ,6]

F
Gx I
J Fx I
+ cos G Jis- The domain of the function f(x) = (2  2x  x2 )
Q.22 The period of function sin
H2 K H2 K Q.32
is -
(A) 4 (B) 6
(C) 12 (D) 24 (A) – 3 x  3
Q.23 The period of the function (B) – 1– 3  x  –1 + 3
f(x) = log cos 2x + tan 4x is -
(C) – 2  x  2
(A) /2 (B) 
(C) 2 (D) 2/5 (D) –2 + 3  x  –2– 3
1
Q.24 The period of the function f(x) = 2 cos (x–) Q.33 Domain of a function f(x) = sin–1 5x is-
3
F
G IJ
1 1 L
M O
1 1
N P
is -
(A) 6 (B) 4 H K
(A)  ,
5 5
(B)  ,
5 5Q
(C) 2 (D)  F
G I
HJ
1
(C) R (D) 0,
K
5

Corporate Office: CP Tower, Road No.1, IPIA, Kota (Raj.), Ph: 0744-2434159 FUNCTION 3
Q.34 If f : R+  R, f(x) = log x, then range of f is -
(A) R0 (B) R Q.43 The range of f : R+  R+, f(x) = ex is -
(C) R+ (D) None of these (A) (0,  ) (B) [1,  )
(C) (1,  ) (D) None of these

Q.35 The range of the function f : R  R, f(x) = tan–1 x Q.44 The range of f(x) = cos 2x – sin 2x contains
is- the set -
L
 
M O O
 L (A) [2,4] (B) [–1,1]
N P
(A)  ,
2 2Q P
Q M
(B)  ,
2 2N (C) [–2,2] (D) [–4,4]
(C) R (D) None of these | x|
 Q.45 If the domain of the function f(x) = be
x
Q.36 The range of f(x) = sin [x] is -
2 [3,7] then its range is-
(A) {–1,1} (B) {–1,0,1} (A) [–1,1] (B) {–1,1}
(C) {0,1} (D) [–1,1]
(C) {1} (D) {–1}

| x  3|
Q.37 Domain and range of f(x) = are
x3 1
respectively- Q.46 The domain of the function f(x) = is-
x  [ x]
(A) R, [–1,1] (B) R– {3}, {1,–1}
(C) R+, R (D) None of these (A) R (B) R–Z
(C) Z (D) None of these
Q.38 The domain of the function f(x) = sin 1/x is - Q.47 The range of the function
(A) R (B) R+
f(x) = 2 + x – [x–3] is-
(C) R0 (D) R– (A) [5,6] (B) [5,6)
(C) R (D) None of these
Q.39 Range of the function f(x) = 9 – 7 sin x is-
(A) (2,16) (B) [2,16]
Questions
(C) [–1,1] (D) (2,16] based on Value of function
Q.40 For real values of x, range of function
1 Q.48 If f(x) = log x, then f (x/y) equals-
y= is - (A) f(x) + f(y) (B) f(x) – f(y)
2  sin 3 x
(C) f(x) / f(y) (D) f(x) . f(y)
1 1
3   3  
(A) y 1 (B) – y 1
2x
Q.49 If f(x) =
1  x2
, then f (tan  ) equals-
1 1
(C) – > y > – 1 (D) > y > 1
3 3 (A) cot 2  (B) tan 2
(C) sec 2 (D) cos 2 
R
S 1, when x  Q
Q.50 If f(x) = ax, then f(x+ y) equals-
 R, f(x) =
Q.41 If f : R
T1, when x Q , then (A) f(x) + f(y) (B) f(x) – f(y)
image set of R under f is - (C) f(x) f(y) (D) f(x) /f(y)
(A) {1,1} (B) (–1,–1)
(C) {1,–1} (D) None of these Q.51 If f(x) = log x, then correct statement is-
(A) f(x + y) = f( x ) + f(y) (B) f(x + y) = f( x) . f(y)
Q.42 If f : R  R, f(x) = x2, then {x| f (x) = –1} equals- (C) f(xy) = f(x) + f(y) (D) f(xy) = f( x) . f(y)
(A) {–1,1} (B) {1}
(C)  (D) None of these x f (a / b)
Q.52 If f (x) = , then =
x1 f (b / a)
(A) ab (B) a/b
(C) b/a (D) 1

Corporate Office: CP Tower, Road No.1, IPIA, Kota (Raj.), Ph: 0744-2434159 FUNCTION 4
Q.53 If f(x) = 2 cos x + sin2 x, then f(2– x) equals- f ( xy)  f ( x / y)
(A) – f(x) (B) f(x) Q.61 If f(x) = cos (log x), then equals-
(C) – 2f(x) (D) 2f(x) f ( x)f ( y)
(A) 1 (B) –1

If f : R  R, f(x) =
R
S1, when x  Q
(C) 0 (D) 2
Q.54
T1, when x Q , then which
 Q.62 If f (x) = |x| + |x – 1|, then for 0 < x < 1, f (x)
of the following statement is wrong ? equals-

(A) f  2 = –1 (B) f() = –1


(A) 1
(C) 2x + 1
(B) –1
(D) 2x – 1

(C) f(e) = 1 (D) f d4 i = 1 | x|


Q.63 The function f(x) = , x > 0 is -
x
F
GI
H3 J
(A) 0 (B) 1
Q.55 If f(x) = 2 sin x, g(x) = cos2x, then (f + g)
K= (C) 2 (D) –2

2 3 1
(A) 1 (B) Q.64 If f : N  R+, f(x) = x , then the value of
4
f (25)
1 is -
(C) 3 + (D) None of these f (9)  f (16)
4
(A) 0 (B) 1
Q.56 If f : R  R , f(x) = 2x ; g : R  R, g(x) = x + 1, (C) 5/7 (D) 9/7
then (f .g) (2) equals -
(A) 12 (B) 6 Q.65 If f(x) = log ax, then f(ax) equals-
(C) 3 (D) None of these
(A) f(a) f(x) (B) 1+ f(x)
b( x  a) a( x  b) (C) f(x) (D) a f(x)
Q.57 If f(x) = + , then f(a + b) =
(b  a) (a  b)
Q.66 If f(x) = (ax – c)/(cx – a) = y, then f(y)
(A) f(a). f(b) (B) f(a) – f(b)
equals-
(C) f(a) /f(b) (D) f(a) + f(b)
(A) x (B) 1/x
x f (a) (C) 1 (D) 0
Q.58 If f( x) = then is equal to -
x1 f (a  1)
(A) f(–a) (B) f(1/a) Questions
Mapping
based on
Fa I
(D) f G J
(C) f(a2)
Ha  1K Q.67 If f : I  I,f (x) = x3+ 1, then f is -
(A) one - one but not onto
x( x  1) (B) onto but not one-one
Q.59 If f (x) = , then the value of f (x + 2) is-
2 (C) One-one onto
(D) None of these
( x  2)
(A) f (x) + f(x + 1) (B) f(x + 1)
x Q.68 Function f : R  R , f(x) = x |x| is -
(A) one-one but not onto
( x  1) ( x  2)
(C) f(x +1) (D) f(x +1) (B) onto but not one- one
2 2 (C) one-one onto
(D) neither one-one nor onto
Q.60 If f(x + ay, x – ay ) = axy, then f (x,y) equals-
x2
2
x y 2 2
x y 2 Q.69 f:R  R , f(x) = , is -
(A) (B) 1 x2
4 4 (A) many- one function (B) odd function
(C) x2 (D) y2 (C) one- one function (D) None of these

Corporate Office: CP Tower, Road No.1, IPIA, Kota (Raj.), Ph: 0744-2434159 FUNCTION 5
Q.80 Which of the following function is onto ?
1
Q.70 If f : R0  R0, f(x) = , then f is - (A) f : R  R ; f(x) = 3x
x
(A) one-one but not onto (B) f : R  R+; f(x) = e–x
(B) onto but not one-one (C) f: [0,  /2]  [–1,1]; f(x) = sin x
(C) neither one-one nor onto (D) f : R  R: f(x) = cosh x
(D) both one-one and onto
Q.81 Which of the following function defined from
Q.71 Function f : R  R, f(x) = x + |x| is R to R is onto ?
(A) one-one (B) onto (A) f(x) = |x| (B) f(x) = e–x
(C) one-one onto (D) None of these
(C) f(x) = x3 (D) f(x) = sin x.
O
 3 L
P
2 2 M
,
Q.72 Function f :
Q N R, f(x) = tan x is Q.82 If f :   , f(x) = x2 – x, then f is -
(A) one-one (B) onto (A) one-one onto (B) one-one into
(C) one-one onto (D) None of these
(C) many-one onto (D) many-one into
L
M 3 O
Q.73 Function f :
N2 , 2 P
Q [–1,1], f(x) = sin x is - Questions
(A) one-one (B) onto based on Composite function
(C) one-one onto (D) None of these
Q.83 If f(x) = 2x and g is identity function, then-
L1 3 O [–1,1], f(x) = cos x is
Function f : M,  P
(A) (fog) (x) = g(x) (B) (g + g) (x) = g(x)
Q.74
N2 2 Q (C) (fog) (x) = (g + g) (x) (D) None of these
(A) many-one onto (B) onto Q.84 gof exists, when-
(C) one-one onto (D) many one into (A) domain of f = domain of g
Q.75 If f : R  R, f(x) = ex + e–x, then f is - (B) co-domain of f = domain of g
(C) co-domain of g = domain of g
(A) one-one but not onto
(D) co-domain of g = co-domain of f
(B) onto but not one-one
(C) neither one-one nor onto Q.85 If f : R  R, f(x) = x2 + 2x – 3 and g : R  R,
(D) both one-one and onto g(x) = 3x – 4 , then the value of fog (x) is-
Q.76 If f : R  [–1,1], f(x) = sin x, then f is - (A) 3x2 + 6x – 13 (B) 9x2 –18x + 5
(C) (3x– 4) + 2x – 3
2
(D) None of these
(A) one-one onto (B) one-one into
(C) many-one onto (D) many-one into
Q.86 If f : R  R, f(x) = x2 – 5x + 4 and g : R  R,
Q.77 If f : R  R , f(x) = sin2 x + cos2 x , then f is - g(x) = log x , then the value of (gof) (2) is -
(A) one-one but not onto (A) 0 (B) 
(B) onto but not one-one (C) –  (D) Undefined
(C) neither one-one nor onto Q.87 If f : R+  R+,f(x) = x2+ 1/x2 and g : R+  R+,
(D) both one-one onto
g(x) = ex then (gof) (x) equals-
Q.78 Which of the following functions from Z to itself x2 1
are bijections ? (A) e x2  e x 2 (B) e  x2
(A) f(x) = x3 (B) f(x) = x + 2 e
(C) f(x) = 2x + 1 (D) f(x) = x2 + x (C) e2 x  e 2 x (D) e x2 . e x2
Q.79 Which of the following functions from Q.88 If f : R  R, g : R  R and f(x) = 3x + 4 and
A = {x: –1  x  1} to itself are bijections ? (gof) (x) = 2x – 1, then the value of g(x) is -
F
Gx I (A) 2x – 1 (B) 2x – 11
H2 J
x
(A) f(x) =
2
(B) g(x) = sin
K (C)
1
(2x – 11) (D) None of these
(C) h(x) = |x| (D) k(x) = x 2 3

Corporate Office: CP Tower, Road No.1, IPIA, Kota (Raj.), Ph: 0744-2434159 FUNCTION 6
Q.89 If f : R  R, g : R  R and g(x) = x + 3 and R
S1, when x  Q
(fof) ( ) will be-
(fog) (x) = (x + 3)2, then the value of f(–3) is -
(A) –9 (B) 0
Q.98 If f(x) =
(A) 2
T0, when x Q , then
(B) 0
(C) 9 (D) None of these
(C) 1 (D) Undefined
Q.90 If f(x) = ax + b and g(x) = cx + d, then y y
f(g(x)) = g(f(x)) is equivalent to- Q.99 If f(y) = , g(y) = , then
1  y2 1  y2
(A) f(a) = g(c) (B) f(b) = g(b) (fog)(y) equals-
(C) f(d) = g(b) (D) f(c) = g(a)
y y
1 x (A) 2 (B)
Q.91 If f : [0,1]  [0,1], f(x) = . g : [0,1]  [0,1], 1 y 1  y2
1 x
g(x) = 4x (1–x), then (fog) (x) equals- 1  y2
(C) y (D)
1  y2
1  4 x  4 x2 8 x (1  x)
(A) (B) Q.100 If f(x) = [x] and g(x) = cos (x), then the
1  4 x  4 x2 (1  x)2 range of gof is -
1  4 x  4 x2 (A) {0} (B) {–1,1}
(C) (D) None of these (C) {–1,0,1} (D) [–1,1]
1  4 x  4 x2

Questions
Q.92 If f, g, h are three functions in any set, then based on Inverse fucntion
wrong statement is -
(A) (fog)–1 = g–1 of –1 Q.101 If f : R  R, f(x) = x2 + 3, then pre- image of 2
(B) gof  fog under f is -
(C) (fog)oh = fo(goh) (A) {1,–1} (B) {1}
(D) (gof)–1 = g–1of –1
(C) {–1} (D) 

1 x Q.102 Which of the following functions has its inverse-


Q.93 If f(x) = , then f [f (sin)] equals -
1 x (A) f : R  R , f(x) = ax
(A) sin  (B) tan (/2) (B) f : R  R, f(x) = |x| + |x – 1|
(C) cot (/2) (D) cosec  (C) f : R0  R+, f(x) = |x|
(D) f : [, 2]  [–1,1], f(x) = cos x
Q.94 If f(x) = (a – x n)1/n, n  N, then f [f(x)] is equal to-
(A) 0 (B) x Q.103 If function f : R  R+, f(x) = 2x , then f –1 (x) will
(C) xn (D) (an – x)n be equal to-
(A) logx 2 (B) log2 (1/x)
 3x  x 3  (C) log2 x (D) None of these
 1 x   
Q.95 If f (x) = log   and g(x) =  1  3x 2  ,
 1 x    ex  e x
Q.104 The inverse of the function f(x) = +2
then f[g(x)] is equal to- e x  e x
(A) –f(x) (B) 3f(x) is given by -
(C) [f(x)]3 (D) None of these Fx  2I
(A) log G J
1/ 2
Fx  1I
(B) log G J
1/ 2

Q.96 If  (x) = x2 + 1 and  (x) = 3x, then  {  (x)}


Hx  1K Hx  1K
and  {  (x)} = F x I1/2
(C) log G J
Fx  1I1/2
(D) log G J
2
1
2
1
H2  xK H3  x K
(A) 32x+1, 3x (B) 32x+1, 3x 1
x 2 1 Q.105 If f : [1, )  [2, ) is given by ƒ(x) = x +
(C) 3 +1, 3
2x
(D) None of these x
then f–1(x) equals -
R
S1, when x  Q
4) x  x2  4 x
Q.97 If function f(x) =
T0, when x Q , (fof) ( (A)
2
(B)
1 x2
the value will be -
(A) 0 (B) 2 x  x2  4
(C) (D) 1 + x 2  4
(C) 1 (D) None of these 2
Corporate Office: CP Tower, Road No.1, IPIA, Kota (Raj.), Ph: 0744-2434159 FUNCTION 7
Q.106 If f(x) = loge(x + 1  x2 ), then f –1
(x) equals-

(A) log (x – 1  x2 )

ex  e x
(B)
2

ex  e x
(C)
2

ex  e x
(D)
ex  e x

Q.107 If f(x) = x3 – 1 and domain of f = {0,1,2,3},


then domain of f–1 is -
(A) {0,1,2,3}
(B) {1,0,–7,–26}
(C) {–1,0,7,26}
(D) {0,–1,–2,–3}

Q.108 If f(x) = {4 – (x – 7)3}1/5, then its inverse is-


(A) 7 – (4 – x5)1/3 (B) 7 – (4 + x5)1/3
(C) 7 + (4 – x5)1/3 (D) None of these

Q.109 If f : R  R, f(x) = ex and g : R  R,


g(x) = 3x – 2 , then the value of (fog)–1(x) is
equal to -
2  log x
(A) log (x – 2) (B)
3
F
Gx  3I
(C) log
H2 JK (D) None of these

Corporate Office: CP Tower, Road No.1, IPIA, Kota (Raj.), Ph: 0744-2434159 FUNCTION 8
LEVEL # 2
1
Q.1 If f(x) = x + , then - Q.11 Function f : R  R + , f(x) = x 2 + 2 and
x
(A) f(x2) = [f(x)]2 (B) f(x + y) = f(x) + f(y) F
G 1 IJ
(C) f(–x) = f(x) (D) f(1/x) = f(x)
g : R+
H K
 R, g(x) = 1  1  x then the value of
gof (2) is -
Q.2 If x is the radius of a circle and f(x) = x2, then
(A) 5/6 (B) 8/7
domain of f is -
(C) 1/6 (D) 6/5
(B) R (B) R+
(C) R ¯
(D) R0
Q.12 The period of function f (x) = |sin3 (x/2)| is
(A) 4  (B) 16 
1
Q.3 If f(x) = x2 – 3x + 1 and g(x) = , then (C) 2  (D) None of these
x2
domain of (f – g) is - Q.13 The inverse of the function y = logex is -
(A) R (B) R+ (A) 10x (B) 10–x
(C) R – {2} (D) None of these (C) ex
(D) e–x
Q.4 If f : R  R, f(x) = tan x, then pre-image of
–1 under f is - 1 x
Q.14 If f(x) = log , when – 1 < x1, x2< 1, then
R
S
(A) n 

n I
U
V R
S
(B) n 

n I
U
V
1 x
f(x1) + f(x2) equals-
T 4 W T 4 W F
x1  x2 IJ F
x1  x2 IJ
(C) {n  | n I } (D) None of these G
H K
(A) f 1  x x
1 2
G
H K
(B) f 1  x x
1 2

x2  2 x  1 Fx1  x2 IJ Fx1  x2 IJ
(C) f G (D) f G
Q.5 f(x) =
x2  3 x  2
is not defined for-
H1 x1x2 K H1 x1x2 K
(A) x = 2 (B) x = 1, 2
(C) x = 2,–1 (D) x = 0 Q.15 Function f : [–1,1]  R, f(x) = sin (  /2) x is -
(A) one-one (B) onto
Q.6 If f : R  R, f(x) = x3 + 3, and g : R  R,
(C) one-one onto (D) None of these
g(x) = 2x + 1, then f–1og–1(23) equals-
(A) 2 (B) 3 Q.16 If the domain of function f(x) = x2 – 6x + 7 is
(C) (14)1/3 (D) (15)1/3 (–  ,  ), then the range of function is -
Q.7 If f(x) = log x, g(x) = x3, then f[g(a)] + f [g(b)] is (A) (–  ,  ) (B) [–2,  )
equal to- (C) (–2,3) (D) (–  ,–2)
(A) f [g(a) + g(b)] (B) 3 f(ab)
(C) g [f(ab)] (D) g [f(a) + f(b)] Q.17 Function f : R  R, f(x) = [x] is -
(A) one-one onto (B) one-one into
Q.8 Function sin–1 x is defined in the interval- (C) many-one onto (D) many-one into
(A) (–1,1) (B) [0,1]
(C) [–1,0] (D) (–1,2) Q.18 If S be the set of all triangles and f : S  R +,
f (  ) = Area of  , then f is -
 (A) One-one onto (B) one-one into
Q.9 The interval for which sin–1 x + cos–1 x= (C) many-one onto (D) many-one into
2
holds-
(A) [0,  ) (B) [0,3] Q.19 If f : C  R , f(z) = |z|, then f is -
(C) [0,1] (D) [0,2] (A) one-one but not onto
(B) onto but not one-one
Q.10 The domain of the function f(x) = x! is - (C) neither one-one nor onto
(A) (0,  ) (B) N (D) both one-one and onto
(C) W (D) R+

Corporate Office: CP Tower, Road No.1, IPIA, Kota (Raj.), Ph: 0744-2434159 FUNCTION 9
L 1 , 1 O
If f : M
Q.29 If f(x) = x2 – x–2, then f(1/x) equals-
Q.20
N2 2 P Q [–1,1], f(x) = sin x, then f (A)
1
(B) –1/f(x)
is - f ( x)
(A) one-one (B) one-one onto (C) f(x) (D) – f(x)
(C) onto (D) None of these

Q.21 If f(x) = 1/x then f(a) – f(b) equals- Q.30 The domain of function
F
Gb  aI F ab I 1
(A) f
Hab JK (B) f G
Ha  b J
K f(x) = log (3  x) + x  2 is -
10
(A) [–2, 3) (B) [–2, 3) – {2}
Fab I
(C) f G J
Fa  b I
(D) f G J
(C) [–3, 2] (D) [–2, 3] – {2}
Hb  a K Ha  b K x3
Q.31 Domain of the function f(x) = is-
x , correct statement is - ( x  1) x2  4
Q.22 f(x) = cos
(A) (1,2)
(A) f(x) is periodic and its period = 2 (B) (–  , –2)  (2,  )
(B) f(x) is periodic and its period = 4  2 (C) (–  ,–2)  (1,  )
(D) (–  ,  ) – {1,  2}
(C) f(x) is periodic and its period = 
(D) f(x) is not periodic Q.32 Range of the function f(x) = sin2(x4) + cos2(x4)
is-
Q.23 If f be the greatest integer function and g be the (A) (–  ,  ) (B) {1}
modulus function, then (C) (–1,1) (D) (0,1)
F
G 5I
J F 5 IJ=
– (fog) G
(gof) 
H3 K H3 K Q.33 Let f : R  R be a function defined by
f(x) = x + x 2 , then f is-
(A) 1 (B) –1
(C) 2 (D) 4 (A) injective (B) surjective
(C) bijective (D) None of these
Q.24 The domain of function f(x) = log |log x| is-
(A) (0,  ) (B) (1,  )
(C) (0,1)  (1,  ) (D) (–  ,1) Q.34 If f (x) = e3x and g(x) =  n x, x > 0, then (fog) (x)
is equal to-
Q.25 Domain of the function tan–1 x + cos–1 x2 is - (A) 3x (B) x3
(A) R– [–1,1] (B) R– (–1,1) (C) log 3x (D) 3 log x
(C) (–1,1) (D) [–1,1]
Q.35 If f : R  R f(x) = cos (5x + 2) then the value
Q.26 Which of the following functions are equal ? of f –1(x) is -
(A) f(x) = x, g(x) = x
2 cos 1( x)  2
(A) (B) cos 1 ( x )  2
(B) f(x) = log x2 , g(x) = 2 log x 5
(C) f(x) = 1, g(x) = sin2x + cos2 x
cos 1( x)
(D) f(x) = x/x, g(x) = 1 (C) 2 (D) Does not exist
5
Q.27 If f : Q  Q, f(x)= 2x and g : Q  Q, R
|S4  x2 U
|
g(x) = x + 2, then (fog)–1(20) equals- Function f(x) = sin log (1  x) Vhas domain
(A) 10 (B) 12
Q.36
|T |W
(C) 8 (D) 6 (A) [–2,1) (B) [–2,1]
(C) (–2,1) (D) (–  ,1)
2 cosh x  sin2 x
Q.28 f(x) = is - Q.37 The domain of function
x2  1
f(x) = log (3x –1) + 2 log (x +1) is -
(A) an algebric function
(A) [1/3,  ) (B) [–1,1/3]
(B) a trigonometrical function
(C) (–1,1/3) (D) None of these
(C) an even function
(D) an implicit function
Corporate Office: CP Tower, Road No.1, IPIA, Kota (Raj.), Ph: 0744-2434159 FUNCTION 10
x
F
G 1I
H xJ
1
Q.38 If f(x) =
1  x2
, then (fofof) (x) is equal to- Q.44 Let f x 
K= x + x2 (x  0), then f(x) equals-
2

3x x (A) x2 – 2 (B) x2 –1
(A) (B) (C) x2 (D) None of these
1  x2 1  3 x2
Q.45 The graph of f(x) = – |x| is -
3x
(C) (D) None of these
1  x2

Q.39 Which one of the following graphs represents


the function y = 1+ |x| for all x  R ?
(A) (B)
(A)

(B)

(C) (D)

(C) (D) Q.46 If a2 + b2 + c2 = 1, then range of ab + bc + ca is-


(A) [–1/2,  ) (B) (0,  )
(C) [–1/2,1] (D) [1,  )

Q.40 If f (x) = x3 – x and g(x) = sin 2x, then - Q.47 If x = logabc, y = log b ca, and z = logcab, then
(A) g [f(1)] = 1 1 1 1
(B) f (g (/12)) = – 3/8 + + equals-
1 x 1 y 1 z
(C) g {f(2)} = sin 2
(A) 1 (B) x + y + z
(D) None of these
(C) abc (D) ab + bc + ca
1 1 Q.48 The range of 5 cos x – 12 sin x + 7 is -
Q.41 If f(x) = and g (x) = , then (A) [–6,20] (B) [–3,18]
x1 x 1
common domain of function is - (C) [–6,15] (D) None of these
(A) {x | x <1, x  R } Q.49 The domain of the function log 2 log 3 log 4(x)
(B) {x | x  0, x  1, x  R} is -
(C) {1} (A) (1,  ) (B) (2,  )
(D) {–1} (C) (3,  ) (D) (4,  )

Q.42 The natural domain of the real valued function

defined by f (x) = x2  1 + x2  1 is-


(A) 1 < x <  (B) –  < x < 
(C) –  < x <–1 (D) (–  ,  ) – (–1,1)

9  x2
Q.43 If f(x) = , then domain of f is -
sin 1(3  x)
(A) [2,3] (B) [2,3)
(C) (2,3] (D) None of these

Corporate Office: CP Tower, Road No.1, IPIA, Kota (Raj.), Ph: 0744-2434159 FUNCTION 11
LEVEL # 3
Q.1 The domain of definition of Q.7 The value of nI for which the function

 x 1  1 sin nx
log0.4   2 f(x) = has 4 as its period is -
f(x) = is– x
 x  5  x  36 sin  
n
(A) (x : x < 0, x  – 6}
(A) 2 (B) 3 (C) 4 (D) 5
(B) (x : x > 0, x  1, x  6}
(C) (x : x > 1, x  6} Q.8 If f(x) is an odd periodic function with period
(D) (x : x  1, x  6} 2, then f (4) equals to -
(A) 0 (B) 2
(C) 4 (D) –4
Q.2 The function f : R  R defined by
f (x) = (x – 1) (x – 2) (x – 3) is -
Q.9 Domain of the function
(A) one-one but not onto
1
 x 2 
(B) onto but not one-one f(x) = sin  log 5 is -
(C) both one and onto  5 

(D) neither one-one nor onto (A) [–5, –1]  [1, 5] (B) [–5, 5]
(C) (–5, –1)  (1, 5) (D) None of these

Q.3 Set A has 3 elements and set B has 4


elements. The number of injections that can 1 | x |
be defined from A to B is - Q.10 Domain of f(x) = is -
2 | x |
(A) 144 (B) 12 (A) R – [–2, 2]
(C) 24 (D) 64 (B) R – [–1, 1]
(C) [–1, 1]  (–, –2)  (2, )
Q.4 The number of bijective functions from set A (D) None of these
to itself when a contains 106 elements -
(A) 106 (B) (106)2
  
 4  x2   is -
(C) 106! (D) 1106 Q.11 Range of sin  log 
  1 x  
  
Q.5 Let A be a set containing 10 distinct (A) (–1, 1) (B) [–1, 1]
elements, then the total number of distinct (C) R (D) None of these
functions from A to A is -
(A) 10 ! (B) 1010
(C) 210 (D) 210 – 1 2
Q.12 If f(x) = 3 sin  x 2 , then values of f(x) lie
16
in -
Q.6 Let f : R  R be a function defined by
  
(A)  ,  (B) [–2, 2]
e | x|  e  x  4 4
f (x)  . Then -
e x  e x  3 
(C) 0 ,  (D) None of these
(A) f is a bijection  2
(B) f is an injection only
(C) f is a surjection only
Q.13 The function f (x) = cos (log (x + x 2  1 )) is-
(D) f is neither an injection nor a surjection
(A) even (B) odd
(C) constant (D) None of these
Corporate Office: CP Tower, Road No.1, IPIA, Kota (Raj.), Ph: 0744-2434159 FUNCTION 12
Q.14 The function f(x) = max. [1 – x, 1 + x, 2] Q.20 If [x] denote the greatest integer  x, the
x  R is equivalent to - domain of definition of function

 1  x , x  1 4  x2
 f (x) = is -
(A) f ( x )  2 ,  1  x  1 [ x]  2
 1 x, x  1 (A) (–, –2)  [–1, 2] (B) [0, 2]

(C) [–1, 2] (D) (0, 2)
 1  x , x  1
 Q.21 The function f : [–1/2, 1/2]  [–/2, /2]
(B) f ( x )  2 ,  1  x  1
 1 x , x  1 defined by f(x) = sin–1(3x – 4x3) is–
 (A) both one-one and onto
(B) neither one-one nor onto
1  x , x  1
 (C) onto but not one-one
(C) f ( x )  1,  1  x  1
(D) one-one but not onto
 1 x, x  1

(D) None of these Q.22 The function f satisfies the functiona equation
 x  59 
3f (x) + 2f    10 x  30 for all real x  1.
Q.15 The domain of the function f(x) = 9–xPx–5 is-  x 1 
(A) [5, 7] (B) {5, 6, 7} The value of f (7) is -
(C) {3, 4, 5, 6, 7} (D) None of these (A) 8 (B) 4
(C) –8 (D) 11
Q.16 The range of the function f(x) = 9–xPx–5 is -
(A) {1, 2, 3} (B) [1, 2] Q.23 The domain of the function
(C) {1, 2, 3, 4, 5} (D) None of these f (x) = log 3+x(x2 – 1) is -
(A) (–3, –1)  (1, )
Q.17 Domain of the function (B) [–3, –1)  [1, )
 (C) (–3, –2)  (–2, –1)  (1, )
 1  
f ( x )  log 2   log1 / 2 1    1 is - (D) [–3, –2)  (–2, –1)  [1, )
 4
x 
   
(A) (0, 1) (B) (0, 1] Assertion & Reason Type Question :-
(C) [1, ) (D) (1, )
All questions are Assertion & Reason type
Q.18 The period of f(x) = [sin 5x] + |cos 6x| is - questions. Each of these questions contains
two statements : Statement-I (Assertion) and

(A) (B)  Statement-2 (Reason). Answer these ques
2
tions from the following four option.
2 (A) Statement-1 is false. Statement-2 is true
(C) 2 (D)
5 (B) Statement-1 is true. Statement-2 is true;
Statement-2 is a correct explanation for
x x Statement-1
Q.19 Period of f (x) = sin x + tan + sin 2 +
2 2 (C) Statement-1 is true. Statement-2 is true;
x x x Statement-2 is not a correct explanation
tan + ... + sin n  1 + tan n is -
23 2 2 for Statement-1
(A)  (B) 2 (D) Statement-1 is true. Statement-2 is false

(C) 2n (D)
2n

Corporate Office: CP Tower, Road No.1, IPIA, Kota (Raj.), Ph: 0744-2434159 FUNCTION 13
Q.24 Statement-1 : The period of Passage :-
1 Let here we define f : R  [–1, 1] and
f(x) = sin 2x cos [2x] – cos 2x sin [2x] is g : R  [–1, 1]. Now f(x) = 2 cos2 x – 1,
2
Statement-2 : The period of x – [x] is 1 g(x) = cos 2x, h(x) = f(x) + g(x),
f(x)
I(x) = f(x) – g(x), j (x) = g(x) are 5 functions.
Q.25 Statement-1 :
If f(x) = |x – 1| + |x – 2| + |x – 3| On the basis of above information, answer
Where 2 < x < 3 is an identity function. the following questions-
Statement-2 : f : A  A defined by
f(x) = x is an identity function. Q.31 Which statement is correct-
(A) Period of f(x), g(x) and h(x) are same
Q.26 Statement-1 : f : R  R defined by 2
and value is
f(x) = sin x is a bijection 3
(B) Period of f(x), g(x) and h(x) makes
Statement-2 : If f is both one and onto it is

bijection the A.P. with common difference
4
Q. 27 Statement-1 : f : R  R is a function defined (C) Sum of periods of f(x), g(x) and
2x  1 h(x) is 3
by f(x) = . (D) None of these
3
–1 3x  1 Q.32 Which statement is correct regarding function
Then f (x) =
2 j(x) and I(x)-
Statement-2 : f(x) is not a bijection. (A) The domain of j(x) and I(x) are the
same
(B) Range of j(x) and I(x) are the same
Q.28 Statement-1 : If f is even function, g is odd (C) The union of domain of j(x) and I(x) are all
f real numbers
function then g , (g  0) is an odd function. (D) None of these

Statement-2 : If f(–x) = –f(x) for every x of its Q.33 If the solution of equation I(x) – g(x) = 0 are
domain, then f(x) is called an odd function and x1, x2, x3, .... xn when x  [0, 10] then which
if f(–x) = f(x) for every x of its domain, then f(x) option is correct-
(A) x 1 , x 2 , x 3 ... x n makes the A.P. with
is called an even function. common difference 
(B) Total no. of solutions of I (x) – g(x) = 0 is 20
Q.29 Statement 1 : Function f(x) = sinx + {x} is for x  [0, 10]
periodic with period 2 (C) Sum of all solutions of the given
equation is 100 in the interval [0, 10]
Statement 2 : sinx and {x} are both periodic
(D) (B) and (C) are correct
with period 2 and 1 respectively.
Q.34 If h : R  [–2, 2], then -
(A) h(x) is one-one function
2 (B) h(x) is one-one and onto function
Q.30 Statement 1 : y = f(x) = x  2x  4 , (C) h(x) is onto function
x 2  2x  5
x  R Range of f(x) is [3/4, 1) (D) h(x) is many one and into function
Q.35 Domain and range of j(x) respectively -
Statement 2 : (x – 1)2 = 4 y  3 . (A) R and {1}
1 y (B) R and {0, 1}
(C) R – {(2n + 1) /4}, n I and {1}
(D) R – {(2n + 1) /2}, n  I and {1}

Corporate Office: CP Tower, Road No.1, IPIA, Kota (Raj.), Ph: 0744-2434159 FUNCTION 14
LEVEL # 4
(Questions asked in Previous AIEEE & IIT-JEE)

SECTION - A Q.8 A function f from the set of natural numbers to


Q.1 Which of the following is not a periodic function - integers defined by
[AIEEE 2002]
 n 1
(A) sin 2x + cos x (B) cos x  , when n is odd
(C) tan 4x (D) log cos 2x f(n) =  2 n is
 , when n is even
 2
[AIEEE 2003]
Q.2 The period of sin2 x is- [AIEEE 2002]
(A) neither one-one nor onto
(A) /2 (B) 
(C) 3/2 (D) 2 (B) one-one but not onto
(C) onto but not one-one
Q.3 The function f : R  R defined by f(x) = sin x is- (D) one-one and onto both
[AIEEE-2002]
Q.9 The range of the function f(x) = 7– xPx–3 is-
(A) into (B) onto
[AIEEE 2004]
(C) one-one (D) many-one
(A) {1, 2, 3} (B) {1, 2, 3, 4, 5, 6}
(C) {1, 2,3,4} (D) {1, 2, 3, 4, 5}
2x
Q.4 The range of the function f(x) = , x  2 is -
2x
Q.10 If f : R  S, defined by f(x) = sin x – 3 cos x + 1,
[AIEEE-2002]
(A) R (B) R – {–1} is onto, then the interval of S is-
(C) R – {1} (D) R – {2} [AIEEE 2004]
(A) [0, 3] (B) [–1, 1]
(C) [0, 1] (D) [–1, 3]
Q.5 The function f(x) = log (x + x 2  1 ), is-
[AIEEE 2003] Q.11 The graph of the function y = f(x) is symmetrical
(A) neither an even nor an odd function about the line x = 2, then- [AIEEE 2004]
(B) an even function (A) f(x+ 2) = f(x – 2) (B) f(2 + x) = f(2 – x)
(C) an odd function (C) f(x) = f(–x) (D) f(x) = – f(–x)
(D) a periodic function

Q.6 Domain of definition of the function sin 1( x  3 )


Q.12 The domain of the function f(x) = is-
3 9  x2
f(x) = + log10 (x3 – x), is- [AIEEE 2003]
4  x2
[AIEEE 2004]
(A) (– 1, 0)  (1, 2)  (2, ) (A) [2,3] (B) [2,3)
(B) (1, 2)
(C) [1,2] (D) [1, 2)
(C) ( – 1, 0) (1, 2)
(D) (1, 2)  (2, )
Q.13 Let f : (–1, 1)  B, be a function defined by

Q.7 If f : R  R satisfies f(x+ y) = f(x) + f(y), for all 2x


f(x) = tan–1 , then f is both one-one and
n 1 x2
x, y  R and f(1) = 7, then  f (r ) is- onto when B is the interval - [AIEEE-2005]
r 1
[AIEEE 2003]    
(A)  0,  (B) 0, 
7n (n  1) 7n  2  2
(A) (B)
2 2      
7(n  1) (C)  ,  (D)   , 
 2 2  2 2
(C) (D) 7n (n+1)
2
Corporate Office: CP Tower, Road No.1, IPIA, Kota (Raj.), Ph: 0744-2434159 FUNCTION 15
Q.14 A real valued function f(x) satisfies the functional SECTION - B
equation f(x – y) = f(x) f(y) – f (a – x) f(a + y)
where a is a given constant and f(0) = 1, then 1  x 
Q.1 If function f(x) = – tan  2  ; (–1 < x < 1)
f(2a – x) is equal to - [AIEEE-2005] 2  
(A) –f(x) (B) f(x)
and g(x) = 3  4 x  4 x 2 , then the domain of
(C) f(a) + f(a – x) (D) f(–x)
gof is – [IIT 90]
 1 1
    (A) (–1, 1) (B)   2 , 2 
Q.15 The largest interval lying in  ,  for which  
2 2
the function is defined, is- [AIEEE 2007]  1  1 
(C)   1, 2  (D)   2 , 1
   
   
(A) [0, ] (B)  , 
2 2
Q.2 If f(x) = cos [2]x + cos [–]x, where [x]
    
(C)  ,  (D) 0,  stands for the greatest integer function, then
 4 2  2 [IIT 91]
Q.16 Let f : N  Y be a function defined as 
(A) f  2  = –1 (B) f () = 1
 
f(x) = 4x + 3 where Y = |y  N : y = 4x + 3 for

some x  N|. Show that f is invertible and its (C) f  4  = 2 (D) None of these
 
inverse is [AIEEE 2008] Q.3 The value of b and c for which the identity
f(x + 1) – f(x) = 8x + 3 is satisfied,
y3 y3
(A) g(y) = 4 + (B) g(y) = where f(x) = bx2 + cx + d, are [IIT 92]
4 4
y 3 3y  4 (A) b = 2, c = 1 (B) b = 4, c = –1
(C) g(y) = (D) g(y) = (C) b = –1, c = 4 (D) None
4 3

Q.17 For real x, let f(x) = x3 + 5x + 1, then - Q.4 Let f(x) = sin x and g(x) = ln |x|. If the ranges
of the compositie functions fog and gof are
[AIEEE 2009]
R1 and R2 respectively, then – [IIT 94]
(A) f is one – one but not onto R (A) R1 = {u : –1 < u < 1},
(B) f is onto R but not one – one R2 = {v : –  < v < 0}
(C) f is one – one and onto R (B) R1 = {u : – < u < 0},
(D) f is neither one – one nor onto R R2 = {v : – 1 < v < 1}
(C) R1 = {u : –1 < u < 1},
Q.18 Let f(x) = (x + 1)2 –1, x > –1 R2 = {v : –  < v < 0}
Statement – 1 : [AIEEE 2009] (D) R1 = {u : –1 < u < 1},
The set {x : f(x) = f–1(x)} = {0, –1}. R2 = {v : –  < v < 0}
Statement – 2 :
f is a bijection. Q.5 Let 2 sin2 x + 3 sin x – 2 > 0 and x2 – x –
(A) Statement -1 is true, Statement -2 is true; 2 < 0 (x is measured in radians). Then x lies
Statement -2 is a correct explanation for in the interval [IIT 94]
Statement -1
  5   5 
(B) Statement -1 is true, Statement -2 is true; (A)  6 , 6  (B)   1, 6 
Statement -2 is not a correct explanation    
for Statement -1.
 
(C) Statement -1 is true, Statement -2 is false. (C) (–1, 2) (D)  6 ,2 
 
(D) Statement -1 is false, Statement -2 is true.
Corporate Office: CP Tower, Road No.1, IPIA, Kota (Raj.), Ph: 0744-2434159 FUNCTION 16
Q.6 Let f(x) = (x + 1)2 – 1, (x > – 1). Then the
Q.11 If g(f(x)) = |sin x| and f(g(x)) = (sin x )2,
set S = {x : f(x) = f –1(x)} is – [IIT 95]
then [IIT 98]
(A) Empty
(A) f(x) = sin2 x, g(x) = x
(B) {0, –1}
(B) f(x) = sin x, g(x) = |x|
(C) {0, 1, –1}
 (C) f(x) = x2, g(x) = sin x
 3  i 3  3  i 3 
(D) 0,1, 2
,
2
 (D) f and g cannot be determined
 

Q.12 If f(x) = 3x – 5, then f–1 (x) [IIT 98]


Q.7 If f(1) = 1 and f(n + 1) = 2f(n) + 1 if n  1, 1
then f(n) is- [IIT 95] (A) is given by
3x  5
(A) 2n+1 (B) 2n x5
(B) is given by
(C) 2n – 1 (D) 2n–1 – 1 3
(C) does not exist because f is not one - one
(D) does not exist because f is not onto
Q.8 If f is an even function defined on the interval
(– 5, 5), then the real values of x satisfying Q.13 If the function f : [1, )  [1, ) is defined
 x 1 by f(x) = 2x(x–1) , then f–1 (x) is [IIT 99]
the equation f(x) = f   are-
 x  2  1
x ( x 1)

(A)  
 2
[IIT 96]

(A)
 1 5  3  5
,
(B)
1
2

1  1  4 log2 x 
2 2

(B)
 1 3  3  3
2
,
2
(C)
1
2

1  1  4 log2 x 
2 5 (D) not defined
(C)
2
(D) None of these Q.14 The domain of definition of the function y(x)
given by the equation 2x + 2y = 2 is –
[IIT Scr. 2000]
  
Q.9 Let f(x) = [x] sin   , where [.] denotes the (A) 0 < x < 1 (B) 0 < x < 1
 [ x  1] 
greatest integer function. The domain of f is ....... (C) – < x < 0 (D) – < x < 1
(A) {x  R| x  [–1, 0)}
(B) {x  R| x  [1, 0)}
Q.15 Let f() = sin (sin + sin 3), then f()
(C) {x  R| x  [–1, 0)} [IIT 2000]
(D) None of these [IIT 96]
(A)  0 only when   0
(B)  0 for all 
  (C)  0 for all real 
Q.10 If f(x) = sin2x + sin2  x  3  + cos x cos
  (D)  0 only when   0
  5
 x   and g   = 1, then (gof) (x) =
 3 4 Q.16 The number of solutions of log4 (x – 1) =
[IIT 96] log2 (x – 3) is – [IIT Scr. 2001]
(A) –2 (B) –1 (A) 3 (B) 1
(C) 2 (D) 1 (C) 2 (D) 0
Corporate Office: CP Tower, Road No.1, IPIA, Kota (Raj.), Ph: 0744-2434159 FUNCTION 17
x Q.22 Let function f : R  R be defined by
Q.17 Let f(x) = , x  – 1, then for what value
x 1 f(x) = 2x + sin x for x  R. Then f is–
of  f{f(x)} = x. [IIT Scr. 2001]
[IIT Scr. 2002]
(A) 2 (B) – 2 (A) one to one and onto
(C) 1 (D) –1 (B) one to one but NOT onto
(C) onto but NOT one to one
(D) neither one to one nor onto
log2 ( x  3)
Q.18 The domain of definition of f (x) =
x2  3 x  2 x
is – [IIT Scr. 2001] Q.23 Let f(x) = defined as [0, )  [ 0, ),
1 x
(A) R / { –2, –2} f(x) is– [IIT Scr.2003]
(B) (– 2, )
(A) one one & onto
(C) R/ {–1, –2, –3} (B) one- one but not onto
(D) (–3, ) / {–1, –2}
(C) not one-one but onto
(D) neither one-one nor onto
1
Q.19 If f : [1, )  [2, ) is given by f(x) = x +
x x2  x  2
then f–1 (x) equals – [IIT Scr. 2001] Q.24 Find the range of f(x) = is–
x2  x  1
[IIT Scr.2003]
x  x2  4 x
(A) (B)  11 
2 1  x2 (A) (1, ) (B) 1, 
 7
x  x2  4  7  7
(C) (D) 1 + x2  4
2 (C) 1,  (D) 1, 
 3  5

Q.20 Let g(x) = 1 + x – [x] and Q.25 Domain of f(x) = sin 1(2x )   / 6 is–
[IIT Scr.2003]
 1 ; x  0
  1 1  1 1
f(x) =  0 ; x  0 . Then for all x, f(g(x)) is (A)  , (B)  ,
1 ; x0  4 2   2 2 

 1 1  1 1
(C)  , (D)  ,
equal to :
 4 4   2 4 
(where [.] denotes the greatest integer
function): [IIT Scr. 2001] Q.26 Let f(x) = sinx + cos x & g(x) = x2 – 1, then
(A) x (B) 1 g(f(x)) will be invertible for the domain-
(C) f(x) (D) g(x) [IIT Scr.2004]
  
(A) x  0,  (B) x   , 
Q.21 Suppose f(x) = (x + 1)2 for x  – 1. If g(x) is  4 4
the function whose graph is the reflection of     
(C) x  0,  (D) x   , 0
the graph of f(x) with respect to the line  2  2
y = x, then g(x) equals– [IIT Scr. 2002]
x x  Q 0 x  Q
(A) – x – 1, x  0 Q.27 f (x)   ; g (x)  
0 x  Q x x  Q
1
(B) , x > – 1 then (f – g) is [IIT Scr.2005]
( x  1)2
(A) one-one , onto
(C) x 1, x  – 1 (B) neither one-one, nor onto
(C) one-one but not onto
(D) x – 1, x  0
(D) onto but not one-one

Corporate Office: CP Tower, Road No.1, IPIA, Kota (Raj.), Ph: 0744-2434159 FUNCTION 18
ANSWER KEY
LEVEL # 1
Q.No. 1 2 3 4 5 6 7 8 9 10 11 12 13 14 15 16 17 18 19 20
Ans. C A C B B A C D C C D A B A A D A B C B

Q.No. 21 22 23 24 25 26 27 28 29 30 31 32 33 34 35 36 37 38 39 40
Ans. B A B A C B D B B B B B B B B B B B B A
Q.No. 41 42 43 44 45 46 47 48 49 50 51 52 53 54 55 56 57 58 59 60
Ans. C C C B C B B B B C C B B C C A D C B B
Q.No. 61 62 63 64 65 66 67 68 69 70 71 72 73 74 75 76 77 78 79 80
Ans. D A B C B A A C A D D C C D C C C B B B
Q.No. 81 82 83 84 85 86 87 88 89 90 91 92 93 94 95 96 97 98 99 100
Ans. C D C B B D D C C C A D A B B C C C C B
Q.No. 101 102 103 104 105 106 107 108 109
Ans. D D C D A C C C B

LEVEL # 2
Q.No. 1 2 3 4 5 6 7 8 9 10 11 12 13 14 15 16 17 18 19 20
Ans. D B C A B A B B C C D C C A A B D C C B
Q.No. 21 22 23 24 25 26 27 28 29 30 31 32 33 34 35 36 37 38 39 40
Ans. C D A C D C C C D B B B A B D C D B C B
Q.No. 41 42 43 44 45 46 47 48 49
Ans. B D B A C C A A D

LEVEL # 3
Q.No. 1 2 3 4 5 6 7 8 9 10 11 12 13 14 15 16 17 18 19 20
Ans. C B C C B D A A A C B C A A B A A C C A

Q.No. 21 22 23 24 25 26 27 28 29 30 31 32 33 34 35
Ans. A B C B B A D B A B C C D C C

LEVEL # 4
SECTION - A
Q.No. 1 2 3 4 5 6 7 8 9 10 11 12 13 14 15 16 17 18
Ans. B B A,D B C A A D A D B B D A D C C B

SECTION - B
Q.No. 1 2 3 4 5 6 7 8 9 10 11 12 13 14 15
Ans. B A B D D B C A C D A B B D C
Q.No. 16 17 18 19 20 21 22 23 24 25 26 27
Ans. B D D A B D A B C A B A

Corporate Office: CP Tower, Road No.1, IPIA, Kota (Raj.), Ph: 0744-2434159 FUNCTION 19
FUNCTION

Preface

As you have gone through the theory part that consists of given fundamental principles,
definitions, concepts involved and solved problems. After going through theory part it
becomes necessary to solve the unsolved problems based on the concepts given. To
solve this purpose we are providing exercise part that comprises of various exercises
based on the theory. By solving various kinds of problems you can check your grasp on
the topic and can determine whether you have been able to find optimum depth in relevant
topic or not.

Students are advised to solve the questions of exercises (Levels # 1, 2, 3, 4) in the same
sequence or as directed by the faculty members, religiously and very carefully.

Level # 3 is not for foundation course students, it will be discussed in fresher & target
courses.

The list of exercises is as following :

Total No.of questions in Function are -

Level # 1 ........................................ 109


Level # 2 ........................................ 49
Level # 3 ........................................ 35
Level # 4 ........................................ 45

Total No. of questions......................................................... 238

Corporate Office: CP Tower, Road No.1, IPIA, Kota (Raj.), Ph: 0744-2434159 FUNCTION 1
LEVEL # 1
Questions
inequation Q.8 If x2 – 1  0 and x2 – x – 2  0, then x line in the
based on
interval/set
(A) (–1, 2) (B) (–1, 1)
2 (C) (1, 2) (D) {– 1}
Q.1 The inequality < 3 is true, when x belongs to-
x
2   2 Questions
Definition of function
(A)  ,   (B)     based on
3   3

2  Q.9 Which of the following relation is a function ?


(C)  ,    (–, 0) (D) none of these (A) {(1,4), (2,6), (1,5), (3,9)}
3  (B) {(3,3), (2,1), (1,2), (2,3)}
(C) {(1,2), (2,2,), (3,2), (4,2)}
(D) {(3,1), (3,2), (3,3), (3,4)}
x4
Q.2 < 2 is satisfied when x satisfies-
x 3 Q.10 If x, y  R, then which of the following rules is
(A) (–, 3) (10, ) (B) (3, 10) not a function-
(C) (–, 3) [10, ) (D) none of these (A) y = 9 –x2 (B) y = 2x2
(C) y = x – |x| (D) y = x2 + 1
x7
Q.3 Solution of > 2 is-
x3 Questions
(A) (–3, ) (B) (–, –13) based on Even and odd function
(C) (–13, –3) (D) none of these
Q.11 Which one of the following is not an odd
2x  3 function -
Q.4 Solution of  3 is- (A) sin x (B) tan x
3x  5
(C) tanh x (D) None of these
 12   5 12 
(A) 1,  (B)  , 
 7  3 7  sin4 x  cos 4 x
Q.12 The function f(x) = is -
 5 12  x  tan x
(C)   ,  (D)  ,   (A) odd
 3 7 
(B) Even
(C) neither even nor odd
Q.5 Solution of (x – 1)2 (x + 4) < 0 is-
(D) odd and periodic
(A) (–, 1) (B) (–, –4)
(C) (–1, 4) (D) (1, 4) Q.13 A function is called even function if its graph is
symmetrical w.r.t.-
Q.6 Solution of (2x + 1) (x – 3) (x + 7) < 0 is- (A) origin (B) x = 0
(C) y = 0 (D) line y = x
 1  1 
(A) (– , –7)    , 3  (B) (– , – 7)   ,3 
Q.14 A function is called odd function if its graph is
 2  2 
symmetrical w.r.t.-
 1  (A) Origin (B) x = 0
(C) (–, 7)    , 3  (D) (–, –7)  (3, ) (C) y = 0 (D) line y = x
 2 
Q.15 The even function is-
Q.7 If x2 + 6x – 27 > 0 and x2 – 3x – 4 < 0, then- (A) f(x) = x2 (x2 +1) (B) f(x) = sin3 x + 2
(A) x > 3 (B) x < 4 (C) f(x) = x (x +1) (D) f(x) = tan x + c
7
(C) 3 < x < 4 (D) x =
2

Corporate Office: CP Tower, Road No.1, IPIA, Kota (Raj.), Ph: 0744-2434159 FUNCTION 2
Q.16 A function whose graph is symmetrical about Q.25 In the following which function is not
the y-axis is given by- periodic-
(A) tan 4x (B) cos 2x
(A) f(x) = loge (x + x2  1 )
(C) cos x2 (D) cos2x
(B) f(x + y) = f(x) + f(y) for all x, y  R
(C) f(x) = cos x + sin x Domain, Co-domain and range
of function
(D) None of these
1
Q.17 Which of the following is an even function ? Q.26 Domain of the function f(x) = is-
x
x2
a 1
(A) x (B) tan x (A) R (B) (–2,  )
ax  1 (C) [2,  ] (D) [0,  ]
ax  a x ax  1
(C) (D) Q.27 The domain where function f(x) = 2x2 – 1 and
2 ax  1
g(x) = 1 – 3x are equal, is-
Q.18 In the following, odd function is - (A) {1/2} (B) {2}
(A) cos x2 (B) (ex + 1)/(ex – 1) (C) {1/2,2} (D) {1/2,-2}
(C) x – |x|
2
(D) None of these
3x
Q.28 The domain of the function log is-
Q.19 The function f(x) = x – |x| is -
2 2
(A) an odd function (A) (3,  ) (B) (–  ,3)
(C) (0,3) (D) (–3,3)
(B) a rational function
(C) an even function Q.29 Domain of the function cos–1 (4x –1) is-
(D) None of these (A) (0,1/2) (B) [0,1/2]
(C) [1/2,2] (D) None of these
Questions
based on Periodic function Q.30 Domain of the function log |x2 – 9| is-
(A) R (B) R– [–3,3]
(C) R – {–3,3} (D) None of these
Q.20 The period of sin4 x + cos4 x is -
(A)  (B) /2
Q.31 The domain of the function-
(C) 2 (D) None of these
Q.21 The period of function |cos 2x| is - f (x) = x  1 + 6  x is-
(A) (B) /2 (A) (1,6) (B) [1,6]
(C) 4 (D) 2 (C) [1,  ) (D) (–  ,6]

F
Gx I
J Fx I
+ cos G Jis- The domain of the function f(x) = (2  2x  x2 )
Q.22 The period of function sin
H2 K H2 K Q.32
is -
(A) 4 (B) 6
(C) 12 (D) 24 (A) – 3 x  3
Q.23 The period of the function (B) – 1– 3  x  –1 + 3
f(x) = log cos 2x + tan 4x is -
(C) – 2  x  2
(A) /2 (B) 
(C) 2 (D) 2/5 (D) –2 + 3  x  –2– 3
1
Q.24 The period of the function f(x) = 2 cos (x–) Q.33 Domain of a function f(x) = sin–1 5x is-
3
F
G IJ
1 1 L
M O
1 1
N P
is -
(A) 6 (B) 4 H K
(A)  ,
5 5
(B)  ,
5 5Q
(C) 2 (D)  F
G I
HJ
1
(C) R (D) 0,
K
5

Corporate Office: CP Tower, Road No.1, IPIA, Kota (Raj.), Ph: 0744-2434159 FUNCTION 3
Q.34 If f : R+  R, f(x) = log x, then range of f is -
(A) R0 (B) R Q.43 The range of f : R+  R+, f(x) = ex is -
(C) R+ (D) None of these (A) (0,  ) (B) [1,  )
(C) (1,  ) (D) None of these

Q.35 The range of the function f : R  R, f(x) = tan–1 x Q.44 The range of f(x) = cos 2x – sin 2x contains
is- the set -
L
 
M O O
 L (A) [2,4] (B) [–1,1]
N P
(A)  ,
2 2Q P
Q M
(B)  ,
2 2N (C) [–2,2] (D) [–4,4]
(C) R (D) None of these | x|
 Q.45 If the domain of the function f(x) = be
x
Q.36 The range of f(x) = sin [x] is -
2 [3,7] then its range is-
(A) {–1,1} (B) {–1,0,1} (A) [–1,1] (B) {–1,1}
(C) {0,1} (D) [–1,1]
(C) {1} (D) {–1}

| x  3|
Q.37 Domain and range of f(x) = are
x3 1
respectively- Q.46 The domain of the function f(x) = is-
x  [ x]
(A) R, [–1,1] (B) R– {3}, {1,–1}
(C) R+, R (D) None of these (A) R (B) R–Z
(C) Z (D) None of these
Q.38 The domain of the function f(x) = sin 1/x is - Q.47 The range of the function
(A) R (B) R+
f(x) = 2 + x – [x–3] is-
(C) R0 (D) R– (A) [5,6] (B) [5,6)
(C) R (D) None of these
Q.39 Range of the function f(x) = 9 – 7 sin x is-
(A) (2,16) (B) [2,16]
Questions
(C) [–1,1] (D) (2,16] based on Value of function
Q.40 For real values of x, range of function
1 Q.48 If f(x) = log x, then f (x/y) equals-
y= is - (A) f(x) + f(y) (B) f(x) – f(y)
2  sin 3 x
(C) f(x) / f(y) (D) f(x) . f(y)
1 1
3   3  
(A) y 1 (B) – y 1
2x
Q.49 If f(x) =
1  x2
, then f (tan  ) equals-
1 1
(C) – > y > – 1 (D) > y > 1
3 3 (A) cot 2  (B) tan 2
(C) sec 2 (D) cos 2 
R
S 1, when x  Q
Q.50 If f(x) = ax, then f(x+ y) equals-
 R, f(x) =
Q.41 If f : R
T1, when x Q , then (A) f(x) + f(y) (B) f(x) – f(y)
image set of R under f is - (C) f(x) f(y) (D) f(x) /f(y)
(A) {1,1} (B) (–1,–1)
(C) {1,–1} (D) None of these Q.51 If f(x) = log x, then correct statement is-
(A) f(x + y) = f( x ) + f(y) (B) f(x + y) = f( x) . f(y)
Q.42 If f : R  R, f(x) = x2, then {x| f (x) = –1} equals- (C) f(xy) = f(x) + f(y) (D) f(xy) = f( x) . f(y)
(A) {–1,1} (B) {1}
(C)  (D) None of these x f (a / b)
Q.52 If f (x) = , then =
x1 f (b / a)
(A) ab (B) a/b
(C) b/a (D) 1

Corporate Office: CP Tower, Road No.1, IPIA, Kota (Raj.), Ph: 0744-2434159 FUNCTION 4
Q.53 If f(x) = 2 cos x + sin2 x, then f(2– x) equals- f ( xy)  f ( x / y)
(A) – f(x) (B) f(x) Q.61 If f(x) = cos (log x), then equals-
(C) – 2f(x) (D) 2f(x) f ( x)f ( y)
(A) 1 (B) –1

If f : R  R, f(x) =
R
S1, when x  Q
(C) 0 (D) 2
Q.54
T1, when x Q , then which
 Q.62 If f (x) = |x| + |x – 1|, then for 0 < x < 1, f (x)
of the following statement is wrong ? equals-

(A) f  2 = –1 (B) f() = –1


(A) 1
(C) 2x + 1
(B) –1
(D) 2x – 1

(C) f(e) = 1 (D) f d4 i = 1 | x|


Q.63 The function f(x) = , x > 0 is -
x
F
GI
H3 J
(A) 0 (B) 1
Q.55 If f(x) = 2 sin x, g(x) = cos2x, then (f + g)
K= (C) 2 (D) –2

2 3 1
(A) 1 (B) Q.64 If f : N  R+, f(x) = x , then the value of
4
f (25)
1 is -
(C) 3 + (D) None of these f (9)  f (16)
4
(A) 0 (B) 1
Q.56 If f : R  R , f(x) = 2x ; g : R  R, g(x) = x + 1, (C) 5/7 (D) 9/7
then (f .g) (2) equals -
(A) 12 (B) 6 Q.65 If f(x) = log ax, then f(ax) equals-
(C) 3 (D) None of these
(A) f(a) f(x) (B) 1+ f(x)
b( x  a) a( x  b) (C) f(x) (D) a f(x)
Q.57 If f(x) = + , then f(a + b) =
(b  a) (a  b)
Q.66 If f(x) = (ax – c)/(cx – a) = y, then f(y)
(A) f(a). f(b) (B) f(a) – f(b)
equals-
(C) f(a) /f(b) (D) f(a) + f(b)
(A) x (B) 1/x
x f (a) (C) 1 (D) 0
Q.58 If f( x) = then is equal to -
x1 f (a  1)
(A) f(–a) (B) f(1/a) Questions
Mapping
based on
Fa I
(D) f G J
(C) f(a2)
Ha  1K Q.67 If f : I  I,f (x) = x3+ 1, then f is -
(A) one - one but not onto
x( x  1) (B) onto but not one-one
Q.59 If f (x) = , then the value of f (x + 2) is-
2 (C) One-one onto
(D) None of these
( x  2)
(A) f (x) + f(x + 1) (B) f(x + 1)
x Q.68 Function f : R  R , f(x) = x |x| is -
(A) one-one but not onto
( x  1) ( x  2)
(C) f(x +1) (D) f(x +1) (B) onto but not one- one
2 2 (C) one-one onto
(D) neither one-one nor onto
Q.60 If f(x + ay, x – ay ) = axy, then f (x,y) equals-
x2
2
x y 2 2
x y 2 Q.69 f:R  R , f(x) = , is -
(A) (B) 1 x2
4 4 (A) many- one function (B) odd function
(C) x2 (D) y2 (C) one- one function (D) None of these

Corporate Office: CP Tower, Road No.1, IPIA, Kota (Raj.), Ph: 0744-2434159 FUNCTION 5
Q.80 Which of the following function is onto ?
1
Q.70 If f : R0  R0, f(x) = , then f is - (A) f : R  R ; f(x) = 3x
x
(A) one-one but not onto (B) f : R  R+; f(x) = e–x
(B) onto but not one-one (C) f: [0,  /2]  [–1,1]; f(x) = sin x
(C) neither one-one nor onto (D) f : R  R: f(x) = cosh x
(D) both one-one and onto
Q.81 Which of the following function defined from
Q.71 Function f : R  R, f(x) = x + |x| is R to R is onto ?
(A) one-one (B) onto (A) f(x) = |x| (B) f(x) = e–x
(C) one-one onto (D) None of these
(C) f(x) = x3 (D) f(x) = sin x.
O
 3 L
P
2 2 M
,
Q.72 Function f :
Q N R, f(x) = tan x is Q.82 If f :   , f(x) = x2 – x, then f is -
(A) one-one (B) onto (A) one-one onto (B) one-one into
(C) one-one onto (D) None of these
(C) many-one onto (D) many-one into
L
M 3 O
Q.73 Function f :
N2 , 2 P
Q [–1,1], f(x) = sin x is - Questions
(A) one-one (B) onto based on Composite function
(C) one-one onto (D) None of these
Q.83 If f(x) = 2x and g is identity function, then-
L1 3 O [–1,1], f(x) = cos x is
Function f : M,  P
(A) (fog) (x) = g(x) (B) (g + g) (x) = g(x)
Q.74
N2 2 Q (C) (fog) (x) = (g + g) (x) (D) None of these
(A) many-one onto (B) onto Q.84 gof exists, when-
(C) one-one onto (D) many one into (A) domain of f = domain of g
Q.75 If f : R  R, f(x) = ex + e–x, then f is - (B) co-domain of f = domain of g
(C) co-domain of g = domain of g
(A) one-one but not onto
(D) co-domain of g = co-domain of f
(B) onto but not one-one
(C) neither one-one nor onto Q.85 If f : R  R, f(x) = x2 + 2x – 3 and g : R  R,
(D) both one-one and onto g(x) = 3x – 4 , then the value of fog (x) is-
Q.76 If f : R  [–1,1], f(x) = sin x, then f is - (A) 3x2 + 6x – 13 (B) 9x2 –18x + 5
(C) (3x– 4) + 2x – 3
2
(D) None of these
(A) one-one onto (B) one-one into
(C) many-one onto (D) many-one into
Q.86 If f : R  R, f(x) = x2 – 5x + 4 and g : R  R,
Q.77 If f : R  R , f(x) = sin2 x + cos2 x , then f is - g(x) = log x , then the value of (gof) (2) is -
(A) one-one but not onto (A) 0 (B) 
(B) onto but not one-one (C) –  (D) Undefined
(C) neither one-one nor onto Q.87 If f : R+  R+,f(x) = x2+ 1/x2 and g : R+  R+,
(D) both one-one onto
g(x) = ex then (gof) (x) equals-
Q.78 Which of the following functions from Z to itself x2 1
are bijections ? (A) e x2  e x 2 (B) e  x2
(A) f(x) = x3 (B) f(x) = x + 2 e
(C) f(x) = 2x + 1 (D) f(x) = x2 + x (C) e2 x  e 2 x (D) e x2 . e x2
Q.79 Which of the following functions from Q.88 If f : R  R, g : R  R and f(x) = 3x + 4 and
A = {x: –1  x  1} to itself are bijections ? (gof) (x) = 2x – 1, then the value of g(x) is -
F
Gx I (A) 2x – 1 (B) 2x – 11
H2 J
x
(A) f(x) =
2
(B) g(x) = sin
K (C)
1
(2x – 11) (D) None of these
(C) h(x) = |x| (D) k(x) = x 2 3

Corporate Office: CP Tower, Road No.1, IPIA, Kota (Raj.), Ph: 0744-2434159 FUNCTION 6
Q.89 If f : R  R, g : R  R and g(x) = x + 3 and R
S1, when x  Q
(fof) ( ) will be-
(fog) (x) = (x + 3)2, then the value of f(–3) is -
(A) –9 (B) 0
Q.98 If f(x) =
(A) 2
T0, when x Q , then
(B) 0
(C) 9 (D) None of these
(C) 1 (D) Undefined
Q.90 If f(x) = ax + b and g(x) = cx + d, then y y
f(g(x)) = g(f(x)) is equivalent to- Q.99 If f(y) = , g(y) = , then
1  y2 1  y2
(A) f(a) = g(c) (B) f(b) = g(b) (fog)(y) equals-
(C) f(d) = g(b) (D) f(c) = g(a)
y y
1 x (A) 2 (B)
Q.91 If f : [0,1]  [0,1], f(x) = . g : [0,1]  [0,1], 1 y 1  y2
1 x
g(x) = 4x (1–x), then (fog) (x) equals- 1  y2
(C) y (D)
1  y2
1  4 x  4 x2 8 x (1  x)
(A) (B) Q.100 If f(x) = [x] and g(x) = cos (x), then the
1  4 x  4 x2 (1  x)2 range of gof is -
1  4 x  4 x2 (A) {0} (B) {–1,1}
(C) (D) None of these (C) {–1,0,1} (D) [–1,1]
1  4 x  4 x2

Questions
Q.92 If f, g, h are three functions in any set, then based on Inverse fucntion
wrong statement is -
(A) (fog)–1 = g–1 of –1 Q.101 If f : R  R, f(x) = x2 + 3, then pre- image of 2
(B) gof  fog under f is -
(C) (fog)oh = fo(goh) (A) {1,–1} (B) {1}
(D) (gof)–1 = g–1of –1
(C) {–1} (D) 

1 x Q.102 Which of the following functions has its inverse-


Q.93 If f(x) = , then f [f (sin)] equals -
1 x (A) f : R  R , f(x) = ax
(A) sin  (B) tan (/2) (B) f : R  R, f(x) = |x| + |x – 1|
(C) cot (/2) (D) cosec  (C) f : R0  R+, f(x) = |x|
(D) f : [, 2]  [–1,1], f(x) = cos x
Q.94 If f(x) = (a – x n)1/n, n  N, then f [f(x)] is equal to-
(A) 0 (B) x Q.103 If function f : R  R+, f(x) = 2x , then f –1 (x) will
(C) xn (D) (an – x)n be equal to-
(A) logx 2 (B) log2 (1/x)
 3x  x 3  (C) log2 x (D) None of these
 1 x   
Q.95 If f (x) = log   and g(x) =  1  3x 2  ,
 1 x    ex  e x
Q.104 The inverse of the function f(x) = +2
then f[g(x)] is equal to- e x  e x
(A) –f(x) (B) 3f(x) is given by -
(C) [f(x)]3 (D) None of these Fx  2I
(A) log G J
1/ 2
Fx  1I
(B) log G J
1/ 2

Q.96 If  (x) = x2 + 1 and  (x) = 3x, then  {  (x)}


Hx  1K Hx  1K
and  {  (x)} = F x I1/2
(C) log G J
Fx  1I1/2
(D) log G J
2
1
2
1
H2  xK H3  x K
(A) 32x+1, 3x (B) 32x+1, 3x 1
x 2 1 Q.105 If f : [1, )  [2, ) is given by ƒ(x) = x +
(C) 3 +1, 3
2x
(D) None of these x
then f–1(x) equals -
R
S1, when x  Q
4) x  x2  4 x
Q.97 If function f(x) =
T0, when x Q , (fof) ( (A)
2
(B)
1 x2
the value will be -
(A) 0 (B) 2 x  x2  4
(C) (D) 1 + x 2  4
(C) 1 (D) None of these 2
Corporate Office: CP Tower, Road No.1, IPIA, Kota (Raj.), Ph: 0744-2434159 FUNCTION 7
Q.106 If f(x) = loge(x + 1  x2 ), then f –1
(x) equals-

(A) log (x – 1  x2 )

ex  e x
(B)
2

ex  e x
(C)
2

ex  e x
(D)
ex  e x

Q.107 If f(x) = x3 – 1 and domain of f = {0,1,2,3},


then domain of f–1 is -
(A) {0,1,2,3}
(B) {1,0,–7,–26}
(C) {–1,0,7,26}
(D) {0,–1,–2,–3}

Q.108 If f(x) = {4 – (x – 7)3}1/5, then its inverse is-


(A) 7 – (4 – x5)1/3 (B) 7 – (4 + x5)1/3
(C) 7 + (4 – x5)1/3 (D) None of these

Q.109 If f : R  R, f(x) = ex and g : R  R,


g(x) = 3x – 2 , then the value of (fog)–1(x) is
equal to -
2  log x
(A) log (x – 2) (B)
3
F
Gx  3I
(C) log
H2 JK (D) None of these

Corporate Office: CP Tower, Road No.1, IPIA, Kota (Raj.), Ph: 0744-2434159 FUNCTION 8
LEVEL # 2
1
Q.1 If f(x) = x + , then - Q.11 Function f : R  R + , f(x) = x 2 + 2 and
x
(A) f(x2) = [f(x)]2 (B) f(x + y) = f(x) + f(y) F
G 1 IJ
(C) f(–x) = f(x) (D) f(1/x) = f(x)
g : R+
H K
 R, g(x) = 1  1  x then the value of
gof (2) is -
Q.2 If x is the radius of a circle and f(x) = x2, then
(A) 5/6 (B) 8/7
domain of f is -
(C) 1/6 (D) 6/5
(B) R (B) R+
(C) R ¯
(D) R0
Q.12 The period of function f (x) = |sin3 (x/2)| is
(A) 4  (B) 16 
1
Q.3 If f(x) = x2 – 3x + 1 and g(x) = , then (C) 2  (D) None of these
x2
domain of (f – g) is - Q.13 The inverse of the function y = logex is -
(A) R (B) R+ (A) 10x (B) 10–x
(C) R – {2} (D) None of these (C) ex
(D) e–x
Q.4 If f : R  R, f(x) = tan x, then pre-image of
–1 under f is - 1 x
Q.14 If f(x) = log , when – 1 < x1, x2< 1, then
R
S
(A) n 

n I
U
V R
S
(B) n 

n I
U
V
1 x
f(x1) + f(x2) equals-
T 4 W T 4 W F
x1  x2 IJ F
x1  x2 IJ
(C) {n  | n I } (D) None of these G
H K
(A) f 1  x x
1 2
G
H K
(B) f 1  x x
1 2

x2  2 x  1 Fx1  x2 IJ Fx1  x2 IJ
(C) f G (D) f G
Q.5 f(x) =
x2  3 x  2
is not defined for-
H1 x1x2 K H1 x1x2 K
(A) x = 2 (B) x = 1, 2
(C) x = 2,–1 (D) x = 0 Q.15 Function f : [–1,1]  R, f(x) = sin (  /2) x is -
(A) one-one (B) onto
Q.6 If f : R  R, f(x) = x3 + 3, and g : R  R,
(C) one-one onto (D) None of these
g(x) = 2x + 1, then f–1og–1(23) equals-
(A) 2 (B) 3 Q.16 If the domain of function f(x) = x2 – 6x + 7 is
(C) (14)1/3 (D) (15)1/3 (–  ,  ), then the range of function is -
Q.7 If f(x) = log x, g(x) = x3, then f[g(a)] + f [g(b)] is (A) (–  ,  ) (B) [–2,  )
equal to- (C) (–2,3) (D) (–  ,–2)
(A) f [g(a) + g(b)] (B) 3 f(ab)
(C) g [f(ab)] (D) g [f(a) + f(b)] Q.17 Function f : R  R, f(x) = [x] is -
(A) one-one onto (B) one-one into
Q.8 Function sin–1 x is defined in the interval- (C) many-one onto (D) many-one into
(A) (–1,1) (B) [0,1]
(C) [–1,0] (D) (–1,2) Q.18 If S be the set of all triangles and f : S  R +,
f (  ) = Area of  , then f is -
 (A) One-one onto (B) one-one into
Q.9 The interval for which sin–1 x + cos–1 x= (C) many-one onto (D) many-one into
2
holds-
(A) [0,  ) (B) [0,3] Q.19 If f : C  R , f(z) = |z|, then f is -
(C) [0,1] (D) [0,2] (A) one-one but not onto
(B) onto but not one-one
Q.10 The domain of the function f(x) = x! is - (C) neither one-one nor onto
(A) (0,  ) (B) N (D) both one-one and onto
(C) W (D) R+

Corporate Office: CP Tower, Road No.1, IPIA, Kota (Raj.), Ph: 0744-2434159 FUNCTION 9
L 1 , 1 O
If f : M
Q.29 If f(x) = x2 – x–2, then f(1/x) equals-
Q.20
N2 2 P Q [–1,1], f(x) = sin x, then f (A)
1
(B) –1/f(x)
is - f ( x)
(A) one-one (B) one-one onto (C) f(x) (D) – f(x)
(C) onto (D) None of these

Q.21 If f(x) = 1/x then f(a) – f(b) equals- Q.30 The domain of function
F
Gb  aI F ab I 1
(A) f
Hab JK (B) f G
Ha  b J
K f(x) = log (3  x) + x  2 is -
10
(A) [–2, 3) (B) [–2, 3) – {2}
Fab I
(C) f G J
Fa  b I
(D) f G J
(C) [–3, 2] (D) [–2, 3] – {2}
Hb  a K Ha  b K x3
Q.31 Domain of the function f(x) = is-
x , correct statement is - ( x  1) x2  4
Q.22 f(x) = cos
(A) (1,2)
(A) f(x) is periodic and its period = 2 (B) (–  , –2)  (2,  )
(B) f(x) is periodic and its period = 4  2 (C) (–  ,–2)  (1,  )
(D) (–  ,  ) – {1,  2}
(C) f(x) is periodic and its period = 
(D) f(x) is not periodic Q.32 Range of the function f(x) = sin2(x4) + cos2(x4)
is-
Q.23 If f be the greatest integer function and g be the (A) (–  ,  ) (B) {1}
modulus function, then (C) (–1,1) (D) (0,1)
F
G 5I
J F 5 IJ=
– (fog) G
(gof) 
H3 K H3 K Q.33 Let f : R  R be a function defined by
f(x) = x + x 2 , then f is-
(A) 1 (B) –1
(C) 2 (D) 4 (A) injective (B) surjective
(C) bijective (D) None of these
Q.24 The domain of function f(x) = log |log x| is-
(A) (0,  ) (B) (1,  )
(C) (0,1)  (1,  ) (D) (–  ,1) Q.34 If f (x) = e3x and g(x) =  n x, x > 0, then (fog) (x)
is equal to-
Q.25 Domain of the function tan–1 x + cos–1 x2 is - (A) 3x (B) x3
(A) R– [–1,1] (B) R– (–1,1) (C) log 3x (D) 3 log x
(C) (–1,1) (D) [–1,1]
Q.35 If f : R  R f(x) = cos (5x + 2) then the value
Q.26 Which of the following functions are equal ? of f –1(x) is -
(A) f(x) = x, g(x) = x
2 cos 1( x)  2
(A) (B) cos 1 ( x )  2
(B) f(x) = log x2 , g(x) = 2 log x 5
(C) f(x) = 1, g(x) = sin2x + cos2 x
cos 1( x)
(D) f(x) = x/x, g(x) = 1 (C) 2 (D) Does not exist
5
Q.27 If f : Q  Q, f(x)= 2x and g : Q  Q, R
|S4  x2 U
|
g(x) = x + 2, then (fog)–1(20) equals- Function f(x) = sin log (1  x) Vhas domain
(A) 10 (B) 12
Q.36
|T |W
(C) 8 (D) 6 (A) [–2,1) (B) [–2,1]
(C) (–2,1) (D) (–  ,1)
2 cosh x  sin2 x
Q.28 f(x) = is - Q.37 The domain of function
x2  1
f(x) = log (3x –1) + 2 log (x +1) is -
(A) an algebric function
(A) [1/3,  ) (B) [–1,1/3]
(B) a trigonometrical function
(C) (–1,1/3) (D) None of these
(C) an even function
(D) an implicit function
Corporate Office: CP Tower, Road No.1, IPIA, Kota (Raj.), Ph: 0744-2434159 FUNCTION 10
x
F
G 1I
H xJ
1
Q.38 If f(x) =
1  x2
, then (fofof) (x) is equal to- Q.44 Let f x 
K= x + x2 (x  0), then f(x) equals-
2

3x x (A) x2 – 2 (B) x2 –1
(A) (B) (C) x2 (D) None of these
1  x2 1  3 x2
Q.45 The graph of f(x) = – |x| is -
3x
(C) (D) None of these
1  x2

Q.39 Which one of the following graphs represents


the function y = 1+ |x| for all x  R ?
(A) (B)
(A)

(B)

(C) (D)

(C) (D) Q.46 If a2 + b2 + c2 = 1, then range of ab + bc + ca is-


(A) [–1/2,  ) (B) (0,  )
(C) [–1/2,1] (D) [1,  )

Q.40 If f (x) = x3 – x and g(x) = sin 2x, then - Q.47 If x = logabc, y = log b ca, and z = logcab, then
(A) g [f(1)] = 1 1 1 1
(B) f (g (/12)) = – 3/8 + + equals-
1 x 1 y 1 z
(C) g {f(2)} = sin 2
(A) 1 (B) x + y + z
(D) None of these
(C) abc (D) ab + bc + ca
1 1 Q.48 The range of 5 cos x – 12 sin x + 7 is -
Q.41 If f(x) = and g (x) = , then (A) [–6,20] (B) [–3,18]
x1 x 1
common domain of function is - (C) [–6,15] (D) None of these
(A) {x | x <1, x  R } Q.49 The domain of the function log 2 log 3 log 4(x)
(B) {x | x  0, x  1, x  R} is -
(C) {1} (A) (1,  ) (B) (2,  )
(D) {–1} (C) (3,  ) (D) (4,  )

Q.42 The natural domain of the real valued function

defined by f (x) = x2  1 + x2  1 is-


(A) 1 < x <  (B) –  < x < 
(C) –  < x <–1 (D) (–  ,  ) – (–1,1)

9  x2
Q.43 If f(x) = , then domain of f is -
sin 1(3  x)
(A) [2,3] (B) [2,3)
(C) (2,3] (D) None of these

Corporate Office: CP Tower, Road No.1, IPIA, Kota (Raj.), Ph: 0744-2434159 FUNCTION 11
LEVEL # 3
Q.1 The domain of definition of Q.7 The value of nI for which the function

 x 1  1 sin nx
log0.4   2 f(x) = has 4 as its period is -
f(x) = is– x
 x  5  x  36 sin  
n
(A) (x : x < 0, x  – 6}
(A) 2 (B) 3 (C) 4 (D) 5
(B) (x : x > 0, x  1, x  6}
(C) (x : x > 1, x  6} Q.8 If f(x) is an odd periodic function with period
(D) (x : x  1, x  6} 2, then f (4) equals to -
(A) 0 (B) 2
(C) 4 (D) –4
Q.2 The function f : R  R defined by
f (x) = (x – 1) (x – 2) (x – 3) is -
Q.9 Domain of the function
(A) one-one but not onto
1
 x 2 
(B) onto but not one-one f(x) = sin  log 5 is -
(C) both one and onto  5 

(D) neither one-one nor onto (A) [–5, –1]  [1, 5] (B) [–5, 5]
(C) (–5, –1)  (1, 5) (D) None of these

Q.3 Set A has 3 elements and set B has 4


elements. The number of injections that can 1 | x |
be defined from A to B is - Q.10 Domain of f(x) = is -
2 | x |
(A) 144 (B) 12 (A) R – [–2, 2]
(C) 24 (D) 64 (B) R – [–1, 1]
(C) [–1, 1]  (–, –2)  (2, )
Q.4 The number of bijective functions from set A (D) None of these
to itself when a contains 106 elements -
(A) 106 (B) (106)2
  
 4  x2   is -
(C) 106! (D) 1106 Q.11 Range of sin  log 
  1 x  
  
Q.5 Let A be a set containing 10 distinct (A) (–1, 1) (B) [–1, 1]
elements, then the total number of distinct (C) R (D) None of these
functions from A to A is -
(A) 10 ! (B) 1010
(C) 210 (D) 210 – 1 2
Q.12 If f(x) = 3 sin  x 2 , then values of f(x) lie
16
in -
Q.6 Let f : R  R be a function defined by
  
(A)  ,  (B) [–2, 2]
e | x|  e  x  4 4
f (x)  . Then -
e x  e x  3 
(C) 0 ,  (D) None of these
(A) f is a bijection  2
(B) f is an injection only
(C) f is a surjection only
Q.13 The function f (x) = cos (log (x + x 2  1 )) is-
(D) f is neither an injection nor a surjection
(A) even (B) odd
(C) constant (D) None of these
Corporate Office: CP Tower, Road No.1, IPIA, Kota (Raj.), Ph: 0744-2434159 FUNCTION 12
Q.14 The function f(x) = max. [1 – x, 1 + x, 2] Q.20 If [x] denote the greatest integer  x, the
x  R is equivalent to - domain of definition of function

 1  x , x  1 4  x2
 f (x) = is -
(A) f ( x )  2 ,  1  x  1 [ x]  2
 1 x, x  1 (A) (–, –2)  [–1, 2] (B) [0, 2]

(C) [–1, 2] (D) (0, 2)
 1  x , x  1
 Q.21 The function f : [–1/2, 1/2]  [–/2, /2]
(B) f ( x )  2 ,  1  x  1
 1 x , x  1 defined by f(x) = sin–1(3x – 4x3) is–
 (A) both one-one and onto
(B) neither one-one nor onto
1  x , x  1
 (C) onto but not one-one
(C) f ( x )  1,  1  x  1
(D) one-one but not onto
 1 x, x  1

(D) None of these Q.22 The function f satisfies the functiona equation
 x  59 
3f (x) + 2f    10 x  30 for all real x  1.
Q.15 The domain of the function f(x) = 9–xPx–5 is-  x 1 
(A) [5, 7] (B) {5, 6, 7} The value of f (7) is -
(C) {3, 4, 5, 6, 7} (D) None of these (A) 8 (B) 4
(C) –8 (D) 11
Q.16 The range of the function f(x) = 9–xPx–5 is -
(A) {1, 2, 3} (B) [1, 2] Q.23 The domain of the function
(C) {1, 2, 3, 4, 5} (D) None of these f (x) = log 3+x(x2 – 1) is -
(A) (–3, –1)  (1, )
Q.17 Domain of the function (B) [–3, –1)  [1, )
 (C) (–3, –2)  (–2, –1)  (1, )
 1  
f ( x )  log 2   log1 / 2 1    1 is - (D) [–3, –2)  (–2, –1)  [1, )
 4
x 
   
(A) (0, 1) (B) (0, 1] Assertion & Reason Type Question :-
(C) [1, ) (D) (1, )
All questions are Assertion & Reason type
Q.18 The period of f(x) = [sin 5x] + |cos 6x| is - questions. Each of these questions contains
two statements : Statement-I (Assertion) and

(A) (B)  Statement-2 (Reason). Answer these ques
2
tions from the following four option.
2 (A) Statement-1 is false. Statement-2 is true
(C) 2 (D)
5 (B) Statement-1 is true. Statement-2 is true;
Statement-2 is a correct explanation for
x x Statement-1
Q.19 Period of f (x) = sin x + tan + sin 2 +
2 2 (C) Statement-1 is true. Statement-2 is true;
x x x Statement-2 is not a correct explanation
tan + ... + sin n  1 + tan n is -
23 2 2 for Statement-1
(A)  (B) 2 (D) Statement-1 is true. Statement-2 is false

(C) 2n (D)
2n

Corporate Office: CP Tower, Road No.1, IPIA, Kota (Raj.), Ph: 0744-2434159 FUNCTION 13
Q.24 Statement-1 : The period of Passage :-
1 Let here we define f : R  [–1, 1] and
f(x) = sin 2x cos [2x] – cos 2x sin [2x] is g : R  [–1, 1]. Now f(x) = 2 cos2 x – 1,
2
Statement-2 : The period of x – [x] is 1 g(x) = cos 2x, h(x) = f(x) + g(x),
f(x)
I(x) = f(x) – g(x), j (x) = g(x) are 5 functions.
Q.25 Statement-1 :
If f(x) = |x – 1| + |x – 2| + |x – 3| On the basis of above information, answer
Where 2 < x < 3 is an identity function. the following questions-
Statement-2 : f : A  A defined by
f(x) = x is an identity function. Q.31 Which statement is correct-
(A) Period of f(x), g(x) and h(x) are same
Q.26 Statement-1 : f : R  R defined by 2
and value is
f(x) = sin x is a bijection 3
(B) Period of f(x), g(x) and h(x) makes
Statement-2 : If f is both one and onto it is

bijection the A.P. with common difference
4
Q. 27 Statement-1 : f : R  R is a function defined (C) Sum of periods of f(x), g(x) and
2x  1 h(x) is 3
by f(x) = . (D) None of these
3
–1 3x  1 Q.32 Which statement is correct regarding function
Then f (x) =
2 j(x) and I(x)-
Statement-2 : f(x) is not a bijection. (A) The domain of j(x) and I(x) are the
same
(B) Range of j(x) and I(x) are the same
Q.28 Statement-1 : If f is even function, g is odd (C) The union of domain of j(x) and I(x) are all
f real numbers
function then g , (g  0) is an odd function. (D) None of these

Statement-2 : If f(–x) = –f(x) for every x of its Q.33 If the solution of equation I(x) – g(x) = 0 are
domain, then f(x) is called an odd function and x1, x2, x3, .... xn when x  [0, 10] then which
if f(–x) = f(x) for every x of its domain, then f(x) option is correct-
(A) x 1 , x 2 , x 3 ... x n makes the A.P. with
is called an even function. common difference 
(B) Total no. of solutions of I (x) – g(x) = 0 is 20
Q.29 Statement 1 : Function f(x) = sinx + {x} is for x  [0, 10]
periodic with period 2 (C) Sum of all solutions of the given
equation is 100 in the interval [0, 10]
Statement 2 : sinx and {x} are both periodic
(D) (B) and (C) are correct
with period 2 and 1 respectively.
Q.34 If h : R  [–2, 2], then -
(A) h(x) is one-one function
2 (B) h(x) is one-one and onto function
Q.30 Statement 1 : y = f(x) = x  2x  4 , (C) h(x) is onto function
x 2  2x  5
x  R Range of f(x) is [3/4, 1) (D) h(x) is many one and into function
Q.35 Domain and range of j(x) respectively -
Statement 2 : (x – 1)2 = 4 y  3 . (A) R and {1}
1 y (B) R and {0, 1}
(C) R – {(2n + 1) /4}, n I and {1}
(D) R – {(2n + 1) /2}, n  I and {1}

Corporate Office: CP Tower, Road No.1, IPIA, Kota (Raj.), Ph: 0744-2434159 FUNCTION 14
LEVEL # 4
(Questions asked in Previous AIEEE & IIT-JEE)

SECTION - A Q.8 A function f from the set of natural numbers to


Q.1 Which of the following is not a periodic function - integers defined by
[AIEEE 2002]
 n 1
(A) sin 2x + cos x (B) cos x  , when n is odd
(C) tan 4x (D) log cos 2x f(n) =  2 n is
 , when n is even
 2
[AIEEE 2003]
Q.2 The period of sin2 x is- [AIEEE 2002]
(A) neither one-one nor onto
(A) /2 (B) 
(C) 3/2 (D) 2 (B) one-one but not onto
(C) onto but not one-one
Q.3 The function f : R  R defined by f(x) = sin x is- (D) one-one and onto both
[AIEEE-2002]
Q.9 The range of the function f(x) = 7– xPx–3 is-
(A) into (B) onto
[AIEEE 2004]
(C) one-one (D) many-one
(A) {1, 2, 3} (B) {1, 2, 3, 4, 5, 6}
(C) {1, 2,3,4} (D) {1, 2, 3, 4, 5}
2x
Q.4 The range of the function f(x) = , x  2 is -
2x
Q.10 If f : R  S, defined by f(x) = sin x – 3 cos x + 1,
[AIEEE-2002]
(A) R (B) R – {–1} is onto, then the interval of S is-
(C) R – {1} (D) R – {2} [AIEEE 2004]
(A) [0, 3] (B) [–1, 1]
(C) [0, 1] (D) [–1, 3]
Q.5 The function f(x) = log (x + x 2  1 ), is-
[AIEEE 2003] Q.11 The graph of the function y = f(x) is symmetrical
(A) neither an even nor an odd function about the line x = 2, then- [AIEEE 2004]
(B) an even function (A) f(x+ 2) = f(x – 2) (B) f(2 + x) = f(2 – x)
(C) an odd function (C) f(x) = f(–x) (D) f(x) = – f(–x)
(D) a periodic function

Q.6 Domain of definition of the function sin 1( x  3 )


Q.12 The domain of the function f(x) = is-
3 9  x2
f(x) = + log10 (x3 – x), is- [AIEEE 2003]
4  x2
[AIEEE 2004]
(A) (– 1, 0)  (1, 2)  (2, ) (A) [2,3] (B) [2,3)
(B) (1, 2)
(C) [1,2] (D) [1, 2)
(C) ( – 1, 0) (1, 2)
(D) (1, 2)  (2, )
Q.13 Let f : (–1, 1)  B, be a function defined by

Q.7 If f : R  R satisfies f(x+ y) = f(x) + f(y), for all 2x


f(x) = tan–1 , then f is both one-one and
n 1 x2
x, y  R and f(1) = 7, then  f (r ) is- onto when B is the interval - [AIEEE-2005]
r 1
[AIEEE 2003]    
(A)  0,  (B) 0, 
7n (n  1) 7n  2  2
(A) (B)
2 2      
7(n  1) (C)  ,  (D)   , 
 2 2  2 2
(C) (D) 7n (n+1)
2
Corporate Office: CP Tower, Road No.1, IPIA, Kota (Raj.), Ph: 0744-2434159 FUNCTION 15
Q.14 A real valued function f(x) satisfies the functional SECTION - B
equation f(x – y) = f(x) f(y) – f (a – x) f(a + y)
where a is a given constant and f(0) = 1, then 1  x 
Q.1 If function f(x) = – tan  2  ; (–1 < x < 1)
f(2a – x) is equal to - [AIEEE-2005] 2  
(A) –f(x) (B) f(x)
and g(x) = 3  4 x  4 x 2 , then the domain of
(C) f(a) + f(a – x) (D) f(–x)
gof is – [IIT 90]
 1 1
    (A) (–1, 1) (B)   2 , 2 
Q.15 The largest interval lying in  ,  for which  
2 2
the function is defined, is- [AIEEE 2007]  1  1 
(C)   1, 2  (D)   2 , 1
   
   
(A) [0, ] (B)  , 
2 2
Q.2 If f(x) = cos [2]x + cos [–]x, where [x]
    
(C)  ,  (D) 0,  stands for the greatest integer function, then
 4 2  2 [IIT 91]
Q.16 Let f : N  Y be a function defined as 
(A) f  2  = –1 (B) f () = 1
 
f(x) = 4x + 3 where Y = |y  N : y = 4x + 3 for

some x  N|. Show that f is invertible and its (C) f  4  = 2 (D) None of these
 
inverse is [AIEEE 2008] Q.3 The value of b and c for which the identity
f(x + 1) – f(x) = 8x + 3 is satisfied,
y3 y3
(A) g(y) = 4 + (B) g(y) = where f(x) = bx2 + cx + d, are [IIT 92]
4 4
y 3 3y  4 (A) b = 2, c = 1 (B) b = 4, c = –1
(C) g(y) = (D) g(y) = (C) b = –1, c = 4 (D) None
4 3

Q.17 For real x, let f(x) = x3 + 5x + 1, then - Q.4 Let f(x) = sin x and g(x) = ln |x|. If the ranges
of the compositie functions fog and gof are
[AIEEE 2009]
R1 and R2 respectively, then – [IIT 94]
(A) f is one – one but not onto R (A) R1 = {u : –1 < u < 1},
(B) f is onto R but not one – one R2 = {v : –  < v < 0}
(C) f is one – one and onto R (B) R1 = {u : – < u < 0},
(D) f is neither one – one nor onto R R2 = {v : – 1 < v < 1}
(C) R1 = {u : –1 < u < 1},
Q.18 Let f(x) = (x + 1)2 –1, x > –1 R2 = {v : –  < v < 0}
Statement – 1 : [AIEEE 2009] (D) R1 = {u : –1 < u < 1},
The set {x : f(x) = f–1(x)} = {0, –1}. R2 = {v : –  < v < 0}
Statement – 2 :
f is a bijection. Q.5 Let 2 sin2 x + 3 sin x – 2 > 0 and x2 – x –
(A) Statement -1 is true, Statement -2 is true; 2 < 0 (x is measured in radians). Then x lies
Statement -2 is a correct explanation for in the interval [IIT 94]
Statement -1
  5   5 
(B) Statement -1 is true, Statement -2 is true; (A)  6 , 6  (B)   1, 6 
Statement -2 is not a correct explanation    
for Statement -1.
 
(C) Statement -1 is true, Statement -2 is false. (C) (–1, 2) (D)  6 ,2 
 
(D) Statement -1 is false, Statement -2 is true.
Corporate Office: CP Tower, Road No.1, IPIA, Kota (Raj.), Ph: 0744-2434159 FUNCTION 16
Q.6 Let f(x) = (x + 1)2 – 1, (x > – 1). Then the
Q.11 If g(f(x)) = |sin x| and f(g(x)) = (sin x )2,
set S = {x : f(x) = f –1(x)} is – [IIT 95]
then [IIT 98]
(A) Empty
(A) f(x) = sin2 x, g(x) = x
(B) {0, –1}
(B) f(x) = sin x, g(x) = |x|
(C) {0, 1, –1}
 (C) f(x) = x2, g(x) = sin x
 3  i 3  3  i 3 
(D) 0,1, 2
,
2
 (D) f and g cannot be determined
 

Q.12 If f(x) = 3x – 5, then f–1 (x) [IIT 98]


Q.7 If f(1) = 1 and f(n + 1) = 2f(n) + 1 if n  1, 1
then f(n) is- [IIT 95] (A) is given by
3x  5
(A) 2n+1 (B) 2n x5
(B) is given by
(C) 2n – 1 (D) 2n–1 – 1 3
(C) does not exist because f is not one - one
(D) does not exist because f is not onto
Q.8 If f is an even function defined on the interval
(– 5, 5), then the real values of x satisfying Q.13 If the function f : [1, )  [1, ) is defined
 x 1 by f(x) = 2x(x–1) , then f–1 (x) is [IIT 99]
the equation f(x) = f   are-
 x  2  1
x ( x 1)

(A)  
 2
[IIT 96]

(A)
 1 5  3  5
,
(B)
1
2

1  1  4 log2 x 
2 2

(B)
 1 3  3  3
2
,
2
(C)
1
2

1  1  4 log2 x 
2 5 (D) not defined
(C)
2
(D) None of these Q.14 The domain of definition of the function y(x)
given by the equation 2x + 2y = 2 is –
[IIT Scr. 2000]
  
Q.9 Let f(x) = [x] sin   , where [.] denotes the (A) 0 < x < 1 (B) 0 < x < 1
 [ x  1] 
greatest integer function. The domain of f is ....... (C) – < x < 0 (D) – < x < 1
(A) {x  R| x  [–1, 0)}
(B) {x  R| x  [1, 0)}
Q.15 Let f() = sin (sin + sin 3), then f()
(C) {x  R| x  [–1, 0)} [IIT 2000]
(D) None of these [IIT 96]
(A)  0 only when   0
(B)  0 for all 
  (C)  0 for all real 
Q.10 If f(x) = sin2x + sin2  x  3  + cos x cos
  (D)  0 only when   0
  5
 x   and g   = 1, then (gof) (x) =
 3 4 Q.16 The number of solutions of log4 (x – 1) =
[IIT 96] log2 (x – 3) is – [IIT Scr. 2001]
(A) –2 (B) –1 (A) 3 (B) 1
(C) 2 (D) 1 (C) 2 (D) 0
Corporate Office: CP Tower, Road No.1, IPIA, Kota (Raj.), Ph: 0744-2434159 FUNCTION 17
x Q.22 Let function f : R  R be defined by
Q.17 Let f(x) = , x  – 1, then for what value
x 1 f(x) = 2x + sin x for x  R. Then f is–
of  f{f(x)} = x. [IIT Scr. 2001]
[IIT Scr. 2002]
(A) 2 (B) – 2 (A) one to one and onto
(C) 1 (D) –1 (B) one to one but NOT onto
(C) onto but NOT one to one
(D) neither one to one nor onto
log2 ( x  3)
Q.18 The domain of definition of f (x) =
x2  3 x  2 x
is – [IIT Scr. 2001] Q.23 Let f(x) = defined as [0, )  [ 0, ),
1 x
(A) R / { –2, –2} f(x) is– [IIT Scr.2003]
(B) (– 2, )
(A) one one & onto
(C) R/ {–1, –2, –3} (B) one- one but not onto
(D) (–3, ) / {–1, –2}
(C) not one-one but onto
(D) neither one-one nor onto
1
Q.19 If f : [1, )  [2, ) is given by f(x) = x +
x x2  x  2
then f–1 (x) equals – [IIT Scr. 2001] Q.24 Find the range of f(x) = is–
x2  x  1
[IIT Scr.2003]
x  x2  4 x
(A) (B)  11 
2 1  x2 (A) (1, ) (B) 1, 
 7
x  x2  4  7  7
(C) (D) 1 + x2  4
2 (C) 1,  (D) 1, 
 3  5

Q.20 Let g(x) = 1 + x – [x] and Q.25 Domain of f(x) = sin 1(2x )   / 6 is–
[IIT Scr.2003]
 1 ; x  0
  1 1  1 1
f(x) =  0 ; x  0 . Then for all x, f(g(x)) is (A)  , (B)  ,
1 ; x0  4 2   2 2 

 1 1  1 1
(C)  , (D)  ,
equal to :
 4 4   2 4 
(where [.] denotes the greatest integer
function): [IIT Scr. 2001] Q.26 Let f(x) = sinx + cos x & g(x) = x2 – 1, then
(A) x (B) 1 g(f(x)) will be invertible for the domain-
(C) f(x) (D) g(x) [IIT Scr.2004]
  
(A) x  0,  (B) x   , 
Q.21 Suppose f(x) = (x + 1)2 for x  – 1. If g(x) is  4 4
the function whose graph is the reflection of     
(C) x  0,  (D) x   , 0
the graph of f(x) with respect to the line  2  2
y = x, then g(x) equals– [IIT Scr. 2002]
x x  Q 0 x  Q
(A) – x – 1, x  0 Q.27 f (x)   ; g (x)  
0 x  Q x x  Q
1
(B) , x > – 1 then (f – g) is [IIT Scr.2005]
( x  1)2
(A) one-one , onto
(C) x 1, x  – 1 (B) neither one-one, nor onto
(C) one-one but not onto
(D) x – 1, x  0
(D) onto but not one-one

Corporate Office: CP Tower, Road No.1, IPIA, Kota (Raj.), Ph: 0744-2434159 FUNCTION 18
ANSWER KEY
LEVEL # 1
Q.No. 1 2 3 4 5 6 7 8 9 10 11 12 13 14 15 16 17 18 19 20
Ans. C A C B B A C D C C D A B A A D A B C B

Q.No. 21 22 23 24 25 26 27 28 29 30 31 32 33 34 35 36 37 38 39 40
Ans. B A B A C B D B B B B B B B B B B B B A
Q.No. 41 42 43 44 45 46 47 48 49 50 51 52 53 54 55 56 57 58 59 60
Ans. C C C B C B B B B C C B B C C A D C B B
Q.No. 61 62 63 64 65 66 67 68 69 70 71 72 73 74 75 76 77 78 79 80
Ans. D A B C B A A C A D D C C D C C C B B B
Q.No. 81 82 83 84 85 86 87 88 89 90 91 92 93 94 95 96 97 98 99 100
Ans. C D C B B D D C C C A D A B B C C C C B
Q.No. 101 102 103 104 105 106 107 108 109
Ans. D D C D A C C C B

LEVEL # 2
Q.No. 1 2 3 4 5 6 7 8 9 10 11 12 13 14 15 16 17 18 19 20
Ans. D B C A B A B B C C D C C A A B D C C B
Q.No. 21 22 23 24 25 26 27 28 29 30 31 32 33 34 35 36 37 38 39 40
Ans. C D A C D C C C D B B B A B D C D B C B
Q.No. 41 42 43 44 45 46 47 48 49
Ans. B D B A C C A A D

LEVEL # 3
Q.No. 1 2 3 4 5 6 7 8 9 10 11 12 13 14 15 16 17 18 19 20
Ans. C B C C B D A A A C B C A A B A A C C A

Q.No. 21 22 23 24 25 26 27 28 29 30 31 32 33 34 35
Ans. A B C B B A D B A B C C D C C

LEVEL # 4
SECTION - A
Q.No. 1 2 3 4 5 6 7 8 9 10 11 12 13 14 15 16 17 18
Ans. B B A,D B C A A D A D B B D A D C C B

SECTION - B
Q.No. 1 2 3 4 5 6 7 8 9 10 11 12 13 14 15
Ans. B A B D D B C A C D A B B D C
Q.No. 16 17 18 19 20 21 22 23 24 25 26 27
Ans. B D D A B D A B C A B A

Corporate Office: CP Tower, Road No.1, IPIA, Kota (Raj.), Ph: 0744-2434159 FUNCTION 19
FUNCTION

Preface

As you have gone through the theory part that consists of given fundamental principles,
definitions, concepts involved and solved problems. After going through theory part it
becomes necessary to solve the unsolved problems based on the concepts given. To
solve this purpose we are providing exercise part that comprises of various exercises
based on the theory. By solving various kinds of problems you can check your grasp on
the topic and can determine whether you have been able to find optimum depth in relevant
topic or not.

Students are advised to solve the questions of exercises (Levels # 1, 2, 3, 4) in the same
sequence or as directed by the faculty members, religiously and very carefully.

Level # 3 is not for foundation course students, it will be discussed in fresher & target
courses.

The list of exercises is as following :

Total No.of questions in Function are -

Level # 1 ........................................ 109


Level # 2 ........................................ 49
Level # 3 ........................................ 35
Level # 4 ........................................ 45

Total No. of questions......................................................... 238

Corporate Office: CP Tower, Road No.1, IPIA, Kota (Raj.), Ph: 0744-2434159 FUNCTION 1
LEVEL # 1
Questions
inequation Q.8 If x2 – 1  0 and x2 – x – 2  0, then x line in the
based on
interval/set
(A) (–1, 2) (B) (–1, 1)
2 (C) (1, 2) (D) {– 1}
Q.1 The inequality < 3 is true, when x belongs to-
x
2   2 Questions
Definition of function
(A)  ,   (B)     based on
3   3

2  Q.9 Which of the following relation is a function ?


(C)  ,    (–, 0) (D) none of these (A) {(1,4), (2,6), (1,5), (3,9)}
3  (B) {(3,3), (2,1), (1,2), (2,3)}
(C) {(1,2), (2,2,), (3,2), (4,2)}
(D) {(3,1), (3,2), (3,3), (3,4)}
x4
Q.2 < 2 is satisfied when x satisfies-
x 3 Q.10 If x, y  R, then which of the following rules is
(A) (–, 3) (10, ) (B) (3, 10) not a function-
(C) (–, 3) [10, ) (D) none of these (A) y = 9 –x2 (B) y = 2x2
(C) y = x – |x| (D) y = x2 + 1
x7
Q.3 Solution of > 2 is-
x3 Questions
(A) (–3, ) (B) (–, –13) based on Even and odd function
(C) (–13, –3) (D) none of these
Q.11 Which one of the following is not an odd
2x  3 function -
Q.4 Solution of  3 is- (A) sin x (B) tan x
3x  5
(C) tanh x (D) None of these
 12   5 12 
(A) 1,  (B)  , 
 7  3 7  sin4 x  cos 4 x
Q.12 The function f(x) = is -
 5 12  x  tan x
(C)   ,  (D)  ,   (A) odd
 3 7 
(B) Even
(C) neither even nor odd
Q.5 Solution of (x – 1)2 (x + 4) < 0 is-
(D) odd and periodic
(A) (–, 1) (B) (–, –4)
(C) (–1, 4) (D) (1, 4) Q.13 A function is called even function if its graph is
symmetrical w.r.t.-
Q.6 Solution of (2x + 1) (x – 3) (x + 7) < 0 is- (A) origin (B) x = 0
(C) y = 0 (D) line y = x
 1  1 
(A) (– , –7)    , 3  (B) (– , – 7)   ,3 
Q.14 A function is called odd function if its graph is
 2  2 
symmetrical w.r.t.-
 1  (A) Origin (B) x = 0
(C) (–, 7)    , 3  (D) (–, –7)  (3, ) (C) y = 0 (D) line y = x
 2 
Q.15 The even function is-
Q.7 If x2 + 6x – 27 > 0 and x2 – 3x – 4 < 0, then- (A) f(x) = x2 (x2 +1) (B) f(x) = sin3 x + 2
(A) x > 3 (B) x < 4 (C) f(x) = x (x +1) (D) f(x) = tan x + c
7
(C) 3 < x < 4 (D) x =
2

Corporate Office: CP Tower, Road No.1, IPIA, Kota (Raj.), Ph: 0744-2434159 FUNCTION 2
Q.16 A function whose graph is symmetrical about Q.25 In the following which function is not
the y-axis is given by- periodic-
(A) tan 4x (B) cos 2x
(A) f(x) = loge (x + x2  1 )
(C) cos x2 (D) cos2x
(B) f(x + y) = f(x) + f(y) for all x, y  R
(C) f(x) = cos x + sin x Domain, Co-domain and range
of function
(D) None of these
1
Q.17 Which of the following is an even function ? Q.26 Domain of the function f(x) = is-
x
x2
a 1
(A) x (B) tan x (A) R (B) (–2,  )
ax  1 (C) [2,  ] (D) [0,  ]
ax  a x ax  1
(C) (D) Q.27 The domain where function f(x) = 2x2 – 1 and
2 ax  1
g(x) = 1 – 3x are equal, is-
Q.18 In the following, odd function is - (A) {1/2} (B) {2}
(A) cos x2 (B) (ex + 1)/(ex – 1) (C) {1/2,2} (D) {1/2,-2}
(C) x – |x|
2
(D) None of these
3x
Q.28 The domain of the function log is-
Q.19 The function f(x) = x – |x| is -
2 2
(A) an odd function (A) (3,  ) (B) (–  ,3)
(C) (0,3) (D) (–3,3)
(B) a rational function
(C) an even function Q.29 Domain of the function cos–1 (4x –1) is-
(D) None of these (A) (0,1/2) (B) [0,1/2]
(C) [1/2,2] (D) None of these
Questions
based on Periodic function Q.30 Domain of the function log |x2 – 9| is-
(A) R (B) R– [–3,3]
(C) R – {–3,3} (D) None of these
Q.20 The period of sin4 x + cos4 x is -
(A)  (B) /2
Q.31 The domain of the function-
(C) 2 (D) None of these
Q.21 The period of function |cos 2x| is - f (x) = x  1 + 6  x is-
(A) (B) /2 (A) (1,6) (B) [1,6]
(C) 4 (D) 2 (C) [1,  ) (D) (–  ,6]

F
Gx I
J Fx I
+ cos G Jis- The domain of the function f(x) = (2  2x  x2 )
Q.22 The period of function sin
H2 K H2 K Q.32
is -
(A) 4 (B) 6
(C) 12 (D) 24 (A) – 3 x  3
Q.23 The period of the function (B) – 1– 3  x  –1 + 3
f(x) = log cos 2x + tan 4x is -
(C) – 2  x  2
(A) /2 (B) 
(C) 2 (D) 2/5 (D) –2 + 3  x  –2– 3
1
Q.24 The period of the function f(x) = 2 cos (x–) Q.33 Domain of a function f(x) = sin–1 5x is-
3
F
G IJ
1 1 L
M O
1 1
N P
is -
(A) 6 (B) 4 H K
(A)  ,
5 5
(B)  ,
5 5Q
(C) 2 (D)  F
G I
HJ
1
(C) R (D) 0,
K
5

Corporate Office: CP Tower, Road No.1, IPIA, Kota (Raj.), Ph: 0744-2434159 FUNCTION 3
Q.34 If f : R+  R, f(x) = log x, then range of f is -
(A) R0 (B) R Q.43 The range of f : R+  R+, f(x) = ex is -
(C) R+ (D) None of these (A) (0,  ) (B) [1,  )
(C) (1,  ) (D) None of these

Q.35 The range of the function f : R  R, f(x) = tan–1 x Q.44 The range of f(x) = cos 2x – sin 2x contains
is- the set -
L
 
M O O
 L (A) [2,4] (B) [–1,1]
N P
(A)  ,
2 2Q P
Q M
(B)  ,
2 2N (C) [–2,2] (D) [–4,4]
(C) R (D) None of these | x|
 Q.45 If the domain of the function f(x) = be
x
Q.36 The range of f(x) = sin [x] is -
2 [3,7] then its range is-
(A) {–1,1} (B) {–1,0,1} (A) [–1,1] (B) {–1,1}
(C) {0,1} (D) [–1,1]
(C) {1} (D) {–1}

| x  3|
Q.37 Domain and range of f(x) = are
x3 1
respectively- Q.46 The domain of the function f(x) = is-
x  [ x]
(A) R, [–1,1] (B) R– {3}, {1,–1}
(C) R+, R (D) None of these (A) R (B) R–Z
(C) Z (D) None of these
Q.38 The domain of the function f(x) = sin 1/x is - Q.47 The range of the function
(A) R (B) R+
f(x) = 2 + x – [x–3] is-
(C) R0 (D) R– (A) [5,6] (B) [5,6)
(C) R (D) None of these
Q.39 Range of the function f(x) = 9 – 7 sin x is-
(A) (2,16) (B) [2,16]
Questions
(C) [–1,1] (D) (2,16] based on Value of function
Q.40 For real values of x, range of function
1 Q.48 If f(x) = log x, then f (x/y) equals-
y= is - (A) f(x) + f(y) (B) f(x) – f(y)
2  sin 3 x
(C) f(x) / f(y) (D) f(x) . f(y)
1 1
3   3  
(A) y 1 (B) – y 1
2x
Q.49 If f(x) =
1  x2
, then f (tan  ) equals-
1 1
(C) – > y > – 1 (D) > y > 1
3 3 (A) cot 2  (B) tan 2
(C) sec 2 (D) cos 2 
R
S 1, when x  Q
Q.50 If f(x) = ax, then f(x+ y) equals-
 R, f(x) =
Q.41 If f : R
T1, when x Q , then (A) f(x) + f(y) (B) f(x) – f(y)
image set of R under f is - (C) f(x) f(y) (D) f(x) /f(y)
(A) {1,1} (B) (–1,–1)
(C) {1,–1} (D) None of these Q.51 If f(x) = log x, then correct statement is-
(A) f(x + y) = f( x ) + f(y) (B) f(x + y) = f( x) . f(y)
Q.42 If f : R  R, f(x) = x2, then {x| f (x) = –1} equals- (C) f(xy) = f(x) + f(y) (D) f(xy) = f( x) . f(y)
(A) {–1,1} (B) {1}
(C)  (D) None of these x f (a / b)
Q.52 If f (x) = , then =
x1 f (b / a)
(A) ab (B) a/b
(C) b/a (D) 1

Corporate Office: CP Tower, Road No.1, IPIA, Kota (Raj.), Ph: 0744-2434159 FUNCTION 4
Q.53 If f(x) = 2 cos x + sin2 x, then f(2– x) equals- f ( xy)  f ( x / y)
(A) – f(x) (B) f(x) Q.61 If f(x) = cos (log x), then equals-
(C) – 2f(x) (D) 2f(x) f ( x)f ( y)
(A) 1 (B) –1

If f : R  R, f(x) =
R
S1, when x  Q
(C) 0 (D) 2
Q.54
T1, when x Q , then which
 Q.62 If f (x) = |x| + |x – 1|, then for 0 < x < 1, f (x)
of the following statement is wrong ? equals-

(A) f  2 = –1 (B) f() = –1


(A) 1
(C) 2x + 1
(B) –1
(D) 2x – 1

(C) f(e) = 1 (D) f d4 i = 1 | x|


Q.63 The function f(x) = , x > 0 is -
x
F
GI
H3 J
(A) 0 (B) 1
Q.55 If f(x) = 2 sin x, g(x) = cos2x, then (f + g)
K= (C) 2 (D) –2

2 3 1
(A) 1 (B) Q.64 If f : N  R+, f(x) = x , then the value of
4
f (25)
1 is -
(C) 3 + (D) None of these f (9)  f (16)
4
(A) 0 (B) 1
Q.56 If f : R  R , f(x) = 2x ; g : R  R, g(x) = x + 1, (C) 5/7 (D) 9/7
then (f .g) (2) equals -
(A) 12 (B) 6 Q.65 If f(x) = log ax, then f(ax) equals-
(C) 3 (D) None of these
(A) f(a) f(x) (B) 1+ f(x)
b( x  a) a( x  b) (C) f(x) (D) a f(x)
Q.57 If f(x) = + , then f(a + b) =
(b  a) (a  b)
Q.66 If f(x) = (ax – c)/(cx – a) = y, then f(y)
(A) f(a). f(b) (B) f(a) – f(b)
equals-
(C) f(a) /f(b) (D) f(a) + f(b)
(A) x (B) 1/x
x f (a) (C) 1 (D) 0
Q.58 If f( x) = then is equal to -
x1 f (a  1)
(A) f(–a) (B) f(1/a) Questions
Mapping
based on
Fa I
(D) f G J
(C) f(a2)
Ha  1K Q.67 If f : I  I,f (x) = x3+ 1, then f is -
(A) one - one but not onto
x( x  1) (B) onto but not one-one
Q.59 If f (x) = , then the value of f (x + 2) is-
2 (C) One-one onto
(D) None of these
( x  2)
(A) f (x) + f(x + 1) (B) f(x + 1)
x Q.68 Function f : R  R , f(x) = x |x| is -
(A) one-one but not onto
( x  1) ( x  2)
(C) f(x +1) (D) f(x +1) (B) onto but not one- one
2 2 (C) one-one onto
(D) neither one-one nor onto
Q.60 If f(x + ay, x – ay ) = axy, then f (x,y) equals-
x2
2
x y 2 2
x y 2 Q.69 f:R  R , f(x) = , is -
(A) (B) 1 x2
4 4 (A) many- one function (B) odd function
(C) x2 (D) y2 (C) one- one function (D) None of these

Corporate Office: CP Tower, Road No.1, IPIA, Kota (Raj.), Ph: 0744-2434159 FUNCTION 5
Q.80 Which of the following function is onto ?
1
Q.70 If f : R0  R0, f(x) = , then f is - (A) f : R  R ; f(x) = 3x
x
(A) one-one but not onto (B) f : R  R+; f(x) = e–x
(B) onto but not one-one (C) f: [0,  /2]  [–1,1]; f(x) = sin x
(C) neither one-one nor onto (D) f : R  R: f(x) = cosh x
(D) both one-one and onto
Q.81 Which of the following function defined from
Q.71 Function f : R  R, f(x) = x + |x| is R to R is onto ?
(A) one-one (B) onto (A) f(x) = |x| (B) f(x) = e–x
(C) one-one onto (D) None of these
(C) f(x) = x3 (D) f(x) = sin x.
O
 3 L
P
2 2 M
,
Q.72 Function f :
Q N R, f(x) = tan x is Q.82 If f :   , f(x) = x2 – x, then f is -
(A) one-one (B) onto (A) one-one onto (B) one-one into
(C) one-one onto (D) None of these
(C) many-one onto (D) many-one into
L
M 3 O
Q.73 Function f :
N2 , 2 P
Q [–1,1], f(x) = sin x is - Questions
(A) one-one (B) onto based on Composite function
(C) one-one onto (D) None of these
Q.83 If f(x) = 2x and g is identity function, then-
L1 3 O [–1,1], f(x) = cos x is
Function f : M,  P
(A) (fog) (x) = g(x) (B) (g + g) (x) = g(x)
Q.74
N2 2 Q (C) (fog) (x) = (g + g) (x) (D) None of these
(A) many-one onto (B) onto Q.84 gof exists, when-
(C) one-one onto (D) many one into (A) domain of f = domain of g
Q.75 If f : R  R, f(x) = ex + e–x, then f is - (B) co-domain of f = domain of g
(C) co-domain of g = domain of g
(A) one-one but not onto
(D) co-domain of g = co-domain of f
(B) onto but not one-one
(C) neither one-one nor onto Q.85 If f : R  R, f(x) = x2 + 2x – 3 and g : R  R,
(D) both one-one and onto g(x) = 3x – 4 , then the value of fog (x) is-
Q.76 If f : R  [–1,1], f(x) = sin x, then f is - (A) 3x2 + 6x – 13 (B) 9x2 –18x + 5
(C) (3x– 4) + 2x – 3
2
(D) None of these
(A) one-one onto (B) one-one into
(C) many-one onto (D) many-one into
Q.86 If f : R  R, f(x) = x2 – 5x + 4 and g : R  R,
Q.77 If f : R  R , f(x) = sin2 x + cos2 x , then f is - g(x) = log x , then the value of (gof) (2) is -
(A) one-one but not onto (A) 0 (B) 
(B) onto but not one-one (C) –  (D) Undefined
(C) neither one-one nor onto Q.87 If f : R+  R+,f(x) = x2+ 1/x2 and g : R+  R+,
(D) both one-one onto
g(x) = ex then (gof) (x) equals-
Q.78 Which of the following functions from Z to itself x2 1
are bijections ? (A) e x2  e x 2 (B) e  x2
(A) f(x) = x3 (B) f(x) = x + 2 e
(C) f(x) = 2x + 1 (D) f(x) = x2 + x (C) e2 x  e 2 x (D) e x2 . e x2
Q.79 Which of the following functions from Q.88 If f : R  R, g : R  R and f(x) = 3x + 4 and
A = {x: –1  x  1} to itself are bijections ? (gof) (x) = 2x – 1, then the value of g(x) is -
F
Gx I (A) 2x – 1 (B) 2x – 11
H2 J
x
(A) f(x) =
2
(B) g(x) = sin
K (C)
1
(2x – 11) (D) None of these
(C) h(x) = |x| (D) k(x) = x 2 3

Corporate Office: CP Tower, Road No.1, IPIA, Kota (Raj.), Ph: 0744-2434159 FUNCTION 6
Q.89 If f : R  R, g : R  R and g(x) = x + 3 and R
S1, when x  Q
(fof) ( ) will be-
(fog) (x) = (x + 3)2, then the value of f(–3) is -
(A) –9 (B) 0
Q.98 If f(x) =
(A) 2
T0, when x Q , then
(B) 0
(C) 9 (D) None of these
(C) 1 (D) Undefined
Q.90 If f(x) = ax + b and g(x) = cx + d, then y y
f(g(x)) = g(f(x)) is equivalent to- Q.99 If f(y) = , g(y) = , then
1  y2 1  y2
(A) f(a) = g(c) (B) f(b) = g(b) (fog)(y) equals-
(C) f(d) = g(b) (D) f(c) = g(a)
y y
1 x (A) 2 (B)
Q.91 If f : [0,1]  [0,1], f(x) = . g : [0,1]  [0,1], 1 y 1  y2
1 x
g(x) = 4x (1–x), then (fog) (x) equals- 1  y2
(C) y (D)
1  y2
1  4 x  4 x2 8 x (1  x)
(A) (B) Q.100 If f(x) = [x] and g(x) = cos (x), then the
1  4 x  4 x2 (1  x)2 range of gof is -
1  4 x  4 x2 (A) {0} (B) {–1,1}
(C) (D) None of these (C) {–1,0,1} (D) [–1,1]
1  4 x  4 x2

Questions
Q.92 If f, g, h are three functions in any set, then based on Inverse fucntion
wrong statement is -
(A) (fog)–1 = g–1 of –1 Q.101 If f : R  R, f(x) = x2 + 3, then pre- image of 2
(B) gof  fog under f is -
(C) (fog)oh = fo(goh) (A) {1,–1} (B) {1}
(D) (gof)–1 = g–1of –1
(C) {–1} (D) 

1 x Q.102 Which of the following functions has its inverse-


Q.93 If f(x) = , then f [f (sin)] equals -
1 x (A) f : R  R , f(x) = ax
(A) sin  (B) tan (/2) (B) f : R  R, f(x) = |x| + |x – 1|
(C) cot (/2) (D) cosec  (C) f : R0  R+, f(x) = |x|
(D) f : [, 2]  [–1,1], f(x) = cos x
Q.94 If f(x) = (a – x n)1/n, n  N, then f [f(x)] is equal to-
(A) 0 (B) x Q.103 If function f : R  R+, f(x) = 2x , then f –1 (x) will
(C) xn (D) (an – x)n be equal to-
(A) logx 2 (B) log2 (1/x)
 3x  x 3  (C) log2 x (D) None of these
 1 x   
Q.95 If f (x) = log   and g(x) =  1  3x 2  ,
 1 x    ex  e x
Q.104 The inverse of the function f(x) = +2
then f[g(x)] is equal to- e x  e x
(A) –f(x) (B) 3f(x) is given by -
(C) [f(x)]3 (D) None of these Fx  2I
(A) log G J
1/ 2
Fx  1I
(B) log G J
1/ 2

Q.96 If  (x) = x2 + 1 and  (x) = 3x, then  {  (x)}


Hx  1K Hx  1K
and  {  (x)} = F x I1/2
(C) log G J
Fx  1I1/2
(D) log G J
2
1
2
1
H2  xK H3  x K
(A) 32x+1, 3x (B) 32x+1, 3x 1
x 2 1 Q.105 If f : [1, )  [2, ) is given by ƒ(x) = x +
(C) 3 +1, 3
2x
(D) None of these x
then f–1(x) equals -
R
S1, when x  Q
4) x  x2  4 x
Q.97 If function f(x) =
T0, when x Q , (fof) ( (A)
2
(B)
1 x2
the value will be -
(A) 0 (B) 2 x  x2  4
(C) (D) 1 + x 2  4
(C) 1 (D) None of these 2
Corporate Office: CP Tower, Road No.1, IPIA, Kota (Raj.), Ph: 0744-2434159 FUNCTION 7
Q.106 If f(x) = loge(x + 1  x2 ), then f –1
(x) equals-

(A) log (x – 1  x2 )

ex  e x
(B)
2

ex  e x
(C)
2

ex  e x
(D)
ex  e x

Q.107 If f(x) = x3 – 1 and domain of f = {0,1,2,3},


then domain of f–1 is -
(A) {0,1,2,3}
(B) {1,0,–7,–26}
(C) {–1,0,7,26}
(D) {0,–1,–2,–3}

Q.108 If f(x) = {4 – (x – 7)3}1/5, then its inverse is-


(A) 7 – (4 – x5)1/3 (B) 7 – (4 + x5)1/3
(C) 7 + (4 – x5)1/3 (D) None of these

Q.109 If f : R  R, f(x) = ex and g : R  R,


g(x) = 3x – 2 , then the value of (fog)–1(x) is
equal to -
2  log x
(A) log (x – 2) (B)
3
F
Gx  3I
(C) log
H2 JK (D) None of these

Corporate Office: CP Tower, Road No.1, IPIA, Kota (Raj.), Ph: 0744-2434159 FUNCTION 8
LEVEL # 2
1
Q.1 If f(x) = x + , then - Q.11 Function f : R  R + , f(x) = x 2 + 2 and
x
(A) f(x2) = [f(x)]2 (B) f(x + y) = f(x) + f(y) F
G 1 IJ
(C) f(–x) = f(x) (D) f(1/x) = f(x)
g : R+
H K
 R, g(x) = 1  1  x then the value of
gof (2) is -
Q.2 If x is the radius of a circle and f(x) = x2, then
(A) 5/6 (B) 8/7
domain of f is -
(C) 1/6 (D) 6/5
(B) R (B) R+
(C) R ¯
(D) R0
Q.12 The period of function f (x) = |sin3 (x/2)| is
(A) 4  (B) 16 
1
Q.3 If f(x) = x2 – 3x + 1 and g(x) = , then (C) 2  (D) None of these
x2
domain of (f – g) is - Q.13 The inverse of the function y = logex is -
(A) R (B) R+ (A) 10x (B) 10–x
(C) R – {2} (D) None of these (C) ex
(D) e–x
Q.4 If f : R  R, f(x) = tan x, then pre-image of
–1 under f is - 1 x
Q.14 If f(x) = log , when – 1 < x1, x2< 1, then
R
S
(A) n 

n I
U
V R
S
(B) n 

n I
U
V
1 x
f(x1) + f(x2) equals-
T 4 W T 4 W F
x1  x2 IJ F
x1  x2 IJ
(C) {n  | n I } (D) None of these G
H K
(A) f 1  x x
1 2
G
H K
(B) f 1  x x
1 2

x2  2 x  1 Fx1  x2 IJ Fx1  x2 IJ
(C) f G (D) f G
Q.5 f(x) =
x2  3 x  2
is not defined for-
H1 x1x2 K H1 x1x2 K
(A) x = 2 (B) x = 1, 2
(C) x = 2,–1 (D) x = 0 Q.15 Function f : [–1,1]  R, f(x) = sin (  /2) x is -
(A) one-one (B) onto
Q.6 If f : R  R, f(x) = x3 + 3, and g : R  R,
(C) one-one onto (D) None of these
g(x) = 2x + 1, then f–1og–1(23) equals-
(A) 2 (B) 3 Q.16 If the domain of function f(x) = x2 – 6x + 7 is
(C) (14)1/3 (D) (15)1/3 (–  ,  ), then the range of function is -
Q.7 If f(x) = log x, g(x) = x3, then f[g(a)] + f [g(b)] is (A) (–  ,  ) (B) [–2,  )
equal to- (C) (–2,3) (D) (–  ,–2)
(A) f [g(a) + g(b)] (B) 3 f(ab)
(C) g [f(ab)] (D) g [f(a) + f(b)] Q.17 Function f : R  R, f(x) = [x] is -
(A) one-one onto (B) one-one into
Q.8 Function sin–1 x is defined in the interval- (C) many-one onto (D) many-one into
(A) (–1,1) (B) [0,1]
(C) [–1,0] (D) (–1,2) Q.18 If S be the set of all triangles and f : S  R +,
f (  ) = Area of  , then f is -
 (A) One-one onto (B) one-one into
Q.9 The interval for which sin–1 x + cos–1 x= (C) many-one onto (D) many-one into
2
holds-
(A) [0,  ) (B) [0,3] Q.19 If f : C  R , f(z) = |z|, then f is -
(C) [0,1] (D) [0,2] (A) one-one but not onto
(B) onto but not one-one
Q.10 The domain of the function f(x) = x! is - (C) neither one-one nor onto
(A) (0,  ) (B) N (D) both one-one and onto
(C) W (D) R+

Corporate Office: CP Tower, Road No.1, IPIA, Kota (Raj.), Ph: 0744-2434159 FUNCTION 9
L 1 , 1 O
If f : M
Q.29 If f(x) = x2 – x–2, then f(1/x) equals-
Q.20
N2 2 P Q [–1,1], f(x) = sin x, then f (A)
1
(B) –1/f(x)
is - f ( x)
(A) one-one (B) one-one onto (C) f(x) (D) – f(x)
(C) onto (D) None of these

Q.21 If f(x) = 1/x then f(a) – f(b) equals- Q.30 The domain of function
F
Gb  aI F ab I 1
(A) f
Hab JK (B) f G
Ha  b J
K f(x) = log (3  x) + x  2 is -
10
(A) [–2, 3) (B) [–2, 3) – {2}
Fab I
(C) f G J
Fa  b I
(D) f G J
(C) [–3, 2] (D) [–2, 3] – {2}
Hb  a K Ha  b K x3
Q.31 Domain of the function f(x) = is-
x , correct statement is - ( x  1) x2  4
Q.22 f(x) = cos
(A) (1,2)
(A) f(x) is periodic and its period = 2 (B) (–  , –2)  (2,  )
(B) f(x) is periodic and its period = 4  2 (C) (–  ,–2)  (1,  )
(D) (–  ,  ) – {1,  2}
(C) f(x) is periodic and its period = 
(D) f(x) is not periodic Q.32 Range of the function f(x) = sin2(x4) + cos2(x4)
is-
Q.23 If f be the greatest integer function and g be the (A) (–  ,  ) (B) {1}
modulus function, then (C) (–1,1) (D) (0,1)
F
G 5I
J F 5 IJ=
– (fog) G
(gof) 
H3 K H3 K Q.33 Let f : R  R be a function defined by
f(x) = x + x 2 , then f is-
(A) 1 (B) –1
(C) 2 (D) 4 (A) injective (B) surjective
(C) bijective (D) None of these
Q.24 The domain of function f(x) = log |log x| is-
(A) (0,  ) (B) (1,  )
(C) (0,1)  (1,  ) (D) (–  ,1) Q.34 If f (x) = e3x and g(x) =  n x, x > 0, then (fog) (x)
is equal to-
Q.25 Domain of the function tan–1 x + cos–1 x2 is - (A) 3x (B) x3
(A) R– [–1,1] (B) R– (–1,1) (C) log 3x (D) 3 log x
(C) (–1,1) (D) [–1,1]
Q.35 If f : R  R f(x) = cos (5x + 2) then the value
Q.26 Which of the following functions are equal ? of f –1(x) is -
(A) f(x) = x, g(x) = x
2 cos 1( x)  2
(A) (B) cos 1 ( x )  2
(B) f(x) = log x2 , g(x) = 2 log x 5
(C) f(x) = 1, g(x) = sin2x + cos2 x
cos 1( x)
(D) f(x) = x/x, g(x) = 1 (C) 2 (D) Does not exist
5
Q.27 If f : Q  Q, f(x)= 2x and g : Q  Q, R
|S4  x2 U
|
g(x) = x + 2, then (fog)–1(20) equals- Function f(x) = sin log (1  x) Vhas domain
(A) 10 (B) 12
Q.36
|T |W
(C) 8 (D) 6 (A) [–2,1) (B) [–2,1]
(C) (–2,1) (D) (–  ,1)
2 cosh x  sin2 x
Q.28 f(x) = is - Q.37 The domain of function
x2  1
f(x) = log (3x –1) + 2 log (x +1) is -
(A) an algebric function
(A) [1/3,  ) (B) [–1,1/3]
(B) a trigonometrical function
(C) (–1,1/3) (D) None of these
(C) an even function
(D) an implicit function
Corporate Office: CP Tower, Road No.1, IPIA, Kota (Raj.), Ph: 0744-2434159 FUNCTION 10
x
F
G 1I
H xJ
1
Q.38 If f(x) =
1  x2
, then (fofof) (x) is equal to- Q.44 Let f x 
K= x + x2 (x  0), then f(x) equals-
2

3x x (A) x2 – 2 (B) x2 –1
(A) (B) (C) x2 (D) None of these
1  x2 1  3 x2
Q.45 The graph of f(x) = – |x| is -
3x
(C) (D) None of these
1  x2

Q.39 Which one of the following graphs represents


the function y = 1+ |x| for all x  R ?
(A) (B)
(A)

(B)

(C) (D)

(C) (D) Q.46 If a2 + b2 + c2 = 1, then range of ab + bc + ca is-


(A) [–1/2,  ) (B) (0,  )
(C) [–1/2,1] (D) [1,  )

Q.40 If f (x) = x3 – x and g(x) = sin 2x, then - Q.47 If x = logabc, y = log b ca, and z = logcab, then
(A) g [f(1)] = 1 1 1 1
(B) f (g (/12)) = – 3/8 + + equals-
1 x 1 y 1 z
(C) g {f(2)} = sin 2
(A) 1 (B) x + y + z
(D) None of these
(C) abc (D) ab + bc + ca
1 1 Q.48 The range of 5 cos x – 12 sin x + 7 is -
Q.41 If f(x) = and g (x) = , then (A) [–6,20] (B) [–3,18]
x1 x 1
common domain of function is - (C) [–6,15] (D) None of these
(A) {x | x <1, x  R } Q.49 The domain of the function log 2 log 3 log 4(x)
(B) {x | x  0, x  1, x  R} is -
(C) {1} (A) (1,  ) (B) (2,  )
(D) {–1} (C) (3,  ) (D) (4,  )

Q.42 The natural domain of the real valued function

defined by f (x) = x2  1 + x2  1 is-


(A) 1 < x <  (B) –  < x < 
(C) –  < x <–1 (D) (–  ,  ) – (–1,1)

9  x2
Q.43 If f(x) = , then domain of f is -
sin 1(3  x)
(A) [2,3] (B) [2,3)
(C) (2,3] (D) None of these

Corporate Office: CP Tower, Road No.1, IPIA, Kota (Raj.), Ph: 0744-2434159 FUNCTION 11
LEVEL # 3
Q.1 The domain of definition of Q.7 The value of nI for which the function

 x 1  1 sin nx
log0.4   2 f(x) = has 4 as its period is -
f(x) = is– x
 x  5  x  36 sin  
n
(A) (x : x < 0, x  – 6}
(A) 2 (B) 3 (C) 4 (D) 5
(B) (x : x > 0, x  1, x  6}
(C) (x : x > 1, x  6} Q.8 If f(x) is an odd periodic function with period
(D) (x : x  1, x  6} 2, then f (4) equals to -
(A) 0 (B) 2
(C) 4 (D) –4
Q.2 The function f : R  R defined by
f (x) = (x – 1) (x – 2) (x – 3) is -
Q.9 Domain of the function
(A) one-one but not onto
1
 x 2 
(B) onto but not one-one f(x) = sin  log 5 is -
(C) both one and onto  5 

(D) neither one-one nor onto (A) [–5, –1]  [1, 5] (B) [–5, 5]
(C) (–5, –1)  (1, 5) (D) None of these

Q.3 Set A has 3 elements and set B has 4


elements. The number of injections that can 1 | x |
be defined from A to B is - Q.10 Domain of f(x) = is -
2 | x |
(A) 144 (B) 12 (A) R – [–2, 2]
(C) 24 (D) 64 (B) R – [–1, 1]
(C) [–1, 1]  (–, –2)  (2, )
Q.4 The number of bijective functions from set A (D) None of these
to itself when a contains 106 elements -
(A) 106 (B) (106)2
  
 4  x2   is -
(C) 106! (D) 1106 Q.11 Range of sin  log 
  1 x  
  
Q.5 Let A be a set containing 10 distinct (A) (–1, 1) (B) [–1, 1]
elements, then the total number of distinct (C) R (D) None of these
functions from A to A is -
(A) 10 ! (B) 1010
(C) 210 (D) 210 – 1 2
Q.12 If f(x) = 3 sin  x 2 , then values of f(x) lie
16
in -
Q.6 Let f : R  R be a function defined by
  
(A)  ,  (B) [–2, 2]
e | x|  e  x  4 4
f (x)  . Then -
e x  e x  3 
(C) 0 ,  (D) None of these
(A) f is a bijection  2
(B) f is an injection only
(C) f is a surjection only
Q.13 The function f (x) = cos (log (x + x 2  1 )) is-
(D) f is neither an injection nor a surjection
(A) even (B) odd
(C) constant (D) None of these
Corporate Office: CP Tower, Road No.1, IPIA, Kota (Raj.), Ph: 0744-2434159 FUNCTION 12
Q.14 The function f(x) = max. [1 – x, 1 + x, 2] Q.20 If [x] denote the greatest integer  x, the
x  R is equivalent to - domain of definition of function

 1  x , x  1 4  x2
 f (x) = is -
(A) f ( x )  2 ,  1  x  1 [ x]  2
 1 x, x  1 (A) (–, –2)  [–1, 2] (B) [0, 2]

(C) [–1, 2] (D) (0, 2)
 1  x , x  1
 Q.21 The function f : [–1/2, 1/2]  [–/2, /2]
(B) f ( x )  2 ,  1  x  1
 1 x , x  1 defined by f(x) = sin–1(3x – 4x3) is–
 (A) both one-one and onto
(B) neither one-one nor onto
1  x , x  1
 (C) onto but not one-one
(C) f ( x )  1,  1  x  1
(D) one-one but not onto
 1 x, x  1

(D) None of these Q.22 The function f satisfies the functiona equation
 x  59 
3f (x) + 2f    10 x  30 for all real x  1.
Q.15 The domain of the function f(x) = 9–xPx–5 is-  x 1 
(A) [5, 7] (B) {5, 6, 7} The value of f (7) is -
(C) {3, 4, 5, 6, 7} (D) None of these (A) 8 (B) 4
(C) –8 (D) 11
Q.16 The range of the function f(x) = 9–xPx–5 is -
(A) {1, 2, 3} (B) [1, 2] Q.23 The domain of the function
(C) {1, 2, 3, 4, 5} (D) None of these f (x) = log 3+x(x2 – 1) is -
(A) (–3, –1)  (1, )
Q.17 Domain of the function (B) [–3, –1)  [1, )
 (C) (–3, –2)  (–2, –1)  (1, )
 1  
f ( x )  log 2   log1 / 2 1    1 is - (D) [–3, –2)  (–2, –1)  [1, )
 4
x 
   
(A) (0, 1) (B) (0, 1] Assertion & Reason Type Question :-
(C) [1, ) (D) (1, )
All questions are Assertion & Reason type
Q.18 The period of f(x) = [sin 5x] + |cos 6x| is - questions. Each of these questions contains
two statements : Statement-I (Assertion) and

(A) (B)  Statement-2 (Reason). Answer these ques
2
tions from the following four option.
2 (A) Statement-1 is false. Statement-2 is true
(C) 2 (D)
5 (B) Statement-1 is true. Statement-2 is true;
Statement-2 is a correct explanation for
x x Statement-1
Q.19 Period of f (x) = sin x + tan + sin 2 +
2 2 (C) Statement-1 is true. Statement-2 is true;
x x x Statement-2 is not a correct explanation
tan + ... + sin n  1 + tan n is -
23 2 2 for Statement-1
(A)  (B) 2 (D) Statement-1 is true. Statement-2 is false

(C) 2n (D)
2n

Corporate Office: CP Tower, Road No.1, IPIA, Kota (Raj.), Ph: 0744-2434159 FUNCTION 13
Q.24 Statement-1 : The period of Passage :-
1 Let here we define f : R  [–1, 1] and
f(x) = sin 2x cos [2x] – cos 2x sin [2x] is g : R  [–1, 1]. Now f(x) = 2 cos2 x – 1,
2
Statement-2 : The period of x – [x] is 1 g(x) = cos 2x, h(x) = f(x) + g(x),
f(x)
I(x) = f(x) – g(x), j (x) = g(x) are 5 functions.
Q.25 Statement-1 :
If f(x) = |x – 1| + |x – 2| + |x – 3| On the basis of above information, answer
Where 2 < x < 3 is an identity function. the following questions-
Statement-2 : f : A  A defined by
f(x) = x is an identity function. Q.31 Which statement is correct-
(A) Period of f(x), g(x) and h(x) are same
Q.26 Statement-1 : f : R  R defined by 2
and value is
f(x) = sin x is a bijection 3
(B) Period of f(x), g(x) and h(x) makes
Statement-2 : If f is both one and onto it is

bijection the A.P. with common difference
4
Q. 27 Statement-1 : f : R  R is a function defined (C) Sum of periods of f(x), g(x) and
2x  1 h(x) is 3
by f(x) = . (D) None of these
3
–1 3x  1 Q.32 Which statement is correct regarding function
Then f (x) =
2 j(x) and I(x)-
Statement-2 : f(x) is not a bijection. (A) The domain of j(x) and I(x) are the
same
(B) Range of j(x) and I(x) are the same
Q.28 Statement-1 : If f is even function, g is odd (C) The union of domain of j(x) and I(x) are all
f real numbers
function then g , (g  0) is an odd function. (D) None of these

Statement-2 : If f(–x) = –f(x) for every x of its Q.33 If the solution of equation I(x) – g(x) = 0 are
domain, then f(x) is called an odd function and x1, x2, x3, .... xn when x  [0, 10] then which
if f(–x) = f(x) for every x of its domain, then f(x) option is correct-
(A) x 1 , x 2 , x 3 ... x n makes the A.P. with
is called an even function. common difference 
(B) Total no. of solutions of I (x) – g(x) = 0 is 20
Q.29 Statement 1 : Function f(x) = sinx + {x} is for x  [0, 10]
periodic with period 2 (C) Sum of all solutions of the given
equation is 100 in the interval [0, 10]
Statement 2 : sinx and {x} are both periodic
(D) (B) and (C) are correct
with period 2 and 1 respectively.
Q.34 If h : R  [–2, 2], then -
(A) h(x) is one-one function
2 (B) h(x) is one-one and onto function
Q.30 Statement 1 : y = f(x) = x  2x  4 , (C) h(x) is onto function
x 2  2x  5
x  R Range of f(x) is [3/4, 1) (D) h(x) is many one and into function
Q.35 Domain and range of j(x) respectively -
Statement 2 : (x – 1)2 = 4 y  3 . (A) R and {1}
1 y (B) R and {0, 1}
(C) R – {(2n + 1) /4}, n I and {1}
(D) R – {(2n + 1) /2}, n  I and {1}

Corporate Office: CP Tower, Road No.1, IPIA, Kota (Raj.), Ph: 0744-2434159 FUNCTION 14
LEVEL # 4
(Questions asked in Previous AIEEE & IIT-JEE)

SECTION - A Q.8 A function f from the set of natural numbers to


Q.1 Which of the following is not a periodic function - integers defined by
[AIEEE 2002]
 n 1
(A) sin 2x + cos x (B) cos x  , when n is odd
(C) tan 4x (D) log cos 2x f(n) =  2 n is
 , when n is even
 2
[AIEEE 2003]
Q.2 The period of sin2 x is- [AIEEE 2002]
(A) neither one-one nor onto
(A) /2 (B) 
(C) 3/2 (D) 2 (B) one-one but not onto
(C) onto but not one-one
Q.3 The function f : R  R defined by f(x) = sin x is- (D) one-one and onto both
[AIEEE-2002]
Q.9 The range of the function f(x) = 7– xPx–3 is-
(A) into (B) onto
[AIEEE 2004]
(C) one-one (D) many-one
(A) {1, 2, 3} (B) {1, 2, 3, 4, 5, 6}
(C) {1, 2,3,4} (D) {1, 2, 3, 4, 5}
2x
Q.4 The range of the function f(x) = , x  2 is -
2x
Q.10 If f : R  S, defined by f(x) = sin x – 3 cos x + 1,
[AIEEE-2002]
(A) R (B) R – {–1} is onto, then the interval of S is-
(C) R – {1} (D) R – {2} [AIEEE 2004]
(A) [0, 3] (B) [–1, 1]
(C) [0, 1] (D) [–1, 3]
Q.5 The function f(x) = log (x + x 2  1 ), is-
[AIEEE 2003] Q.11 The graph of the function y = f(x) is symmetrical
(A) neither an even nor an odd function about the line x = 2, then- [AIEEE 2004]
(B) an even function (A) f(x+ 2) = f(x – 2) (B) f(2 + x) = f(2 – x)
(C) an odd function (C) f(x) = f(–x) (D) f(x) = – f(–x)
(D) a periodic function

Q.6 Domain of definition of the function sin 1( x  3 )


Q.12 The domain of the function f(x) = is-
3 9  x2
f(x) = + log10 (x3 – x), is- [AIEEE 2003]
4  x2
[AIEEE 2004]
(A) (– 1, 0)  (1, 2)  (2, ) (A) [2,3] (B) [2,3)
(B) (1, 2)
(C) [1,2] (D) [1, 2)
(C) ( – 1, 0) (1, 2)
(D) (1, 2)  (2, )
Q.13 Let f : (–1, 1)  B, be a function defined by

Q.7 If f : R  R satisfies f(x+ y) = f(x) + f(y), for all 2x


f(x) = tan–1 , then f is both one-one and
n 1 x2
x, y  R and f(1) = 7, then  f (r ) is- onto when B is the interval - [AIEEE-2005]
r 1
[AIEEE 2003]    
(A)  0,  (B) 0, 
7n (n  1) 7n  2  2
(A) (B)
2 2      
7(n  1) (C)  ,  (D)   , 
 2 2  2 2
(C) (D) 7n (n+1)
2
Corporate Office: CP Tower, Road No.1, IPIA, Kota (Raj.), Ph: 0744-2434159 FUNCTION 15
Q.14 A real valued function f(x) satisfies the functional SECTION - B
equation f(x – y) = f(x) f(y) – f (a – x) f(a + y)
where a is a given constant and f(0) = 1, then 1  x 
Q.1 If function f(x) = – tan  2  ; (–1 < x < 1)
f(2a – x) is equal to - [AIEEE-2005] 2  
(A) –f(x) (B) f(x)
and g(x) = 3  4 x  4 x 2 , then the domain of
(C) f(a) + f(a – x) (D) f(–x)
gof is – [IIT 90]
 1 1
    (A) (–1, 1) (B)   2 , 2 
Q.15 The largest interval lying in  ,  for which  
2 2
the function is defined, is- [AIEEE 2007]  1  1 
(C)   1, 2  (D)   2 , 1
   
   
(A) [0, ] (B)  , 
2 2
Q.2 If f(x) = cos [2]x + cos [–]x, where [x]
    
(C)  ,  (D) 0,  stands for the greatest integer function, then
 4 2  2 [IIT 91]
Q.16 Let f : N  Y be a function defined as 
(A) f  2  = –1 (B) f () = 1
 
f(x) = 4x + 3 where Y = |y  N : y = 4x + 3 for

some x  N|. Show that f is invertible and its (C) f  4  = 2 (D) None of these
 
inverse is [AIEEE 2008] Q.3 The value of b and c for which the identity
f(x + 1) – f(x) = 8x + 3 is satisfied,
y3 y3
(A) g(y) = 4 + (B) g(y) = where f(x) = bx2 + cx + d, are [IIT 92]
4 4
y 3 3y  4 (A) b = 2, c = 1 (B) b = 4, c = –1
(C) g(y) = (D) g(y) = (C) b = –1, c = 4 (D) None
4 3

Q.17 For real x, let f(x) = x3 + 5x + 1, then - Q.4 Let f(x) = sin x and g(x) = ln |x|. If the ranges
of the compositie functions fog and gof are
[AIEEE 2009]
R1 and R2 respectively, then – [IIT 94]
(A) f is one – one but not onto R (A) R1 = {u : –1 < u < 1},
(B) f is onto R but not one – one R2 = {v : –  < v < 0}
(C) f is one – one and onto R (B) R1 = {u : – < u < 0},
(D) f is neither one – one nor onto R R2 = {v : – 1 < v < 1}
(C) R1 = {u : –1 < u < 1},
Q.18 Let f(x) = (x + 1)2 –1, x > –1 R2 = {v : –  < v < 0}
Statement – 1 : [AIEEE 2009] (D) R1 = {u : –1 < u < 1},
The set {x : f(x) = f–1(x)} = {0, –1}. R2 = {v : –  < v < 0}
Statement – 2 :
f is a bijection. Q.5 Let 2 sin2 x + 3 sin x – 2 > 0 and x2 – x –
(A) Statement -1 is true, Statement -2 is true; 2 < 0 (x is measured in radians). Then x lies
Statement -2 is a correct explanation for in the interval [IIT 94]
Statement -1
  5   5 
(B) Statement -1 is true, Statement -2 is true; (A)  6 , 6  (B)   1, 6 
Statement -2 is not a correct explanation    
for Statement -1.
 
(C) Statement -1 is true, Statement -2 is false. (C) (–1, 2) (D)  6 ,2 
 
(D) Statement -1 is false, Statement -2 is true.
Corporate Office: CP Tower, Road No.1, IPIA, Kota (Raj.), Ph: 0744-2434159 FUNCTION 16
Q.6 Let f(x) = (x + 1)2 – 1, (x > – 1). Then the
Q.11 If g(f(x)) = |sin x| and f(g(x)) = (sin x )2,
set S = {x : f(x) = f –1(x)} is – [IIT 95]
then [IIT 98]
(A) Empty
(A) f(x) = sin2 x, g(x) = x
(B) {0, –1}
(B) f(x) = sin x, g(x) = |x|
(C) {0, 1, –1}
 (C) f(x) = x2, g(x) = sin x
 3  i 3  3  i 3 
(D) 0,1, 2
,
2
 (D) f and g cannot be determined
 

Q.12 If f(x) = 3x – 5, then f–1 (x) [IIT 98]


Q.7 If f(1) = 1 and f(n + 1) = 2f(n) + 1 if n  1, 1
then f(n) is- [IIT 95] (A) is given by
3x  5
(A) 2n+1 (B) 2n x5
(B) is given by
(C) 2n – 1 (D) 2n–1 – 1 3
(C) does not exist because f is not one - one
(D) does not exist because f is not onto
Q.8 If f is an even function defined on the interval
(– 5, 5), then the real values of x satisfying Q.13 If the function f : [1, )  [1, ) is defined
 x 1 by f(x) = 2x(x–1) , then f–1 (x) is [IIT 99]
the equation f(x) = f   are-
 x  2  1
x ( x 1)

(A)  
 2
[IIT 96]

(A)
 1 5  3  5
,
(B)
1
2

1  1  4 log2 x 
2 2

(B)
 1 3  3  3
2
,
2
(C)
1
2

1  1  4 log2 x 
2 5 (D) not defined
(C)
2
(D) None of these Q.14 The domain of definition of the function y(x)
given by the equation 2x + 2y = 2 is –
[IIT Scr. 2000]
  
Q.9 Let f(x) = [x] sin   , where [.] denotes the (A) 0 < x < 1 (B) 0 < x < 1
 [ x  1] 
greatest integer function. The domain of f is ....... (C) – < x < 0 (D) – < x < 1
(A) {x  R| x  [–1, 0)}
(B) {x  R| x  [1, 0)}
Q.15 Let f() = sin (sin + sin 3), then f()
(C) {x  R| x  [–1, 0)} [IIT 2000]
(D) None of these [IIT 96]
(A)  0 only when   0
(B)  0 for all 
  (C)  0 for all real 
Q.10 If f(x) = sin2x + sin2  x  3  + cos x cos
  (D)  0 only when   0
  5
 x   and g   = 1, then (gof) (x) =
 3 4 Q.16 The number of solutions of log4 (x – 1) =
[IIT 96] log2 (x – 3) is – [IIT Scr. 2001]
(A) –2 (B) –1 (A) 3 (B) 1
(C) 2 (D) 1 (C) 2 (D) 0
Corporate Office: CP Tower, Road No.1, IPIA, Kota (Raj.), Ph: 0744-2434159 FUNCTION 17
x Q.22 Let function f : R  R be defined by
Q.17 Let f(x) = , x  – 1, then for what value
x 1 f(x) = 2x + sin x for x  R. Then f is–
of  f{f(x)} = x. [IIT Scr. 2001]
[IIT Scr. 2002]
(A) 2 (B) – 2 (A) one to one and onto
(C) 1 (D) –1 (B) one to one but NOT onto
(C) onto but NOT one to one
(D) neither one to one nor onto
log2 ( x  3)
Q.18 The domain of definition of f (x) =
x2  3 x  2 x
is – [IIT Scr. 2001] Q.23 Let f(x) = defined as [0, )  [ 0, ),
1 x
(A) R / { –2, –2} f(x) is– [IIT Scr.2003]
(B) (– 2, )
(A) one one & onto
(C) R/ {–1, –2, –3} (B) one- one but not onto
(D) (–3, ) / {–1, –2}
(C) not one-one but onto
(D) neither one-one nor onto
1
Q.19 If f : [1, )  [2, ) is given by f(x) = x +
x x2  x  2
then f–1 (x) equals – [IIT Scr. 2001] Q.24 Find the range of f(x) = is–
x2  x  1
[IIT Scr.2003]
x  x2  4 x
(A) (B)  11 
2 1  x2 (A) (1, ) (B) 1, 
 7
x  x2  4  7  7
(C) (D) 1 + x2  4
2 (C) 1,  (D) 1, 
 3  5

Q.20 Let g(x) = 1 + x – [x] and Q.25 Domain of f(x) = sin 1(2x )   / 6 is–
[IIT Scr.2003]
 1 ; x  0
  1 1  1 1
f(x) =  0 ; x  0 . Then for all x, f(g(x)) is (A)  , (B)  ,
1 ; x0  4 2   2 2 

 1 1  1 1
(C)  , (D)  ,
equal to :
 4 4   2 4 
(where [.] denotes the greatest integer
function): [IIT Scr. 2001] Q.26 Let f(x) = sinx + cos x & g(x) = x2 – 1, then
(A) x (B) 1 g(f(x)) will be invertible for the domain-
(C) f(x) (D) g(x) [IIT Scr.2004]
  
(A) x  0,  (B) x   , 
Q.21 Suppose f(x) = (x + 1)2 for x  – 1. If g(x) is  4 4
the function whose graph is the reflection of     
(C) x  0,  (D) x   , 0
the graph of f(x) with respect to the line  2  2
y = x, then g(x) equals– [IIT Scr. 2002]
x x  Q 0 x  Q
(A) – x – 1, x  0 Q.27 f (x)   ; g (x)  
0 x  Q x x  Q
1
(B) , x > – 1 then (f – g) is [IIT Scr.2005]
( x  1)2
(A) one-one , onto
(C) x 1, x  – 1 (B) neither one-one, nor onto
(C) one-one but not onto
(D) x – 1, x  0
(D) onto but not one-one

Corporate Office: CP Tower, Road No.1, IPIA, Kota (Raj.), Ph: 0744-2434159 FUNCTION 18
ANSWER KEY
LEVEL # 1
Q.No. 1 2 3 4 5 6 7 8 9 10 11 12 13 14 15 16 17 18 19 20
Ans. C A C B B A C D C C D A B A A D A B C B

Q.No. 21 22 23 24 25 26 27 28 29 30 31 32 33 34 35 36 37 38 39 40
Ans. B A B A C B D B B B B B B B B B B B B A
Q.No. 41 42 43 44 45 46 47 48 49 50 51 52 53 54 55 56 57 58 59 60
Ans. C C C B C B B B B C C B B C C A D C B B
Q.No. 61 62 63 64 65 66 67 68 69 70 71 72 73 74 75 76 77 78 79 80
Ans. D A B C B A A C A D D C C D C C C B B B
Q.No. 81 82 83 84 85 86 87 88 89 90 91 92 93 94 95 96 97 98 99 100
Ans. C D C B B D D C C C A D A B B C C C C B
Q.No. 101 102 103 104 105 106 107 108 109
Ans. D D C D A C C C B

LEVEL # 2
Q.No. 1 2 3 4 5 6 7 8 9 10 11 12 13 14 15 16 17 18 19 20
Ans. D B C A B A B B C C D C C A A B D C C B
Q.No. 21 22 23 24 25 26 27 28 29 30 31 32 33 34 35 36 37 38 39 40
Ans. C D A C D C C C D B B B A B D C D B C B
Q.No. 41 42 43 44 45 46 47 48 49
Ans. B D B A C C A A D

LEVEL # 3
Q.No. 1 2 3 4 5 6 7 8 9 10 11 12 13 14 15 16 17 18 19 20
Ans. C B C C B D A A A C B C A A B A A C C A

Q.No. 21 22 23 24 25 26 27 28 29 30 31 32 33 34 35
Ans. A B C B B A D B A B C C D C C

LEVEL # 4
SECTION - A
Q.No. 1 2 3 4 5 6 7 8 9 10 11 12 13 14 15 16 17 18
Ans. B B A,D B C A A D A D B B D A D C C B

SECTION - B
Q.No. 1 2 3 4 5 6 7 8 9 10 11 12 13 14 15
Ans. B A B D D B C A C D A B B D C
Q.No. 16 17 18 19 20 21 22 23 24 25 26 27
Ans. B D D A B D A B C A B A

Corporate Office: CP Tower, Road No.1, IPIA, Kota (Raj.), Ph: 0744-2434159 FUNCTION 19

You might also like